Jump to content

Wikipedia:Reference desk/Science

From Wikipedia, the free encyclopedia

This is an old revision of this page, as edited by Aaron hart (talk | contribs) at 09:48, 3 September 2007 (→‎Drink mixing). The present address (URL) is a permanent link to this revision, which may differ significantly from the current revision.

Wikipedia:Reference desk/headercfg

August 26

Prozac and Weight Change

I was wondering what Prozac does to cause weight loss and weight gain in some people. Is it from a change in appetite or does it have to do with something else? Which is more common? Weight loss or gain?

Prozac is a psychoactive drug. "Weight loss" and "wieght gain" are terms usually used to describe human behaviors. Prozac can have major effects on human behavior. I cannot cite any specific studies, but I I would guess that any effect on weight is a secondary effect. For example, Prozac is apparently used to treat bulemia nervosa. -Arch dude 03:54, 26 August 2007 (UTC)[reply]
I don't know either but I think the answer is easy to guess. Change in appetite is a very common effect in depression. I suppose everybody knows people who start eating a lot when they are depressed or stressed or whatever, and other people who eat less and less in such times. So, given that our appetite is so much coupled to our mood, shouldn't it be obvious that any mood-influencing drug changes appetite and hence over time body weight? Simon A. 10:28, 26 August 2007 (UTC)[reply]
Yes many depressed people tend to eat more to comfort themselves and gain weight, but many also gain weight rather then lose weight when taking prozac. I don't think the answer is that easy.
I gained thirty pounds while taking Prozac in 1996, and forty more over the next two years. During that time I had no acute depressive episodes, and they came back when I stopped gaining weight, or vice versa. Draw your own conclusions. —Tamfang 07:37, 27 August 2007 (UTC)[reply]

Psychology

I am currently in grade 12 and thinking of entering the field of psychology, specifically criminal psychology and I was wondering what route I should take regarding schooling. And what schools in Canada would be best? 75.154.106.228 04:47, 26 August 2007 (UTC)[reply]

Don't make hasty decisions, just because you think CSI or Criminal Intent or whatever it is on TV these day is cool doesn't necessarily mean it's true (if you are one of the millions of 'kids' that decide your career on TV programmes). --antilivedT | C | G 05:54, 26 August 2007 (UTC)[reply]
Don't condescend. He/she's a senior in high school and asked a well-directed question, I think we can afford to take it seriously. Anyway CSI is all about forensic investigation, not criminal psychology, if I recall, so if you are going to cast aspersions about his/her motivations, you might as well get the reference right! --24.147.86.187 13:05, 27 August 2007 (UTC)[reply]

I have not made this career choice because of shows on TV I have never watched any of the CSI shows or anything like that. I have done extensive research on this career, and chose it because it interested me.

You'll presumably want a general sciences background, with perhaps some pre-law stuff thrown in. Keeping your education focused on fundamentals will afford you the greatest flexibility later on. As for schools, I personally would go to University of British Columbia, as it's a great school in a world-class city. --Sean 19:47, 26 August 2007 (UTC)[reply]

I don't think there's anything wrong to having a goal for your education to a specific goal career — we can't all be generalists, and most people who graduate from undergrad these days have so inspecific of skills that they can't do anything without an additional year or two of education. Unfortunately I don't know the specifics of what that particular career requires, and I'm averse to just speculating on it. My recommendation would be to find a university with a criminal psychology program (even one you are not planning to go to) and try to contact someone there for advice. Someone "on the inside" will be able to give you good advice as to what sorts of backgrounds are more desirable, what level of education is normal for a practitioner (does it require a master's degree?), and what universities have good programs for that sort of goal. --24.147.86.187 13:03, 27 August 2007 (UTC)[reply]

Two general routes to becoming a shrink; one via MD one not. If you don't have an MD, you're not allowed to prescribe drugs, which is kind of a big part of the biz these days. But, not everybody is cut out for med school, for sure. They have to send patients out to an MD to do the drug prescribing and periodic monitoring and so on. Anyway, something to think about when you're sort of thinking through your plans. Gzuckier 14:48, 27 August 2007 (UTC)[reply]
  • The MacLeans survey might help (most of it requires a subscription, sadly). --M@rēino 17:24, 27 August 2007 (UTC)[reply]

You might want to seriously consider something fairly local since it'll likely be cheaper and easier for you. I'm not a Canadian but I would assume most Canadian schools/universities are fairly decent from a general perspective. For a more specific perspective some might be better then others but often the student matters more then the university. As for what route, I would suggest you speak to a career counsellor in your current school and probably one in some of the prospective universities as well. The fact that you have a specific goal in mind makes it a lot easier since a lot of people have no idea. Also, you can likely find information on the university websites. Finally since your job is likely to be with your law enforcement, you might want to contact them for advice and particularly see if you can get in contact with someone working in the field to try and help you decide whether it's really the job for you (they can likely offer you advice on universities and stuff too). Nil Einne 00:51, 28 August 2007 (UTC)[reply]

Actually on second thought you don't really have to be working with law enforcement that much. You work could be more in the academic area so you could try talking to someone in that area too Nil Einne 00:59, 28 August 2007 (UTC)[reply]

Newtonian camera

Would it be possible to make a camera (lens) that uses the same principle as a newtonian telescope? I imagine it could be done simpler, with a mirror behind the lens (if any) that reflects the light to the side, where the sensor then should be. That would be a way to make the camera flatter, a solution to a substantial practical problem, especially with the better lenses, which are always too bulky to always carry around. Actually, come to think of it, why aren't photo cameras made the way film cameras are, with the lens at the narrow end? That would also get rid of the sticking out lens. Actually, some cameras are moving in that direction, but why don't they take the final step? DirkvdM 09:09, 26 August 2007 (UTC)[reply]

partial answer - newtonian telescopes don't make very good zoom lenses...that said some lens do utilise mirrors - Catadioptric systems
traditionally photo cameras were that shape because of the shape of the negative+film roll..
For maximum stability of a camera holding it with two hands helps - it could be argued that the classic shape is better for two hands holding than a cine-camera shape?87.102.44.85 10:41, 26 August 2007 (UTC)[reply]
It's actually quite difficult to make a 'fast' Newtonian reflector – that is, one with a small focal ratio that will fit in a short tube – that produces good images. Consider the workhorse zoom telephoto lens on my camera. It gathers light through an opening 72 mm across, and focuses it onto a film plane only about 150 mm away. In that length, I also get all the optics I need to focus, control my depth of field, and zoom the apparent focal length from 28 mm to 80 mm. In contrast, a typical Newtonian reflector will tend to have a length that is at least four to six times its diameter (and often quite a bit more). The highly curved mirror needed for a short body is costly and hard to manufacture precisely. The particular aberrations associated with the Newtonian optics (particularly coma) are also worsened with short focal ratios.
Telescope manufacturers have adopted (at least) three strategies to deal with this. 'Slower' optics with longer focal lengths relative to the telescope's diameter reduce the effect of the aberrations, but at the cost of a longer instrument. In astronomy, ignoring the problem is actually often an acceptable solution—stuff in the middle of the field is bright and sharp, and any weirdness can be confined to material off at the edges. Finally, telescope makers have started to add lenses to reflecting telescopes: catadioptric systems. It's that final solution that most often appears in the world of non-astronomical photography. Nevertheless, it's still only used where the cost and weight of glass lenses would be prohibitively high. TenOfAllTrades(talk) 11:23, 26 August 2007 (UTC)[reply]
Mostly agree - but.. (note not technically a newtonian - but still a reflector) consider this http://www.zenit-camera.com/rubinar_1000_lens.htm a 1000mm lens - note the short length to width ratio (especially when compared with the equivalent 'glass' lens).. also consider the reduced weight (not such a big factor nowadays with plastic lens) - I just wanted to pick up on that "length that is at least four to six times its diameter" bit - newtonians can be better than glass in this respect due the 'folded' nature of the light path.(and can get very wide for good light collection..
OK I admit I'm just trying to sell this lens (also available in other fittings!).87.102.44.85 13:14, 26 August 2007 (UTC)[reply]
You need to be careful with your terminology—while all Newtonians are reflectors, not all reflectors are Newtonians. All commercial (as opposed to home-built) reflecting camera attachments will contain lens elements as well as reflecting elements, both to correct the aberrations of the basic Newtonian design and to greatly reduce the length of the instrument. The focal ratio caveat obviously doesn't apply to catadioptric systems.
Finally, you can't 'fold' the light path in a Newtonian system down to much less than its focal length. The closer the secondary mirror is to the primary's surface (and the further it is from the primary's focal point) the larger the secondary needs to be to capture all of the light. This, in turn, increases the size of the central obstruction, reducing the telescope's light-gathering ability. TenOfAllTrades(talk) 14:39, 26 August 2007 (UTC)[reply]
Interesting - looks like I meant a Schmidt-Cassegrain telescope or similar - in my ignorance I assumed it would be considered just a variation on the newtonian - thinking all mirrored telescopes/lens were newtonians. Oops!.87.102.11.213 16:51, 26 August 2007 (UTC)[reply]
My oops. When I said newtonian telescope I meant reflecting telescope. So the title should read 'reflecting camera'. DirkvdM 09:23, 27 August 2007 (UTC)[reply]
87.102.11.213, you said that reflective telescopes don't make very good zoom 'lenses', but I get the notion that the opposite is true (or did you mean that reflective telescopes have a limited range of zoom?). They're very good at zoom (after all, that's what telescopes do) and are much more portable than comparable lenses. But I just realise there might be another problem here if the secondary mirror is in the path of the light, as indicated by the macro-limit of the Zenit 'lens' linked to by you. In a telescope that mirror being in the way doesn't matter too much because the objects are huge and far away, so it will only affect the overall light intensity. But for objects closer by there is a more serious problem. Take macro photography. If the object is small and only a cm away from the camera then its light might be completely blocked out by the secondary mirror. That is an extreme example, but for objects not that close but still close the 'blockage' of light would still be unevenly distributed. Does that make sense and from which distance would it no longer be a (serious) problem?
That would be solved by not having the secondary mirror in the path of the light, but I remember from a previous thread here that that would cause more problems in constructing the primary mirror. Then again, maybe economics of scale might solve this if the cost is largely in the development of the technique and the production of the machine that makes the mirrors. Telescopes are never mass produced, but cameras are. Once you have a (polishing) machine that can make this type of mirror then the production of each individual mirror would not be so expensive, so it's just a matter of how many you can sell. Right? DirkvdM 09:23, 27 August 2007 (UTC)[reply]
Re - zoom - by zoom I meant the ability to change the focal length. I never heard of a 'zoom' telescope as such, but I know different eyepieces can change the magnification - is that the same.. Otherwise I agree the other points. I think the question is (in the case of a camera) why use mirrored optics in preference to lenses - especially when the abberations due the the second mirror are avoidable with a lens system..?87.102.45.106 11:45, 27 August 2007 (UTC)[reply]
disclaimer I'm no optics expert - it's probably the thing I know least about - I think you DirkvdM should ask a new question about the reasons why we are not using newtonians in cameras - I'd be interested to see a proper answer. 87.102.45.106 12:10, 27 August 2007 (UTC)[reply]

stepped transmission

why geometric progression of speeds is preferred in stepped transmission systems? thank you210.212.24.7 09:55, 26 August 2007 (UTC)[reply]

you mean like gear ratios 1:1,1:2,1:4,1:8 etc?
a geometric progression gives a wider range of gear ratios than a linear progression cf 1:1,1:2,1:3,1:4
So that enables both slow crawls and high speed driving.
There may another reason I've missed?87.102.44.85 11:42, 26 August 2007 (UTC)[reply]
It's not just automatic transmissions, the gear ratios (reverse, 1st, 2nd, ...etc) for four manual transmission cars I've owned recently are:
  • 1963 Austin Mini (37hp): 13.66, 13.66, 8.18, 5.32, 3.77
  • 2003 BMW MINI Cooper S (165hp): 11.13, 11.42, 7.18, 5.40, 4.40, 3.66, 2.99
  • 2005 BMW MINI Cooper S (172hp): 11.94, 12.79, 7.79, 5.65, 4.61, 3.83, 3.13
  • 2007 BMW MINI Cooper S (178hp): 11.78, 12.06, 7.77, 5.41, 4.15, 3.46, 2.97
All of them have the property that the difference between consecutive ratios reduces by smaller and smaller amounts as you get into higher gears. I believe this is because they wish to move the optimum shift points closer and closer together (in RPM terms) so that the car may be kept running within narrower and narrower torque bands - allowing you to employ more torque at higher speeds when air resistance becomes a significant factor. They also seem to have been chosen to provide efficient cruising at typical speed limits. If you look at the differences in gear ratios between 2003 and 2005 - and note that the 2005 model had just 7hp more than the 2003, you can see that careful selection of gear ratios to match small engine changes is sufficiently important to car manufacturers that even a 4% engine performance improvement warranted redesigning the gearbox! This tells you that there is a lot of science going into picking those ratios. SteveBaker 12:48, 26 August 2007 (UTC)[reply]

Building a wall or fence to block traffic noise

A noisy road has been built about 70 metres away from my house. I was wondering if I could reduce or remove the traffic noise by building a wall adjacent to my house so that I would have a small enclosed courtyard area that was open to the sky yet comparatively quiet.

The main determinant of how silent the courtyard would be, I think, would be the noise coming over the top of the wall and diffracting into the space beyond it. Thus even though below the height of the wall, you would still hear noise because of the sound waves diffracting from the top of the wall. Does anyone know how to calculate this effect? A taller wall would be better, but I'd like to estimate if an acceptable height of wall (say two metres) would still reduce the noise enough to be worth building.

I'd be grateful for any practical ideas about how to reduce the noise - for example trellisses that may reduce noise by acting as diffractors and creating destructive interference of the sound waves. Thanks. 80.2.200.132 10:33, 26 August 2007 (UTC)[reply]

I can't give you an equation - but can confirm that a wall (taller than you) will significantly cut the noise.
The other methods are pretty much as you describe - anything that can absorb a sound wave - anything solid - trees may help.
A wall would definately be 'worth building' the sound reduction effects would be significant. A hedge or row of trees would also help - pines grow fast and tall.87.102.44.85 11:46, 26 August 2007 (UTC)[reply]
This page says: A noise barrier can achieve a 5 dB noise level reduction, when it is tall enough to break the line-of-sight from the highway to the home or receiver. After it breaks the line-of-sight, it can achieve approximately 1.5dB of additional noise level reduction for each meter of barrier height". I've also seen rows of fast-growing cypress trees that are very dense, nice to look at, and apparently good for noise abatement. --Sean 12:08, 26 August 2007 (UTC)[reply]
My first guess would be that trees (especially the thick cypress variety) would form a better sound barrier than a brick or concrete wall. The trees will block the line-of-sight as effectively, but will muffle the sound better. Also, they can grow to very tall heights, which has been empirically shown (as above - 1.5 dB per meter of height) to reduce the noise significantly. Nimur 18:16, 26 August 2007 (UTC)[reply]
  • Another way to reduce noise is rubberized asphalt. If this is a new road built near existing houses, your community might be able to force the use of a surface of rubberized asphalt. --JWSchmidt 00:04, 27 August 2007 (UTC)[reply]
There are many street-noise blocking walls where I live. They are 12-15 feet tall, corrugated steel walls. Very ugly, but work well. -- Kainaw(what?) 00:23, 27 August 2007 (UTC)[reply]
I've studied sound breaking and pretty much what you'd need is a really dense, solid wall with lots of mass. Brick and concrete probably. Traffic noise is often low frequency so the wall would need to be a ways into the ground (I think--anyone know for sure??) and then I know it'd need to be high and pretty fully surround your home at least in the noise's direction. Traffic noise is worst as loud engines and loud car stereos. Juanita Hodges 01:34, 27 August 2007 (UTC)[reply]
This question has been asked before, last year I believe. Alas I don't know of an easy way to search for that. But 'the answer' I believe was a plant-barrier. This might have the added advantage that you will be allowed to make it taller than a wall. For an obvious reason - it's not as ugly. I don't know how the costs would compare, especially if you don't want to wait and want to plant big trees (with plants below them). DirkvdM 09:31, 27 August 2007 (UTC)[reply]
You'll want to wait for winter when plants are dormant for a bio-solution, but a fence of living willow would work. You can use a species that grows at a rate to match your need. Bendž|Ť 09:42, 27 August 2007 (UTC)[reply]
I strongly support the tree idea, as well, for acoustic, aesthetic, and environmental reasons. One caution is to use evergreen trees, not deciduous, as bare trees wouldn't provide much sound protection in winter. Another idea is to create an earthen berm, possibly with bushes and trees on top:
     /\
    /  \ TREES/BUSHES
     || 
   +----+
   |    |
 +-+    +-+
 |  BERM  |
-+        +-
StuRat 02:09, 28 August 2007 (UTC)[reply]
Or maybe one of those wooden fences with thin 'planks' woven through them. Horribly bourgeois, but you can have climber plants cover it. DirkvdM 06:28, 28 August 2007 (UTC)[reply]

Unidentified disease of Marigold

I have a marigold plant. It was flowering really well. Suddenlty one day I saw cobweb like stuff had developed on the flowers. On close inspection I saw small dot like white things moving in that web. This web is more like a thin veil of cotton rather than a web actually. I wasn't sure what to do so I banished the pot to the fire exit and closed the window. I did not want my other plants to get whatever it is and I thought sun might be helpful. I live in NY but am used to a more tropical weather garden. The questions are:

What is this disease? Is it harmful to other plants or to humans? How do I cure it?

Some added information: It rained heavily and the plant was soaked. I brought it indoors so that it is not harmed by the cold rain. It remained indoors for 4 days before it developed this thing. --Kaveri 13:59, 26 August 2007 (UTC)[reply]

Sounds like red spider mite. Easy to get rid of if you spray with water mist daily. (Or possibly mealy bug.)--Shantavira|feed me 15:25, 26 August 2007 (UTC)[reply]
I agree, red spider mite. They love hot, sunny, dry places so giving them a fine water spray once or twice a day will certainly control them. Check your other plants because these little monkeys can spread round a greenhouse at the speed of light. Richard Avery 10:13, 27 August 2007 (UTC)[reply]
Uh oh, looks like we're going to have to augment our disclaimer: no medical, legal, or horticultural advice! :-) --Steve Summit (talk) 17:35, 27 August 2007 (UTC)[reply]

lenses

Can't seem to find a page on 'lens manufacture' other than history of lensmaking. Is there one? I'm thinking specifically on how plastic and glass lenses are made (cast, then ground ??) rather than lens design.87.102.11.213 16:55, 26 August 2007 (UTC)[reply]

Float glass details the process of float glass, but I think this is not the common procedure for small lenses. We seem to have the historical perspective well-represented, but I can't find much on modern lensmaking either. Nimur 18:19, 26 August 2007 (UTC)[reply]

????Does "ref desk collaboration of the week" still exist - if so I'd like to put this forward. Where is it. I've also made a requested article thing as well..87.102.11.213 19:23, 26 August 2007 (UTC)[reply]

Lenses are ground into the desired shape. One problem with the grinding of large lenses is that gravity deforms them to a significant degree, so that changing their orientation or the strength of the gravity field (say by launching them into space) between production and usage can cause significant distortion. StuRat 01:59, 28 August 2007 (UTC)[reply]

Acoustics

When, say a trumpet plays a note, and then a piano play a note of the same pitch, what is the difference between the sound waves that the two instruments make that allows us to distinguish between them? Imaninjapiratetalk to me 17:31, 26 August 2007 (UTC)[reply]

The second,third,fourth etc harmonics differ. Overtones is probably the easier/better article to read here and both give you the answer.87.102.11.213 17:37, 26 August 2007 (UTC)[reply]
Also you are listening to different waveforms. See the various articles linked from that one, some of which have samples of what different waveforms sound like. The only thing the two notes have in common is their wavelength.--Shantavira|feed me 17:45, 26 August 2007 (UTC)[reply]
It's worth noting (as it's not in the article waveform) that the waveform consists of the (wavelength) fundamental frequency plus all the overtones or harmonics - bringing all three articles together.
Effectively the presence of harmonics or overtones is means that the 'note' consists of multiple pure tones - there are multiple notes being played simultaneously when a piano key is pressed - the note (on the musical score) corresponds to the first harmonic.87.102.11.213 18:07, 26 August 2007 (UTC)[reply]
Also, "imperfections" in each instrument (such as breathy noise in the trumpet) result in non-harmonic elements of the sound spectrum. You might want to investigate psychoacoustic perception, which is an ongoing research topic. Nimur 18:22, 26 August 2007 (UTC)[reply]
Then there is also the rate of attack and decay (how quickly the sound reaches max volume and how quickly it ends), which tends to vary by instrument. A grand piano, for example, will have a slower decay rate than an upright piano. StuRat 22:13, 26 August 2007 (UTC)[reply]
And the technical term for it is timbre. Confusing Manifestation 22:26, 26 August 2007 (UTC)[reply]
And Stu is referring to the adsr envelope (at least, that's the simplified model used in electronic musical instruments). Note that during attack, the pitch may also be different. DirkvdM 09:37, 27 August 2007 (UTC)[reply]
Also, the relative amplitude of the overtones can change with time. Also, some partials can be more or less out of tune (as per the 7th on a piano).
In truth, this question is being asked backwards! There is almost nothing the same between the acoustic waveform of a trumpet and a piano playing the same note. Just about all you can say is that the majority of the energy present in the sound is delivered at some specific frequency for most of the duration of both notes. All else is up for grabs and will be different. It's interesting that our brains are able to discern any similarity between the two notes at all! I suspect that some aspect of the way we percieve sound is responsible for that - but I'm no audio expert. My domain is in the world of light and eyes - and our ability to see hundreds of shades of red as all being fundamentally more or less the same "colour" is perhaps a similar ability. If so, it's an artifact of the approximate way we percieve such things ("It's a bug - not a feature!") SteveBaker 16:30, 27 August 2007 (UTC)[reply]
I'd say it's rather different for light. Calling different shades of red red is a linguistic thing - we can't have a name for every discernible shade. And perceiving a band of similar colours as one colour is a matter of lack of precision. But picking out the frequency of a note is more like a mathematical thing, probably some fourier analysis. Also note that we designed the instruments to make those notes. Non-living nature rarely produces them and then usually only a vague hint of a note. We put nature in a straightjacket to make it produce pure notes. So it's not so much that there was something there for which we developed a sense, but we had this sense for which we manipulated nature, so it would produce something for us to perceive. DirkvdM 06:39, 28 August 2007 (UTC)[reply]
The tiny hairs in the Cochlea are each tuned to a different frequency - the hairs that vibrate the most will be those tuned to middle-C - and those exact same hairs will vibrate for both the trumpet and the piano. The other hairs that respond to the overtones will also vibrate (differently in the case of piano and trumpet) - but those aren't "shouting the loudest" as the information goes to the brain. If we concentrate on the biggest signal from the cochlea - we'll hear the piano and the trumpet "playing the same note" - if we pay attention to the lesser signals, we'll hear the more subtle tonal differences between the two instruments. Our eyes only see at three different frequencies (unlike our ears which can hear all frequencies within our audible range) - but the result is the same. When the red sensor shouts the loudest - we can see that pink and red are varieties of the same thing. If we concentrate on 'overtones' from the green and blue sensors then we can tell that there are a variety of shades and brightnesses of 'red'.
I agree with you that orangy-red and red don't differ in the same way that a trumpet and a piano playing middle-C differ - an orangy-red would be analogous to an out-of-tune trumpet. But the difference between a bright red and a pinkish-red is very similar. The fourier series for pink has a peak at 'red' and 'overtones' of blue and green. But we still think of 'pink' as being 'a kind of red' - we don't imagine it to be 'a kind of green' even though there is plenty of green in there. This is not dissimilar to the idea that the overtones of a trumpet and in a piano being relatively unimportant to our hearing it as a middle-C.
SteveBaker 15:06, 29 August 2007 (UTC)[reply]

If a tree falls in the woods, does it make a sound?

answer the question!!!!--arab 20:56, 26 August 2007 (UTC) —Preceding unsigned comment added by TerrorSonghai (talkcontribs) 20:56, 26 August 2007

If a tree falls in a forestMatt Eason (Talk • Contribs) 21:05, 26 August 2007 (UTC)[reply]
And this entire discussion, from June 30. Without doubt, sound is produced. Nimur 21:54, 26 August 2007 (UTC)[reply]
Without doubt? That's not the answer. The answer depends on what kind of question you are really asking, as the normal one requires qualification to be precise. --24.147.86.187 22:00, 26 August 2007 (UTC)[reply]
Not really. Nobody was able to come up with any kind of reasonable version of the question in the June 30th discussion that produced a tree that falls in a forest without making a sound. The nearest anyone came (IMHO) required a redefinition of the word "sound" in a way that fits no dictionary definition. What do you think is unclear? SteveBaker 23:47, 26 August 2007 (UTC)[reply]
OK, I made many comments that I removed now. Science simply cannot answer any question, including this one, as the article on science correctly states. A.Z. 03:11, 27 August 2007 (UTC)[reply]
This thread is at grave risk of turning into the kind of debate which the reference desks frown on, but: this last is of course nonsense. Science can answer many (if not most) questions, and it can answer them to an arbitrary degree of accuracy. There are, to be sure, untestable philosophical questions which science cannot answer, and it's also true that "an arbitrary degree of accuracy" is not synonymous with "absolute accuracy". But if science can't answer anything, then how is it we're not all still living on nuts and berries? —Steve Summit (talk) 03:42, 27 August 2007 (UTC)[reply]
I'm sorry, but I do not know if we are living on nuts and berries. I do not know absolutely and unquestionably that a berry or a nut have ever existed. If you can't realize that science is nothing, and that it doesn't make you know nothing and that no truth comes from it, you are just fooling yourself. A.Z. 03:44, 27 August 2007 (UTC)[reply]
Yes, yes, it's fun to play the I-can't-prove-that-all-of-observable-reality-isn't-a-figment-of-my-imagination game. We've all played that game before. But if you want to cure disease or feed the masses or travel faster than you can walk, it's not a useful game to play. The "practical everyday purposes" which you seem to be pooh-poohing are (for practical everyday purposes) the only purposes that matter. Scientific truths may not be isomorphic with philosophical truths, but neither one has a monopoly on the concept of truth.
To A.Z.: This will be my last post in this thread. To everyone else: I apologize for prolonging a philosophical debate on this science desk. —Steve Summit (talk) 03:57, 27 August 2007 (UTC)[reply]
I profoundly disagree with you. I'm not playing a game, and I'm sorry you're mocking me. Believing in science is like believing in the Bible. You don't know if there are masses, you don't know if there is food, and that does matter. I feel really sorry to see a question that does matter being mocked here, and philosophers who are just people that don't have a blind faith in science or the Bible being called pathetic. You should admit that no, you don't know if a tree makes a sound or not when it falls, either in the forest or in front of you. I think we should feed the masses and live our practical ignorant everyday life, and I belive there should be science, but I won't pretend to know truths, and pretend I'm not ignorant. A.Z. 04:07, 27 August 2007 (UTC)[reply]
Here's the problem for the "nothing can be proven to be real" folks. If nothing were real outside of your thoughts then you could simply cease to interact with this phoney world. So shut off your sensory inputs and just sit inside your mind and all should be well. The trouble is that after you have thought a significant number of thoughts (what we scientists would describe as "a few hours of time passing"), your "fake" sensory devices are going to start making some alarmingly real demands on your thoughts. You'll find that you get sleepy, hungry and thirsty. There is simply no way to shut out those demands no matter how unreal you believe them to be. So whether or not these things are real, you have to act in pretty much every way as if they were. If the demands of food and sleep were illusions brought about by our senses - why are your thoughts so utterly dependent on them? If you go for 48 hours without sleep - you'll find your "inner thoughts" become very hard to keep on track...which would be a surprising thing if the real world were utterly illusionary. So the value of an unprovable (and unfalsifiable) hypothesis such as yours drops to zero within a surprisingly short period of time. The "real world" (as I would prefer to describe it) is 100% compelling - you can't shut it out no matter how hard you try. Time passes in your mind at a rate not too dissimilar to the real world - you can't count to 100 in your head (or think 100 interesting thoughts or...whatever) without at least (say) half a minute or so passing in the real world. Increasingly, we scientists out here in the real world can tell (in the form of various brain scanners) approximately what's going on in your 'inner thought world' - we can see regions of your brain relating to vision light up when you think about pictures and regions related to reasoning light up when you think about math problems. In an increasingly real sense, I can compare what's going on in your mind with what's happening in mine - and find some sharp correlations. So your personal 'inner world' is also increasingly able to be viewed from within my personal 'inner world' - and as far as can be discerned, they work the same way. So what is the point (other than to annoy Steve Summit - which, I agree may be entertaining!) in holding the bizarre view that the outside world is fake and your thoughts are all there is - when the reverse seems to be the only thing that actually works for you? You have absolutely no evidence for the view that your inner world is all that there is and that the real world is faked - yet there is a mountain of evidence that says the opposite. If all of that evidence was out here in the 'real' world - you'd have some grounds for regarding that evidence as illusionary - but it's not - some of the evidence for the existance of a real world exists inside your inner world. Your absolute inability to keep your inner world from turning to thoughts of food, drink, sleep, etc as the real world places demands on it reveals evidence that is totally independent of your senses. Sure the 'feeling' of hunger is a sensory input from your stomach - but if it were illusionary, you could have it not affect your inner world...but you just can't do that. Try this: Try counting to 100,000 in your head without getting hungry. If the world were illusionary, you'd be able to do it...you can't. You just can't. Ergo, the real world is...real. QED. SteveBaker 16:21, 27 August 2007 (UTC)[reply]
I fail to see how the fact that I apparently cannot choose not to feel hungry implies that the real world is real. A.Z. 23:55, 1 September 2007 (UTC)[reply]
[response to now-deleted prior comment referring to our article on Science]
While the statement, "Science does not and can not produce absolute and unquestionable truth" is certainly true in the context of the philosophy of science, it's profoundly misleading in the context of everyday life. If you've heard that scientific truths aren't absolute, you can easily conclude that (say) biblical truths are just as good as scientific ones. In a way, it'd be really great if we could publicly claim that scientific truths are absolute.
For everyday purposes, saying that scientific truth is not absolute is like saying that falling trees do not produce sound, or that the sky is not blue, or that 1+1=3, or that gasoline doesn't power automobiles, or that heavy objects fall faster than light ones, or that death from cholera is inevitable, or that easy transmutation of lead into gold might be possible tomorrow. —Steve Summit (talk) 03:31, 27 August 2007 (UTC)[reply]
But the tree question couldn't care less about everyday life. It's a question about truth, not a ridiculous question about whether your recorder will record something if you place it next to a tree that is about to fall. And, yes, I admit not to have a clue about whether the sky is blue. When I say it's blue, I'm assuming everyone knows that's just for practical everyday purposes. A.Z. 03:38, 27 August 2007 (UTC)[reply]
If you take the view that you cannot trust your senses then you have deeper problems. It's not just that you have to doubt that the tree makes a sound - you have to say that the words "tree", "fall", "forest" and "sound" are completely neutral audio signals that your senses are feeding to you with no verifiable meaning whatever. You don't know that you read the question correctly (or at all) or that you did in fact answer it. You don't know whether you are reading what I wrote - you don't even know that anything exists. If that is indeed your belief - then why are you trying to answer this question? In any not-believing-ones-own-senses situation, the question may never have been asked.
The truth is that the question (like everything else we do or say in the real world) is merely shorthand for: "If 'trees' (defined as the following bundle of possibly-illusory sensory inputs....) existed in the way our senses depict them - and if one 'fell' (defined as...) in a manner subsequently seemingly revealed to our senses but not reported directly to our senses at that time..."...and so on for about another four pages. The answer is: "If all of those things were hypothetically true then the answer would be (with many, many more caveats) - Yes!"...which is all you can ever answer about any question on any topic. The point being that you have to understand that the question is wrapped around with all manner of unwritten caveats and assumptions about the real world existing as perceived - and it is implied and understood that your answer should strictly be wrapped in the same unwieldy bundle of linguistic torture. If you accept the question under those terms then the answer is unquestionably "Yes" under those same terms. If you don't accept the question because you refuse to accept those layers of implied meaning then why are you answering it? If the sound of the tree might not exist - then neither might the question be about trees in forests at all - so you can't have a valid answer either way. SteveBaker 18:38, 27 August 2007 (UTC)[reply]
Actually in a number of ways you can't trust your senses. I'm not referring on the level of religious zealots who refuse to believe in science but simply that they have numerous limitations and a lot of things things are subjective in ways we often don't realise. I'm not saying this to be contrary and I suspect SB realises this, simply pointing it out for the general reader. Any philosophy of science book is likely useful to get an idea of the limitations of observation, one that I've read before and found good is What is this thing called science? by A.F. Chalmers. Nil Einne 22:34, 27 August 2007 (UTC)[reply]
That 1+1=2 is an absolute and unquestionable truth, not discovered by science. A.Z. 03:42, 27 August 2007 (UTC)[reply]
I don't know about that. It's (essentially) an axiom built into mathematics - and math (done that way) happens to be useful. But there are other ways that don't come out with that. Suppose you did all of your math in logarithmic domain or perhaps if math were invented by creatures who lived their lives moving at speeds close to the speed of light relative to each other and who learned to add velocities, times and masses using the Lorentz transform: 1c+1c doesn't equal 2c, it equals 1c. Sure they'd have to come up with some new, fancy representation for counting apples - but then we had to go to quite a lot of trouble to bend our math to do relativistic stuff that would come naturally to them. That doesn't make either their or our math 'wrong' - it just doesn't apply. Just as we say that the three angles of a triangle add to 180 degrees - that's not true in a severely warped gravitational field and beings who lived in proximity of a black hole would never have believed what Euclid had to say. It's not entirely obvious that every imaginable culture would see addition of integers as particularly useful. We can imagine beings who can only count to three (one, two, three, many) - who would have an entirely different arithmetic system. 1+1=2 and 1+2=3 but 2+2=many and 2+3=many. Also 2-1=1 but many-1=undefined. You would say that 1-1=0 was also an absolute, unquestionable truth - but the ancient Greeks had an entire system of arithmetic that didn't even contain the concept of zero. You could come up with an entire self-consistent (but not very useful in our world) set of mathematics based around denying our basic rules of arithmetic - in which 1+1=2 would not be quite the slam-dunk you think it is. SteveBaker 18:08, 27 August 2007 (UTC)[reply]
Besides, in most axiomatizations of basic arithmetic, 1 + 1 = 2 isn't an axiom (although something closely related, such as 2 = succ(1) in Peano arithmetic, may be taken as the definition of 2). For an example of how one might go about proving it while constructing arithmetic based on axiomatic set theory, see Principia Mathematica#Quotations and particularly this image there. (Ps. What all that old-fashioned set-theoretic notation in the image basically says is that, given two disjoint sets of one member each, their union has two members. Addition is later defined based on the union of sets.) —Ilmari Karonen (talk) 00:23, 29 August 2007 (UTC)[reply]
To the Original Poster: the answer to your question is, "yes". (Why do you ask?) —Steve Summit (talk) 02:56, 27 August 2007 (UTC)[reply]

The answer is yes. Those kids at the Humanities desk may tell you something different, but here we value objective reality and empirical verifiability. Unless you ask the quantum people, who will likely disagree as to whether the falling tree even hits to ground... Plasticup T/C 03:31, 27 August 2007 (UTC)[reply]

Sound of a bong bubbling* *Cough* Wow, like, you guys just totally blew my mind. 24.22.163.169 06:58, 27 August 2007 (UTC)[reply]


This requires the true definition of the word "sound" in a way that newer dictionary nolonger use. According to Webster's new international dictionary published 1931 2620 pg. Definition "Sound": 3. To Speak; utter; express "audibly". Now Rare. Definition of audibly which is an adv. form of "Audiblo" its definition is "Audiblo"- Capable of being heard. Not the word capable, as in if present or not present! key. it will be capable of being heard., it is capable of being heard even if no one there. Key word capable so it someone happens to be there it can be heard but does not require someone to be there, then it would be heard, not just capable.... Sound waves being produced,, case solved!!! Q.E.D. --Aaron hart 09:50, 27 August 2007 (UTC) so the ansuer is yes no matter how you twist your modern definitions......[reply]

Like in any debate, the person who defines the terms wins the argument. -- JSBillings 10:22, 27 August 2007 (UTC)[reply]

Answer=yes.87.102.45.106 13:19, 27 August 2007 (UTC)[reply]

Crack a tree, leave a tape recorder near it, and wait until it falls. Then listen to the tape to see if it made a sound.

How do you demolish a skyscraper?

What is the normal non-terrorism way to demolish a skyscraper? Mapper of the streets 22:54, 26 August 2007 (UTC)[reply]

Tall buildings are typically demolished using excavators. Controlled implosion has been used on buildings ~40 stories tall, although the danger and environmental hazards make this a non-prefered method of doing it (and this would imaginably be impossible in sufficiently dense urban areas). The tallest building ever demolished (not by terrorists) was only 47 stories tall. See also, demolition, high-reach excavator. Someguy1221 23:13, 26 August 2007 (UTC)[reply]
Explosives are planted in the base of the tower and then the tower falls vertically rather than on its side. It takes a lot of planning to get a tower to fall vertically. Juanita Hodges 23:25, 26 August 2007 (UTC)[reply]
Occasionally, buildings are dismantled piece by piece, in sort of the reverse order they were built. I watched this being done to the top floor or two of a building in Seattle that was being remodeled, and the same thing is currently and notably being done to the damaged Deutsche Bank building next to the WTC site. Presumably manual dismantling is more timeconsuming, more expensive, but less disruptive than either explosive demolition or excavator demolition. —Steve Summit (talk) 02:55, 27 August 2007 (UTC)[reply]
It is also noticeably less exciting. Plasticup T/C 03:33, 27 August 2007 (UTC)[reply]
Well, except for a couple of recent incidents at the Deutsche Bank building. :-( —Steve Summit (talk) 03:37, 27 August 2007 (UTC)[reply]
You can find a very good article on building_implosion at [1] -- WikiCheng | Talk 04:11, 27 August 2007 (UTC)[reply]
If you're (understandably) worried about the building falling to its side during demolition, think about the New York WTC (the terrorist thing you refer too, I presume). Both those towers fell straight down, despite the fact that they had been hit violently in the side. I can imagine that skyscrapers are even designed to fall like that if explosive demolition is so much cheaper than excavator demolition. Btw, funny that you specify non-terrorist. Loads of skycrapers must have been demolished (makes me wonder - how many?) and only once by terrorists. But the images have been repeated on tv so often that indeed I also instantly thought of the New York WTC when I read the header. We've been brainwashed. :) DirkvdM 07:13, 27 August 2007 (UTC)[reply]
A lot of people say there were explosives planted in the twin towers and people heard explosions at the base of the towers, too. Juanita Hodges 11:05, 27 August 2007 (UTC)[reply]
That would of course be the controlled demolition hypothesis. Worth noting is that – aside from a tiny minority of conspiracy theorists – it is widely acknowledged as bunk. Beware the 'major events must have major causes' trap in human psychology: [2]. TenOfAllTrades(talk) 13:27, 27 August 2007 (UTC)[reply]

You might enjoy David Macaulay's book Unbuilding, in which he descrbes the hypothetical dismanteling of the Empire State Building.

Atlant 12:00, 27 August 2007 (UTC)[reply]

The lesson the twin towers does bring is how relatively easy it is to bring buildings down more or less vertically. In that case, both essentially unplanned collapses happened fairly close to vertically. Demolition experts who are trying to achieve this kind of effect ought to be able to produce it on demand. One wonders how much thought about the ultimate demolition of the building goes into the original design process. Are architects inadvertantly making buildings that are going to be costly to demolish when the time comes? SteveBaker 15:49, 27 August 2007 (UTC)[reply]
Don't mean to brag, but I already said that. :) DirkvdM 19:09, 28 August 2007 (UTC)[reply]
Twin towers spilled debris out all over the place, presented a major health hazard, and rained concrete on the city. I doubt it is a good model for how things ought to work, demolition-wise. It superficially came straight down (under the massive weight of the top bits of it) but it was nowhere near a model for a safe demolition, and if you look at the footage/pictures you can see that there is a tremendous plume of materials as the buildings comes down (i.e. Image:South WTC Collapse.jpg). --24.147.86.187 18:22, 27 August 2007 (UTC)[reply]
Ok, so I definitely wasn't bragging. :)
A way to prevent this debris-damage is to make the building sag in the middle first, so the outer walls collapse inwards. Don't know how well this would work for skyscrapers, though.
But this brings me to a reply to TenOfAllTrades: A respectable Dutch tv programme, Zembla, debunked a lot of those 'conspiracy stories', but when they showed the film of the imploding adjacent CIA building (which was much smaller) to the major Dutch demolitioner he instantly said that that was without a doubt a controlled demolition job - it sagged in middle first, just the way demolitioners do it, and that couldn't possibly have happened in an uncontrolled collapse. Which fits in with the idea that the CIA knew about the attacks beforehand and decided this provided them with a nice excuse to 'counterattack' 'rogue states'. The collapse of their own building supposedly got rid of evidence. It was also oddly already evacuated before there was sufficient reason to do so. And after it became apparent that there was something wrong with those flights, normal procedure was not followed - they should have been instantly intercepted, but were left free to finish their 'job'. And there's more evidence pointing that way, I believe. DirkvdM 19:09, 28 August 2007 (UTC)[reply]
Btw, all references to this appear to have been removed from Wikipedia. They used to be somewhere, but I can't find them anymore. All there is is debunking of controlled demolition of the twin towers, but that's not the main issue, as I understand it. I wouldn't be surprised if the CIA started those stories themselves - create a lot of confusion with bull stories to distract from the real issue. DirkvdM 19:21, 28 August 2007 (UTC)[reply]


August 27

Noise reduction history

What 'playback only' audio noise reduction techniques were available in 1989 in the US? IE how did they go about reducing tape hiss in those days?--88.110.232.152 04:24, 27 August 2007 (UTC)[reply]

Dolby noise reduction systems were quite common in 1989, but they are not "playback only". I'm not sure there are any reasonable techniques for reducing tape hiss on playback only. Since tape hiss occurs across a very broad frequency range you can't really reduce it without also reducing everything, signal as well as noise (I could be wrong, perhaps there are some Magical Algorithms out there I haven't heard about). Still, it's best to take noise reduction steps at the recording stage. Pfly 05:59, 27 August 2007 (UTC)[reply]
On prerecorded tapes at that time dolby B was common.87.102.45.106 11:34, 27 August 2007 (UTC)[reply]
The playback-only techniques that are commonly used depend upon having a "moment of silence" from which one can deduce the various noise sources in the system (hiss, turntable rumble, etc.). One way it is done is to take the Fourier transform of the noise and decide how much noise is in each of the frequency bins. The normal (non-silent) signal is then run through the transform and any frequency bin that has about the same energy as was present in the noise sample is just dropped (zeroed out). The signal is then inverse-transformed back into an analog waveform and voila!, some of the noise has been removed.
Other techniques look for discrete events like "clicks" and "pops"; the real sound source is muted during these events and inferences are drawn (again, often using the Fourier Transform) as to what the sound would have been had the click or pop not occurred.
Back in 1989, it would have been a possible but expensive proposition. Nowadays, you can buy reasonably-priced software for your PC or Mac that will do all of this automagically (e.g., Sound Soap).
Atlant 12:07, 27 August 2007 (UTC)[reply]
thanks you. l am aware of the modern spectral subtraction techniques such as is used in Cool Edit etc. I have a reissued CD digitally remixed in 1989, where they say on the sleeve note that they have managed to remove most of the tape hiss (caused by too low a recording level) on the original master tape. What noise reduction technique are they likely to have used? (BTW is not low pass filtering as the upper frequencies are OK) Oh yeah, and why would they keep the very low amplitude and refrain from eqing the tracks back to something acceptable?--88.110.232.152 13:53, 27 August 2007 (UTC)[reply]

Plants Water Purifacation

How plants can discern the particles of substances and elements , with what special characteristic of them?Flakture 07:42, 27 August 2007 (UTC)[reply]

As a general answer, the process is based on similar concepts in both plants and animals. Membrane transport proteins are generally sensitive to the charge and size of molecules and ions. Proteins can also be "designed" to take advantage of chemical characteristics of target molecules (ie, the propensity of oxygen to bind to iron in hemoglobin). Someguy1221 08:08, 27 August 2007 (UTC)[reply]


Proteins can also be evolved to take advantage of chemical characteristics of target molecules (ie, the propensity of oxygen to bind to iron in hemoglobin).--Funnyguy555 21:49, 28 August 2007 (UTC)[reply]

Photon duality

--Aaron hart 11:30, 27 August 2007 (UTC) An Old subject but could someone please explain the current theory of partical duality. I thought I had it figured out, "Transformes from energy to matter for a brief instant durring an interaction. Durring the interaction I thought that when it existed as matter is speed was <<C then back to energy propegating through space time at C. Like the particles in a accelerator that result from a smash, these particles exist as mass and in an electromagnetic field spiral and pop into energy Likewise in space-time particles are continually popping from energy fluxuations to matter, then back to energy!., not nececerrially the pairs just similar....--Aaron hart 09:25, 27 August 2007 (UTC) ANY IDEAS From top Physicist giving up secrets that are well kept. Has it been proven that photons always travel at c, is it possible that durring an interaction velcity is <C since it interacts and transforms into mass, then imidiatly back??? Just wondering I mean is anyone really sure at this time??--Aaron hart 10:18, 27 August 2007 (UTC)[reply]

You have a very distorted idea of what matter and energy are. The interactions you think you are talking about, transformation from matter to energy, is actually not this at all. The interactions you speak of are transformations from waves to particles. Light never slows down below C when it interacts with something, and it never stops being energy (the "m" in E=mc^2 is not matter, but mass. A lot of people think that mass and energy are two very different things that can simply be interconverted, but their distinction is actually quite non-apparent in high-energy physics. A photon of "pure energy" has a measurable mass, that given by E=mc^2, and an atom holds within it the energy calculated from E=mc^2, using its measured mass. If you remove through chemical interaction some measurable amount of energy from an atom, it actually weighs that much less as well, even though you removed no particles from it). Now, what it means when light turns from a wave into a particle is that it goes from having no definite position to having a definite position (it's a very difficult concept to wrap your mind around that an object can actually not have a position. The wave the photon is said to be is a sort of map of the probability that you will find it at any particular point at a given instant in time. The very important idea to keep in mind is that before the photon interacts, it isn't actually anywhere! It has no position before it interacts, merely a probability of being found at each point in space). Now, as to what's going on in those particle accelerators is that a pair of particles, typically those generally associated with matter (ie, protons) are smashed together, and interactions that can be described only by quantum mechanics (these interactions are impossible to understand, no point trying to explain them) transform the proton into new particles. As for the particles popping in and out of existence in space time, there are two sources for these. Some are photons or or something else transforming into another type of particle, and then back. Others just, well, pop out of nowhere. These are called virtual particles. They don't actually require any energy to pop into existence, they just do. You can't detect them unless you supply them with energy somehow, but that's another story. But back to the energy/mass/matter thing. Matter is typically used to describe "that which is tangible" (atoms and their constituents). Energy, well, I have no way of explaining that one. You can read energy if you like. The most basic physical idea is that energy is a property of all particles that is conserved (the total amount of it in the universe never changes) in all possible detectable interactions, but is not a measurable physical property like mass. And mass, of course, is merely that which has inertia. I'm sure my answer will not satisfy you, please ask more, and do consider taking introductory quantum physics when you enter college (if you haven't already). Someguy1221 10:25, 27 August 2007 (UTC)[reply]
About light dropping below C, while the waves of light can do very funny things indeed, and appear to speed up or slow down, the most basic answer is that photons cannot ever change speed. This is not something to be proven or disproven, but rather it is a mathematically necessary property of photons derivable from the fundamental laws of physics. Someguy1221 10:26, 27 August 2007 (UTC)[reply]

Yes I worded it rather incorectly I did mean waves to particles which I meant to refer to as mass. Now my question is how can a photon have mass when only massless waves can travel at the speed of light? Yes I did mean that the waves convert to particles of a mass with the equivalent energy. I also knew that when chemicals interact exothermically they lose the mass equivalent to their evergy, Is this not a form of mass to energy, and could it not similary be waves to energy and energy to mass, and back. could some of the mass be changed to energy, in a wavelike packet? But the part I am interested in is the mass leaving the molecules when energy leaves, is this itself not a transformation as I have proposed?!, is not energy from a exothermic reaction transmitted as a photon? Also The particles poping into existance being photons does this not support my theory. Also I tought this was from energy fluctuations in space time......--Aaron hart 10:58, 27 August 2007 (UTC) p>s> I really appreciate someone with your calaber repling to me thoughts.[reply]

Also I was refering to energy to mass, or wave to particle the mass being equivalent to the energy of the particle. It has been a while since I have studied any of this, and quite a while since I have thought of any of it, it started to interfear with my being quote "a normal person" sorry--Aaron hart 11:30, 27 August 2007 (UTC)[reply]

I'll try to answer the new questions point by point. To say that only massless waves can travel at the speed of light is not entirely correct. The relativistic equation for kinetic energy (γ-1)m where "m" is the measurable inertial mass of the particle at rest shows that a particle with non-zero rest mass requires an infinite amount of energy to move at the speed of light, since γ = ∞ at the speed of light. This is not to say that photons have no mass, and in theory this is measurable by their gravitational influence (I can hop over to one of my labs right now and measure their momentum easily enough, but this may not satisfy you). Simply all of their mass resides in what is defined as their kinetic energy. Now, back to this mass to energy bit. Firstly, I'll declare that a particle is actually a special case of a wave. If you are concerning yourself with waves that represent a potential particle's probable position in space, then an actual particle can be thought of (ignoring a tad bit of physics at the moment) as nothing more than a Dirac delta wave. Further, one can think of energy and mass as abstract mathematical properties of waves, calculable from the use of mass/energy operators if the wave form is known. Now, an atom is a wave, or rather a superposition of many waves representing the numerous particles that can be found within. The waves have a calculable mass, and therefore energy. When what you normally think of as "pure energy" is extracted from the atom (this can be through emission of a photon or as causing something else to speed up) the wave forms will change in some way. The new wave forms, while describing the same potential particles, will have a different calculable mass/energy. Sometimes it's very easy to ascribe to some nature the type of energy one is dealing with, ie, kinetic energy, thermal energy, photonic/phononic energy, gravitational/electrical potential energy, etc. When you're dealing with a static system, like a lone atom, for example, it's very hard to ascribe the location of the energy, and we generally shrug it off and say that it's all just mass. But in truth, the "energy" is merely a calculable property of the wave functions describing that atom, and this point of view happens to work for all other forms of energy as well. This is all simply to say that classical views of the distinction between mass and energy somewhat collapse in the presence of quantum mechanics. Now, to the last question, the nature of any possible "energy fluctuations" in space-time is largely a matter of interpretation of the meaning of vacuum energy. But if you prescribe yourself to believing that particles pop into existence, then by quantum mechanics, as hard as it may be to swallow this, they didn't actually come from anything. They just start existing, and very quickly stop existing. Now unfortunately I must go to bed, but I'll be back in about 8 hours. Someguy1221 11:29, 27 August 2007 (UTC)[reply]

measurable inertial mass of the particle at rest this is where I believe it is a particle, sice it acourding to quantum mechanics it can never be at rest. Also I don't completly believe in Quantum Mechanics, Is it not based on we can not know the position and the velocity since we have to interact with it, which messes with the velocity. But this is only due to scientific measurement, not actuall fact. In fact Relitivity states the Big bang began at a Singularity, Impossible for Quantum Mechanics, but only by our measurments, thus I believe it is basied upon an unkonwn, which is unkown to observers but not God. Thus I believe in singularities, and sure it stands up to evidence, since we are observing the phenomenon, thus it fits. From our observation.--Aaron hart 11:52, 27 August 2007 (UTC) P.S. thank you[reply]

It is actually a very good question you bring up, and a common and key misunderstanding of quantum mechanics. We are often taught in high-school or introductory college-level chemistry classes that Heisenberg's uncertainty principle prohibits us from knowing the simultaneous position and momentum of a particle at a given moment in time to greater than a specified accuracy. What we normally aren't told is that it is not our inability to measure these quantities, or our inability to measure one quantity without disturbing the other (that one is only partially true. After measuring a particle's position, applying the momentum operator to the new wave function of the particle, now something like a Dirac delta, will show a very large range of probable values). Rather, the uncertainty in a particle's position/momentum arises from the fact that it doesn't actually have a single defined position and momentum. This is a very alien concept to classical physics, and why it is a common misunderstanding. As I said in above posts, the wave function, through the application of operators, gives the probability of the particle having any given position, momentum, or energy. Before the measurement is made (any arbitrary event whose outcome depends on the particle's position/momentum/energy), the particle exists only as this wave, and occupies no actual position in space. And so, from a quantum perspective, it is not that there is a limit to what we can know about the universe, but rather there is a limit to how much information can actually exist in the universe. As to the singularity thing, Quantum doesn't forbid the existence of a singularity, it merely can't explain it. But neither can relativity. Someguy1221 19:22, 27 August 2007 (UTC)[reply]
I misunderstood, I thought that a particles position and momentum could not both be measured at the same time, and that this was a conciquece of the observation. Also I don't understand how something with mass can travel at the speed or light, You site rest mass, but this is the total energy of the particle, which can not exist at rest. I just don't seem to get the concept; a particle with rest mass traveling at the speed of light, has all of its matter or more correctly mass converted to energy (i.e. electromagnetic wave), then when it interacts I believe it is possible however short lived, to transfer from energy back to mass, to interact. Then the mass transfers back to a EM wave, due to its unstable form, (being so small). Which I think all has to do with the properties of Space-time. I am most likely way off, but it makes sence to me, and it is very hard to relate in words. Note, I did not think Physicist completely understand Photon Duality at this time; only theorys with mathmatical formulas that all fit nice and neat (for the observer)--Aaron hart 21:48, 27 August 2007 (UTC)[reply]
You're right on the very last point, quantum mechanics was only ever meant to make predictions of the outcomes of experiments, nothing more. Although this has been the nature of science since Newton, moving away from attempts at providing an antiquated sense of satisfaction in explaining natural phenomena. Just a couple more points I'll repeat and/or throw in: a particle that has non-zero rest mass cannot travel at the speed of light, as this would mathematically imply infinite kinetic energy. As for photons interacting, I think all that I can say is that one can find a time-dependent wave function that describes the interaction. I've even said before, and I'll say here, if one is actually trying to solve a QM problem, it's best to ignore why any of it works, and just accept it (darned good predictions come out of it). Someguy1221 21:48, 27 August 2007 (UTC)[reply]
Thank you for your time and patience, I learned much from you, but still wonder about the natural phenomenon; and what actually takes place, but I am over 30 so I guess I should give it up. Again thank you for your time--Aaron hart 04:32, 28 August 2007 (UTC)[reply]
(Being 30 is no excuse - there are plenty of 60 year old physicists!) The problem is that you are trying to think about what "actually takes place" - implying that there is some 'classical' event going on behind the math that you simply don't understand. It's not like that at all. The reality is that all of this weird quantum stuff is what "actually takes place". Fundamental particles are indeed simultaneously particles and waves - they really don't have a well defined position or momentum - masses and time compression really does happen when objects move fast. That is literally how the universe operates. The only reason we find that strange is because we've evolved and educated ourselves in an environment where we only see and interact with slow moving macroscopic objects. This idea of 'classical' Newtonian physics is simply not how things really are - it's merely a shorthand for producing approximate answers for questions that relate to our slow moving macroscopic lifestyle. SteveBaker 11:58, 28 August 2007 (UTC)[reply]
Yes, I do believe something actually takes place, but I don't know if anyone understands it beyond mathmatical derivations from partial differentals of the wave equation. I do not believe it is a classical event, though I believe photons have simultanious mass/momentum, it is just impossible to observe, or measure. I believe that Heizenberg basied his uncertanty principle on the interaction durring observation. While Debrolie and Schrodinger used mathmatics alone, my question is did they do this to fit the observations; "this may be an ignorant question, but I am curious." And to be honest I prefer modern physics to classical newtonian, since it is more accurate and really makes more sence, I can use the equations just fine, I just wish to know what actually takes place!. I realize this may be of no concern for finding the outcome of situations, but it would be beautiful to know how particles actually interact in space-time.--Aaron hart 14:17, 28 August 2007 (UTC)[reply]
How things 'actually' happen is something that's unknowable though - two fuzzy clouds of probability approach each other - and some short (but fuzzy) amount of time later a number of differently fuzzy probability clouds come out the other side. We can't look more deeply into it than that because whatever we prod and poke the system with to examine it changes the answer...which we couldn't measure precisely in the first place. We're beyond the point where we could make progress by observing what happens and writing it down. Nowadays we have to theorize what the larger scale effect of some quantum scale event might be if it works like this rather than like that - then think up some kind of devious experiment to show that happening. Annoyingly, these experiments require greater and greater energies - resulting in exponentially more expensive equipment. The great hope of using string theory for answering these questions suffers from this exact problem. It is beautiful, elegant and has great explanatory powers - but it's unable to predict anything that we can test - so whilst it may well be right, we can't tell whether it's true or not. In the end (if we buy into string theory), the universe appears to be the result of the mathematics of probability wave packets and the more carefully we look, the less there is to see. SteveBaker 19:15, 28 August 2007 (UTC)[reply]

Riverstone pebble tiles

Hi, under composite materials I have found the above product, which I require for my knitchen worktop. Problem is I cannot find any reference to a maker, where can I source this? MMany thanks <email address redacted>

Email address removed — Matt Eason (Talk &#149; Contribs) 11:01, 27 August 2007 (UTC)[reply]

Do you mean like this http://www.alibaba.com/catalog/10880060/River_Stone_Natural_Stone_Stone_Tile_Cobble_Pebble.html or http://www.ecvv.com/offerdetail/I1424081.html ?87.102.45.106 13:14, 27 August 2007 (UTC)[reply]

Thanks, close but I need a uk supplier any thoughts?

Try searching for "riverstone pebble tiles" and restrict your search to the uk - google allows you to do this. eg http://ww.google.co.uk/search?hl=en&q=riverstone+pebble+tiles&btnG=Search&meta=cr%3DcountryUK%7CcountryGB —Preceding unsigned comment added by 83.100.251.36 (talk) 06:31, August 28, 2007 (UTC)

True open circuit- calculation

How does one achieve a true open circuit (at RF) at the end of a piece of 50 ohm coaxial cable (or other sort of line) bearing in mind that the impedance of free space is always 377 ohms? Is it possible, or must one always have some radiation from the line end?--88.110.232.152 13:41, 27 August 2007 (UTC)[reply]

ie since :, is it true that gamma can only be a maximum of (377 -50)/(377+50) = 0.7658? as opposed to the ideal of 1?

A very good question, and I'm disturbed that I can't get my thought in order to answer it myself. My intuition tells me that if you enlarge the cable (keeping its impedance constant, of course) so that the centre conductor and shield are far apart, the situation would be more close to the ideal open circuit. Then again, you know how intuitions can be. Please, can someone help out here? —Bromskloss 19:23, 27 August 2007 (UTC)[reply]
I suspect that the 377 ohms is irrelevant in this case, but I can't be certain. At a guess, it's because the sequence of impedances is 50R-open-377R and not 50R-377R, so the wave is reflected from the "open" condition at the end of the cable and never sees the 377 ohms of free space beyond.
I can more easily tackle the OP's conclusion, because a simple ETDR experiment shows that an open-circuited cable has a reflection coefficient close to 1. See Figure 11 in LeCroy App Note 016 for an example. --Heron 19:32, 27 August 2007 (UTC)[reply]
Ah! but how close? cant be closer than 0.7658--88.111.124.125 20:44, 27 August 2007 (UTC)[reply]
So you didn't look at the example I linked to, then. The example I linked to above shows a gamma of at least 0.95, as nearly as I can judge from the waveform. --Heron 19:12, 28 August 2007 (UTC)[reply]
Yes I did look at your link. It gives the same response as it does on my oscilloscope. My Q was how the hell can it be better than the 0.7856 predicted by theory. Some other responses are starting to answer this question--88.109.90.214 22:45, 28 August 2007 (UTC)[reply]
OK to help anyone pursuing this line of inquiry, I quote from the article on coaxial cable:
  • The characteristic impedance in ohms (Ω) is calculated from the ratio of the inner (d) and outer (D) diameters and the dielectric constant (). The characteristic impedance is given by .
Certainly a lower source impedance would make a better o/c, but still not perfect. Is perfection possible/--88.110.52.40 19:35, 27 August 2007 (UTC)[reply]
On the topic of electromagnetic radiation from an open end of a coax it can happen, but the coupling to free space is very low, and unless the coax diameter is comparable to the wavelength the escaping radio waves will be minuscule. The reason is that the radial arrangements of fields cancel out at a distance. I will see if I can find a formula for this. Graeme Bartlett 22:39, 27 August 2007 (UTC)[reply]
So you are saying less than 10% of the incident power leaks out the end of the cable-- is that correct?--88.111.187.180 22:50, 27 August 2007 (UTC)[reply]
Much less that 10%. 0.01% would be more typical. Graeme Bartlett 06:18, 28 August 2007 (UTC)[reply]
The leakage is proportional to the area of the coax between the conductors, and to the square of the frequency. Graeme Bartlett 22:48, 28 August 2007 (UTC)[reply]
That's an excellent question! You made me think, and almost had me convinced. I don't think it's true, though. The reflection coefficient is really close to 1. Why? The formula you gave is specifically for a resistive load that terminates the cable. If you hook up a resistor to the end, that's what you get. But the 377 Ohms of impedance of free space is not a resistive load! Of course, you can't measure 377 Ohms of resistance across an open circuit. Therefore, the simple formula is not applicable. --Reuben 06:44, 28 August 2007 (UTC)[reply]
Indeed. I filled a whole notebook with uglier versions of the reflection coefficient equations (practical, empirical, and theoretical equations). There are boatloads of "special case" treatments which you can find in a good textbook on circuit or antenna theory. Nimur 16:53, 29 August 2007 (UTC)[reply]

toothpaste

does toothpaste ever expire? thanks.

The Chicago librarians who researched this question say yes. [3] --Sean 14:27, 27 August 2007 (UTC)[reply]
Based on my readings, toothpaste has no expiration. There is a harmless chemical that keep it from expiringWikiPoTechizen 09:57, 3 September 2007 (UTC)[reply]

reaction machenism

conversion of suger to 2,5-dimethlyfuran with complete reaction machanism

  • Check your spelling; I believe you mean 2,5-Dimethylfuran. The reference materials on that page should answer your question. --M@rēino 17:28, 27 August 2007 (UTC)[reply]
I looks like you might be thinking of a method with Hydroxymethylfurfural as an intermediate- that page has a lot of useful mechanistic infomation (on the production from sugar).87.102.85.15 17:32, 27 August 2007 (UTC)[reply]

Telescope Physics

I know how a telescope works, however, I am a little shady on the physics of light travel and a telescope. It's fairly simple when you talk about seeing a couple of miles away with binoculars as you are seeing what is happening right then. What I don't get is when astronomers look into space billions of light years away. I understand that light travels at approx 186,000 miles per second and so what is being seen happened billions of years ago. So the question is when you look through a telescope are you simply pin pointing an area of the sky as though it were a 2-dimensional image? So the more closely you can pin point an image, the farther away from earth you can see? If I were able to take an extremely high resolution picture of the sky would I be able to see as far away with the telecope? --Beckerj99 18:25, 27 August 2007 (UTC)[reply]

Your picture needs to be both high resolution and high sensitivity, as the images of distant objects will be very faint - but the basic answer to your question is "yes" - see Hubble Deep Field, Hubble Deep Field South and Hubble Ultra Deep Field. Gandalf61 18:40, 27 August 2007 (UTC)[reply]

Awesome, I always thought that was the case, but couldn't find an article specifying it. Thx!!

Earths magnetic pole swapping

Does anyone know thy from time to time, the earths magnetic poles swap from N to S.--Aaron hart 21:30, 27 August 2007 (UTC)[reply]

Geomagnetic reversal#Causes. Someguy1221 21:50, 27 August 2007 (UTC)[reply]
Thank you, from now on I will search before I ask, It was very interesting. It has been bothering me for years. Two thumbs up for Wikipedia..--Aaron hart 14:33, 28 August 2007 (UTC)[reply]

Time dilation at the speed of light

Does time dilation become infinite at the speed of light? If so, then does time pass for light? 69.150.209.13 22:18, 27 August 2007 (UTC)[reply]

time dilation becomes infinite for light. No time passes from the point of view of light. So it is emitted and absorbed at the same time, (in its own reference frame), even when it crosses astronomical distances between galaxies. Graeme Bartlett 22:45, 27 August 2007 (UTC)[reply]

composite material, kitchen work top made of clear resin and white pebbles

Were can I source this product?

Try your local hardware or DIY store perhaps. Or stores used by builders if they serve you. Given than you're provided zero information about where you live (I'm guessing the US but who knows) we can't help more. Nil Einne 22:40, 27 August 2007 (UTC)[reply]
Again a search engine is your friend "pebble resin tile" into any generic search engine will produce results eg http://www.google.co.uk/search?hl=en&q=pebble+resin+tile&btnG=Search&meta=cr%3DcountryUK%7CcountryGB (same poster for UK as above?) gives multiple choices - is that ok?83.100.251.36 06:42, 28 August 2007 (UTC)[reply]


August 28

When is the best time to change gear in a manual car?

In the question Wikipedia:Reference_desk/Archives/Miscellaneous/2007_August_18#transmissin_output_torque_.3D_engine_power, User:SteveBaker talked about obtaining the best 0-60 time in his Mini Cooper S by shifting to keep the engine at it highest torque output. I notice SteveBaker answers alot of questions on the reference desk so I have been reluctant to ask, but I can't work it out for myself -- Why is gearing ignored in his answer?

For example, I created some tables using these gear ratios I found on the net for a Mini Cooper S:

1st Gear 11.450
2nd Gear 7.181
3rd Gear 5.397
4th Gear 4.407
5th Gear 3.656
6th Gear 2.986

According to the graph he linked, in first gear the engine at 5500 RPM would be creating 150 ft-lbs of torque and turning the wheels at around 480.35 RPM (5500 RPM / 11.45) with 1717.5 ft-lbs of torque (150 ft-lbs * 11.45). If we shifted up now, the engine would be running at 3449.39 RPM (480.35 RPM * 7.181) and producing still around 150 ft-lbs of torque but our torque at the wheels has dropped to 1077.15 (150 ft-lbs * 7.181). Had we have stayed in first gear to nearer the redline, say 6500 RPM we would have been making only 140 ft-lbs at the engine but 1603 ft-lbs (140 ft-lbs * 11.45) at the wheels.

Help! Where am I going wrong?

Alan 78.32.138.240 00:12, 28 August 2007 (UTC)[reply]

I think what you say sounds good. As far as I can tell, you are correct. —Bromskloss 06:31, 28 August 2007 (UTC)[reply]
What you are missing is that if you stay in 1st gear for longer than I would, your torque in 1st is dropping rapidly as the RPM's increase - so whilst you'd be getting more torque in 2nd after you shift at redline, you'll have lost ground over someone who shifted earlier. SteveBaker 11:47, 28 August 2007 (UTC)[reply]
By my basic understanding, acceleration is determined by force at the wheels, force at the wheels is given by engine torque * gear ratio. Even if torque dropped to 130 ft-lbs at the flywheel, 130 * 11.450 is still more then 150 ft-lbs * 7.181. Maybe my confusion is coming from the use of the word torque for both "flywheel torque" and "driving wheels torque". Also see [4] which is linked from the article on Torque, specifically the section "A Simple Example" where he shows that by changing from 3rd to 2nd even though he produces less flywheel torque, it results in more force at the wheels. Alan. 78.32.138.240 12:29, 28 August 2007 (UTC)[reply]
Really? Decreasing torque as a result of staying in gear is OK, if shifting would have given you even less torque. (I assume shifting is instantaneous, as I believe we all do.) —Bromskloss 12:01, 28 August 2007 (UTC)[reply]
mathematically speaking: plot rear wheel torque vs car speed in all gears. you will get a bunch of peaks, one for each gear; but what's important is where the descending tail end of each gear crosses the ascending front end of the next gear. obviously, that's the shift point, in mph. in real life, you have to allow for a drop in rpms while shifting, so run it a bit higher. Gzuckier 15:57, 28 August 2007 (UTC)[reply]
That is exactly what I did and from 0-60 I can see no point where the Mini Cooper S generates more torque at the wheels in a higher gear (lower ratio) before hitting the redline. I am admitedly very much a beginner when it comes to automotive engineering but my theory is that, as the Cooper S is supercharged it has a very flat torque curve through-out the RPM range much like a naturally aspirated engine with a higher displacement - coupled with it's wide gear ratio means the curves do not overlap. Although if SteveBaker has had his Mini Cooper S on a drag strip I cannot argue with empirical evidence. Alan. 78.32.138.240 16:50, 28 August 2007 (UTC)[reply]
For 'drag strip' read 'nice, level, deserted freeway on-ramp'! I used an accellerometer to measure 0-60 times. Please bear in mind that mine is not a 'normal' MINI Cooper'S - its got a bunch of 'go faster' stuff, so my torque curve doesn't look like the one in that article. Also, in practice, an awful lot depends on how fast/smoothely you shift and how quickly the engine will drop RPMs as you shift - and (not least) whether you can keep a close enough eye on the tachometer while remaining on the road! It took months of practice and a computer program that I wrote to simulate the entire process in order to get the best 0-60 time out of my car - and without an accellerometer, you can't measure your times accurately enough to know whether you are getting better. SteveBaker 19:02, 28 August 2007 (UTC)[reply]
For what it's worth, I've used an accelerometer (A Perfmeter Pro I think it was called), but I'm not sure I have a lot of confidence in their accuracy. The one I used depended on being perfectly level when the car is sitting there. But, a hard launch will jerk it around, possible causing it to move off-level, which could skew results for that run. I suppose as long as you confirm that it's still level once you've stopped, maybe this problem is reduced. But, to my knowledge, car magazines for example don't depend on such devices, presumably because they're willing to put in the cost and effort for more accurate means of measurement. But, maybe a typical accelerometer is plenty accurate for casual usage as long as it's used correctly. Mine was pretty easy to use incorrectly, tho. Friday (talk) 19:08, 28 August 2007 (UTC)[reply]
Mine's a G-Tech Pro - it compensates for being level because you tell it when you are stationary and it measures it's slope at that point. It seems to survive the worst jerks I can give it with my MINI - it beeps when it thinks you're at 60mph - and that agrees well with my GPS's speed estimate (but not with the car speedometer - which like most cars is deliberately off by a couple of mph). But in any case, I'm not looking for absolute numbers - just "did I do better than yesterday", and since I used this same freeway on-ramp (on my way to work after dropping my kid off at school - every day for many months) - all of the slope issues and other variables ought to cancel out. The things that don't cancel out are things like ambient air temperature (which affects the car's performance noticably) and the amount of gas in the gas tank (which is a not-inconsiderable weight difference in such a light car as the MINI). SteveBaker 21:24, 28 August 2007 (UTC)[reply]

katydids/diet

Your entry on katydids reports that larger, predatorial katydids have been known to eat small children. This is a joke, yes? My son, Andrew, is worried. Thanks for your clarification. —Preceding unsigned comment added by 72.133.249.135 (talk) 03:31, August 28, 2007 (UTC)

That was vandalism. Interestingly enough, added by an IP address registered in Wisconsin, same as yours! Someguy1221 03:51, 28 August 2007 (UTC)[reply]


It is ashame that someone would alegedly lible a reputable person, did they think it was funny, or a joke, "yes"., I don't find it funny.--Aaron hart 14:29, 28 August 2007 (UTC)[reply]
Apparently, not a lot of people think about the consequences of their actions, unfortunately. x42bn6 Talk Mess 15:44, 28 August 2007 (UTC)[reply]

Laser

So, we've all heard of those handheld green laser pointers that can burn through paper, but would shining a "normal" laser pointer through a telescope of magnificvating glass make it as powerfull (ya know, like when you do it with the sun?) 82.198.250.72 14:55, 28 August 2007 (UTC) 82.198.250.72 14:55, 28 August 2007 (UTC)[reply]

Probably not : a lens focuses the rays from the sun onto a point - but in the case of a laser there is really only one ray. That single ray does have width however so it will be possible to concentrate the narrow cylinder of light onto a point - in fact people have done this - creating flashes at the focus point (in air) where the atoms or whatever are heated very hot and ionised.87.102.90.8 15:16, 28 August 2007 (UTC)[reply]
I've never heard of this being done with a normal laser pointer (1mW) but it might be possible if focused enough. If you want to do this, you'll have to widen a lot your beam (because otherwise you'll be limited by diffraction) and then refocus it. Then, you'll have problems with the quality of your optics. You can't make your calculations with ray optics, you at least have to consider gaussian beams optics or a full simulation taking into account diffraction. It's difficult to focus much narrower than 1 micron^2. Might be easier with a green pointer of equivalent continuous power because those ones are pulsed (passive Q-switch) so it interacts a bit differently with matter. --81.67.15.32 18:09, 28 August 2007 (UTC)[reply]
In my optics lab we used lenses to focus lasers very similar to those laser pointers (the only difference was that ours were more coherent) into fiber-optic systems. The typical width of a laser from such a pointer is 1 mm, but fiber optic wires we used were about 10 micrometers wide. The Gaussian nature alone prevented us from making it much smaller than a few micrometers wide, though. Someguy1221 03:00, 29 August 2007 (UTC)[reply]

XRF Bremsstrahlung Shape

I've done my homework on the wikipedia X-Ray Fluorescence Spectroscopy and Bremsstrahlung articles, as well as some others, but I can't seem to find any sort of equation for the Bremsstrahlung itself. I don't care how many parameters it calls upon -- I just need to find some sort of legitimate baseline that works better than a basic interpolation fit. Any ideas? 146.139.76.94 14:06, 29 August 2007 (UTC)[reply]

http://rkb.home.cern.ch/rkb/PH14pp/node16.html (any good? - didn't check!)87.102.90.8 16:16, 28 August 2007 (UTC)[reply]
Have you tried searching the web itself for "Bremsstrahlung" already?87.102.90.8 16:17, 28 August 2007 (UTC)[reply]

I appreciate the link - although I can't seem to get a fittable equation out of the one in integral format. I've tried googling a range of related search topics, but none of the results actually show how the background Bremsstrahlung was fitted. 146.139.76.94 14:06, 29 August 2007 (UTC)[reply]

I'm afraid I don't understand/can't explain bremsstrahlung fully, and nobody else here seems to be clued up on it..
In the link I gave - it seemed to cover bremsstrahlung for an electron in an atomic field - this is the behaviour you are asking about I hope..
Assuming the link was in the right subject area but not detailed enough maybe the maths desk could help you more?
You said "I just need to find some sort of legitimate baseline that works better than a basic interpolation fit" - did that mean you want an expression for the whole spectra, or do you just need a relationship between highest bremsstrahlung energy and electron X-ray energy (latter is much easier)
I'll try to help further if you wish. though I'm currently confused how conservation of spin (selection rules) can be made to fit with the spectra obtained..If you can explain that to me I might be able to have a go at deriving the equation myself for you. ("Hope Springs Eternal")87.102.18.14 14:39, 29 August 2007 (UTC)[reply]

I'm guessing that you have a real spectra and need a theoretical bremsstrahlung curve so that you can subtract it to get a clean baseline...?87.102.18.14 14:43, 29 August 2007 (UTC) (Note - if you are doing x-ray fluoresence won't you be measuring the electron energy and not bremstrallung photon energy? - though the two are closely related)[reply]

By the way it's possible to quite easily remove the bremsstrahlung hump without knowing its equation using simple analysis (on a computer) - and just leaving the peaks on a flat baseline - maybe this is all you need?87.102.18.14 15:51, 29 August 2007 (UTC)[reply]

Area of Earth one can see.

Just say I were in the middle of a flat ocean, and I look 360 degrees around myself, how much of the surface area of the earth will I be able to see as a percentage of the total area of earth, and as just a figure? Assuming perfect vision. Phgao 15:44, 28 August 2007 (UTC)[reply]

The horizon article is a good place to start. If you know the distance you can see, you can figure out the area of the sphere within that distance. The Mathematics Desk might be a better place to discuss the details of these geometrical calculations. DMacks 16:00, 28 August 2007 (UTC)[reply]
(ec)According to Horizon#Distance_to_the_horizon, the true horizon is only 4.7 km away for a person 1.7 m tall. (You may be able to see farther due to refraction dependent upon atmospheric conditions.) Thus you can see an area of 69.4 km². The surface area of the earth is 510,065,600 km². Percentage-wise that's tiny: 1.37 x 10-5 %. -- Flyguy649 talk contribs 16:02, 28 August 2007 (UTC)[reply]
For a perfectly spherical earth, I think the answer is exactly , where R is the radius of the earth and h is the height of your eye above the surface. This agrees with Flyguy649's number for the special case of h = 1.7 m. -- BenRG 16:25, 28 August 2007 (UTC)[reply]
BenRG messed up somehow - that can't be the right answer - if you increase R, the distance you can see gets smaller - that's not right. When I plug in the numbers, I wind up with about 1 micron as the distance to the horizon!! SteveBaker 18:51, 28 August 2007 (UTC)[reply]
The area you see increases, but the fraction of the total area decreases. BenRG's answer is dimensionless, so it could never have been the area itself. —Bromskloss 19:10, 28 August 2007 (UTC)[reply]
OH! I see, sorry I didn't realise BenRG was talking about the ratio of the area. That makes sense. My bad! SteveBaker 21:07, 28 August 2007 (UTC)[reply]
Now wait here. I'm getting something else. Did you neglect the curvature of the Earth at some point? —Bromskloss 17:59, 28 August 2007 (UTC)[reply]
Hmmm - let's derive this from first principles: Imagine a right triangle - one vertex at the center of the earth (C), another where your eye is (E), the third where your line of sight hits the horizon (H) where it is a tangent to the surface of the earth. The angle C-H-E is a right angle (because E-H is a tangent). The distance you can see is given by Pythagoras: (R+h)2=d2+R2 (where 'd' is the distance you can see, 'R' is the radius of the earth and 'h' is the height of your eye above the surface of the earth). Rearranging, I get: d = sqrt((R+h)2-R2) which you can multiply out to give: d=sqrt(h2+2Rh) - Some people drop the h2 part because it's negligable compared to 2Rh, so you wind up with d=sqrt(2Rh) - which is a reasonable approximation for small values of h. SteveBaker 18:43, 28 August 2007 (UTC)[reply]
(And plugging in the numbers - I get 4.656km for a 1.7m eye height - which is in good agreement with Flyguy649's answer). SteveBaker —Preceding unsigned comment added by SteveBaker (talkcontribs) 18:48, August 28, 2007 (UTC)
Sorry, I was wrong. I did it over again, and got exactly the same as BenRG, without any approximations. The area seen is , and if you divide by the area of the whole Earth () you get . —Bromskloss 19:06, 28 August 2007 (UTC)[reply]
So you could see someone else of that height at almost 10 km away in the eye - assuming perfect vision. DirkvdM 06:13, 29 August 2007 (UTC)[reply]

How much of a difference does atmospheric refraction make? Could you see more of the earth if you were in a big desert (a hot one) than if you were at the antarctic (due to temperature gradients)? How much more? Capuchin 06:55, 29 August 2007 (UTC)[reply]

Yes - refraction through the air does extend the distance you can theoretically see by a small amount. Exactly at the horizon there is a deflection of about a half degree (at sunset, for example, when the bottom edge of the sun touches the horizon, in reality the sun is almost completely below the horizon and you can only see it because of diffraction). Translating this into extra distance is kinda tricky - but the difference isn't great. You can see a teeny-bit further at the poles than at the equator because the earth isn't quite spherical...but again, the effect is negligable. It's rather pointless to figure all of these subtle effects into the equation because you never have sufficiently perfect conditions for these things to matter - at sea, you are bobbing up and down - so your estimation of eye height is not accurate. On land, the ground is never flat enough for these calculations to have real meaning. The numbers we've already given are plenty good enough! SteveBaker 14:41, 29 August 2007 (UTC)[reply]
Pointless but interesting :) Capuchin 14:59, 29 August 2007 (UTC)[reply]
Hardly pointless! International treaties have been made and broken over such details as "how far into the ocean can I see from my watchtower?". How far from the coast is international water? Were these guys in sight of land or not? Modern GPS surely doesn't eliminate the problem, or else such incidents would easily be settled by consulting a hand-held display and pleasantly agreeing on coordinates. Nimur 17:02, 29 August 2007 (UTC)[reply]

Gull feathers - do they make better duvet filling than duck/goose/chicken feathers?

I was told this today by someone who used to make feather duvets. I don't know if she was joking or not but apparently, seagull feathers make the warmest duvet fillings of all - better than eiderdown. The only downside is that they're hard to gather and expensive (thinking about it, gulls have very thick coats, don't they?). Anyone have more info? I know some of the gull people here will be interested in this. --90.240.209.98 15:45, 28 August 2007 (UTC)[reply]

Generally down is better than feathers (in terms of warmness) I believe..You must have meant seagul down?87.102.90.8 16:24, 28 August 2007 (UTC)[reply]
Question - can it really be any better than any other 100% down filling?87.102.90.8 16:29, 28 August 2007 (UTC)[reply]
Good question. I wonder if the down of the gulls of the high Arctic (e.g. the Glaucous Gull) has superior insulating properties? Gulls are generally *very* hardy birds, so maybe there's something to it. Heh, I wonder if a penguin down duvet would be any good? --Kurt Shaped Box 16:37, 28 August 2007 (UTC)[reply]
It's possible, but you would need to research which particular species of gull, and you would need an awful lot of them. I've no doubt penguin down is also an excellent insulator when supplemented with a layer of fat.--Shantavira|feed me 17:42, 28 August 2007 (UTC)[reply]
The person tasked with collecting the gull down has my deepest sympathy. They should get those bear bile guys to do it. --Kurt Shaped Box 21:50, 28 August 2007 (UTC)[reply]

experiment to determine planck's constant

hi

can someone suggest an experiment to determine planck's constant (apart from the common photoelectric effect one)? thanks. —Preceding unsigned comment added by 202.63.233.12 (talk) 15:54, August 28, 2007 (UTC)

Seaching the web for "planck constant determination" throws up several different methods eg http://ww.google.co.uk/search?q=planck+constant+determination&hl=en87.102.90.8 16:22, 28 August 2007 (UTC)[reply]

ANTI-GRAVITY WORKSPACE

This question is really aimed I guess for an Astronaut, being that they are a well documented breed of workhorse, I'm going to assume there is a wiki-individual that knows something to the matter. ?Being in an Anti-Gravity environ, the lack of stress on the body, what types of effects does this have on the mindframe of a worker?(can they work longer than 8 hours? is time an issue? on time off, do you still work just to do something? how much sleep is needed? who evaluates fatigue?) In the same sense of T.H.White's explanations on how things came to be from observations in the wild kingdom. I'm going to theorize the pressures of the battle regalia of Samurai/Knight class warriors. Pressures stressed on body, flare up a driving force to focus intently on whatever it is you are doing. Like all the mail a knight would have to wear, and his visibility from helmet visor, and/or Samurai scary mask, covers much of the face and vision., and yet the extreme warrior gets the job done. Or the today's and Victorian English Gentleman and Gentlewomen, with the extreme stresses on body with tight corsets and neckties. Putting pressure on a human body, the physical part, does in fact amplify the mental performance. And yet, the exact reverse, no pressure at all., hence my question on anti-gravity. This may fall under the catagory of Corporate Mass Psychology, there's a term for that, I just forgot what it was. Thank you. 216.100.216.5 16:57, 28 August 2007 (UTC)Jacob V[reply]

I've tried to parse your stream-of-consciousness "question", and have a couple comments. 1.) Astronauts work in zero (or near-zero) gravity, not Anti-gravity. 2.) Anti-gravity doesn't actually exist outside of theory and science fiction. 3.) The psychophysical problems with living in the Weightlessness article. -- JSBillings 17:23, 28 August 2007 (UTC)[reply]
Strictly, it's not even 'zero g' - there is almost as much gravity up there as there is down here. The correct term is 'free fall'. Astronauts and the craft they are in is falling - hence they don't FEEL the gravity. But it's definitely still there! The only time humans have truly experienced zero g was during the moon shots when the astronauts were at the exact point between earth and moon where the two gravity wells cancelled out...and even then, they were experiencing the force of gravity from the sun. SteveBaker 18:26, 28 August 2007 (UTC)[reply]
I would suggest also that the notion of encumbrance increasing effectiveness is flawed, at least as presented. A suit of armor doesn't make someone a better swordsman, it just makes him harder to kill. Additionally, much of the encumbrance you mention (such as corsets and neckties) contributes no meaningful weight, which is the only change when moving to weightlessness. A suit of armor is still incredibly restrictive when weightless -- just look at spacesuits! — Lomn 18:18, 28 August 2007 (UTC)[reply]
Plus, a lack of gravity does not negate mass's inertia. So, if you're wearing a half-ton suit of armor in space, you still need to be strong enough to get it moving and to slow it down, move your arms and legs. -- JSBillings 18:24, 28 August 2007 (UTC)[reply]
Our bodies evolved to work efficiently at 1g. I would expect there to be in almost all cases some deterioration to functions in any other environment without artificial support. Capuchin 07:40, 29 August 2007 (UTC)[reply]
As far as ability to work goes, microgravity is far better than no gravity. Microgravity makes heavy things easier to lift (although inertia does become an issue with really heavy items), but doesn't cause the annoyances of zero gravity, like not being able to walk and having your tools float away when you set them "down". While inertia can be overcome by moving massive objects extremely slowly, at some point it becomes too slow to be practical. StuRat 04:48, 30 August 2007 (UTC)[reply]
I remember a scene in fiction where a man annoyed by a cat picked it up and placed it high in the middle of the room, where, thanks to microgravity, it would take twenty minutes to hit the floor. The cat, having been subjected to this treatment before, resigned itself and took a nap. Anyone know what story that was? —Tamfang 07:29, 2 September 2007 (UTC)[reply]
Sounds like a good story (that would have to be like 1 millionth of a g). StuRat 02:15, 3 September 2007 (UTC)[reply]

White House

Why is it called white house? Is it because they only allow white presidents? 211.28.78.74 18:02, 28 August 2007 (UTC)[reply]

Read the article White House before making ridiculous assumptions. -- Kainaw(what?) 18:05, 28 August 2007 (UTC)[reply]
that article is incomplete. just says http://en.wikipedia.org/wiki/White_House#Naming_conventions which says nothing about the origins of name?
The article states (under Naming Conventions): "...it is also speculated that the name of the traditional home of the President of the United States may have derived from Martha Custis Washington's home, White House Plantation in New Kent County, Virginia...". It is my opinion that that is SOMETHING about the origins of the name. -- Kainaw(what?) 18:14, 28 August 2007 (UTC)[reply]
Yes i did read that part, but it is speculation, which shouldnot be in there. You should remove it. I want to know the truth, what is the real reason. Why would you call it "white" house? Has it got to do with American Foreign Policy? 211.28.78.74 18:26, 28 August 2007 (UTC)[reply]
The phrase in the article is properly cited and there is no need to remove it. You are correct that speculation on the part of Wikipedia editors shouldn't be in articles, but this is not an example of such speculation. There is no conspiracy or secret agenda behind the naming of the White House. — Lomn 19:38, 28 August 2007 (UTC)[reply]
Given that when it was named, only whites were allowed to vote anyways, they would see no need to point this out. It is like referring to it as the "Human Palace" on the off chance great apes get the vote some day. Eran of Arcadia 19:53, 28 August 2007 (UTC)[reply]
Just a minor point of politeness -- I know you probably didn't intend it, but the analogy comparing Whites::Nonwhites and Humans::Apes is somewhat offensive. -- JSBillings 20:07, 28 August 2007 (UTC)[reply]
It seems fairly clear that Eran was refering to how people perceived things at the time, not how things actually are. Skittle 14:05, 29 August 2007 (UTC)[reply]
(de-indent) Not true, strictly speaking. While race was abolished as a condition for suffrage in 1870, many states permitted multiracial voting from inception. — Lomn 20:05, 28 August 2007 (UTC)[reply]
But don't forget the poll tax though Nil Einne 20:20, 28 August 2007 (UTC)[reply]
There were plenty of voting restrictions to be sure. However, "only whites were allowed to vote anyway" is flat-out false. — Lomn 20:47, 28 August 2007 (UTC)[reply]
After reading the article, it seems that they called it the "White House" just as another name amongst others, such as the "President's Castle", even before it was burned to the ground and rebuilt. Could it possibly be that it was called the "White House" because.... it was painted white? -- JSBillings 20:13, 28 August 2007 (UTC)[reply]
That question is as absurd as asking "Why is the Casa Rosada called like that? Is it because they only allow pink presidents?" Titoxd(?!? - cool stuff) 20:15, 28 August 2007 (UTC)[reply]
While I'm not challenging the idea the question is absurd, to be fair no one ever really called themselves pink AFAIK (ever if many people are more accurately called pink then anything else) whereas people did and do call themselves white. So it's not really the same thing IMHO (but reread the first part of my response). Nil Einne 20:23, 28 August 2007 (UTC)[reply]
Yeah, but the original question was gasping at straws to make a political statement out of an insignificant fact... Titoxd(?!? - cool stuff) 22:55, 28 August 2007 (UTC)[reply]
This question doesn't really have anything to do with science does it Nil Einne 20:24, 28 August 2007 (UTC)[reply]
The "urban legend" of its name [5] went that after the British burned it in 1814, it was stained black from the soot, and after it was rebuilt, it had to be painted white, so it came to be called the "white house." In truth, the stone it was built from was porous and tended to absorb moisture and spall during freeze-thaw cycles, so the exterior was painted white from the day it was built. Furthermore, after the celebrated burning, it was demolished down to the basement level except for parts of the south face, and rebuilt with new stone. It was first officially called the "White House" on presidential stationary during the Theodore Roosevelt presidency. Edison 02:00, 29 August 2007 (UTC)[reply]
It's a house. It's white. DirkvdM 06:14, 29 August 2007 (UTC)[reply]
One might go so far as to make the political interpretation that it is a house and not a palace because America is a Republic with a President, not an aristocracy with a monarch. Our presidents are private citizen and they live in a house. Nimur 17:20, 29 August 2007 (U
Wasn't it originally called the Presidential Palace? —Tamfang 07:26, 2 September 2007 (UTC)[reply]

I think I agree to what Edison explained on how the "White House" name came about. By the way, Presidents are public personalities because they are public servants.

Yes, and many, including the current President, have thoroughly serviced both the US electorate and the world at large. StuRat 04:37, 30 August 2007 (UTC)[reply]
I don't think any more political statements are needed after the troll's initial question, but... please leave it to the "world at large" to decide whether they feel "thoroughly serviced"... --Ibn Battuta 23:24, 30 August 2007 (UTC)[reply]
Many have been. Whether they liked it is a different matter. DirkvdM 06:25, 31 August 2007 (UTC)[reply]

Does ice conduct electricity?

I ask because I work at a facility which has just purchased an AED. When using an AED, the victim must not be in a puddle, and must have as dry a chest as is possible. Our policy also includes moving victims off ice (we have an ice rink), which seems ridiculous if ice doesn't conduct electricity. Mike.lifeguard | talk 19:50, 28 August 2007 (UTC)[reply]

But the thin film of water that forms when you place your feet over the ice, due to conduction and radiation, does conduct electric charges. Titoxd(?!? - cool stuff) 20:17, 28 August 2007 (UTC)[reply]
Neither pure ice nor pure water conduct electricity very well - it's the stuff in the water that changes that:
  • Ultra pure water: 5 x 10-6 S/m
  • Drinking water: 0.005 – 0.05 S/m
  • Sea water: 5 S/m
  • Ice: 6x10-6 S/m
But ice is weird stuff and it conducts across surface defects rather than through the bulk of the stuff. Weird. SteveBaker 21:02, 28 August 2007 (UTC)[reply]

Unidentified Fixed Object

This

Can anyone tell me what this is?

I saw this thing under a light microscope at low magnification (something like 100X if I remember correctly). Some background if it helps: My friend and I were preparing a karyotype for fun. I collected some of his blood, went through the whole procedure, and I came across this thing on one of the slides. —LestatdeLioncourt 21:24, 28 August 2007 (UTC)[reply]

It looks vaguely like the cellulose skeleton of a plant cell wall but that is a guess. Graeme Bartlett 22:50, 28 August 2007 (UTC)[reply]
Well, it seems awfully hard to believe - but it looks suspiciously like a Stent of some kind. Quite how you'd manage to extract one from someone's blood stream is kinda hard to imagine. If that's correct then whoever you yanked this out of REALLY needs to seek medical help ASAP! SteveBaker 22:51, 28 August 2007 (UTC)[reply]
I agree that it looks man-made (like a stent). I would blame contamination from lab materials before assuming that something came out of your friend's bloodstream. -- Kainaw(what?) 23:01, 28 August 2007 (UTC)[reply]
Can you estimate the size and describe the shape more? How does it appear to you when you fiddle with the focus? just describe anything that might not be obvious from the picture. -- Diletante 00:52, 29 August 2007 (UTC)[reply]

I'm not sure about the size, but if it helps those little dots here and there are T lymphocytes. By the way, I'm leaning towards the contamination theory; I don't really think this was acutally there in his blood. I'd just like to know what it could be. Aren't stents too big? I don't know if anyone noticed, but I find its membrane's pattern very interesting (you can see it most clearly on the right side). —LestatdeLioncourt 06:32, 29 August 2007 (UTC)~[reply]

It's really not clear from the description exactly how big this thing is. If that's a 100x magnified image and those dots are lymphocytes then this thing would have to be a fraction of a millimeter across - so I guess my stent suggestion is not it. It sure looks artificial though - but maybe there is some kind of natural structure that looks like this. Weird. SteveBaker 14:25, 29 August 2007 (UTC)[reply]
It looks vaguely diatomish to me. DuncanHill 19:25, 29 August 2007 (UTC)[reply]
I'd like y'alls to really work on this one more. You've got me wondering. If a Diatom of some sort, how does it live in the bloodsystem? t-cells, on attack? Could it be some sort of metalic crystalisation of sodiom of sorts? Or again the plant theory?-some bacteria of sorts? Come on people! bear down. Ask a scientist! --Jacob Vi am the kwisatz haderach 21:16, 29 August 2007 (UTC) —Preceding unsigned comment added by Specialagent777 (talkcontribs)
If this was "for fun," then what was your lab procedure? If you are working with a set of slides or cover slips that had previoiusly been used for diatoms and not scrupuluosly cleaned, or on a bench that was contaminated, or in a labe with some dust fo the right sort, this object might have gotten into you sample. My swimming pool filter uses "diatomaceous earth," which is essentially finely ground chalk and which (I guess) may have such structures in it. -Arch dude 00:10, 30 August 2007 (UTC)[reply]

Well the first part of the procedure (culturing) was carried out in an aseptic environment (it had better be, cuz it was in a blood-drawing room), but later on (centrifugation, hypotonic treatment, fixation, etc...) the chances of outside contamination was very high, because we were working in the same compartment that they were doing all the bacterial cultures in. The lab technician who was there said he hadn't seen anything like it in his 15 years of lab work, so I doubt it was due to contamination from the compartment we were working in. The slides were also brand new and previously chilled in a sterile cup. —LestatdeLioncourt 22:18, 30 August 2007 (UTC)[reply]

If you were 'shooting up' 'bad drugs' the 'product' might be 'tamped' with some 'white stuff' eg chalk - but then you'd know if you were doing that wouldn't you.. Seriously though (I HOPE) The probability of external diatom contamination is quite high - if for instance you - "use a lot of chalk"..87.102.14.233 08:27, 30 August 2007 (UTC)[reply]

Another suggestion is that it is the wing off an insect, perhaps a mosquito or fly. It is broken off on the lower end and complete on the upper rounded side. Initially I thought it must have been a cut out piece of seagull feather, but the line around the edge argues against this. Diatom occurred to me, but its nothing like the diatoms I have seen, that tend to have spikes on them. Graeme Bartlett 10:51, 30 August 2007 (UTC)[reply]

It doesn't look at all like a fly wing. (I've looked at very, very many!) At high mag, they have a regular pattern of "hairs" that all point in the same direction. And epithelial cells in general are hexagonal in shape (to maximize cell-cell contacts). Flyguy649 talk contribs 13:58, 30 August 2007 (UTC)[reply]

Another thing is skin (reminds me of snake skin) - what exactly is the length of this thing? —Preceding unsigned comment added by 87.102.14.233 (talk) 11:11, August 30, 2007 (UTC) Also too big, plus (above) diatoms don't have seems (or do they?)87.102.14.233 11:14, 30 August 2007 (UTC)[reply]

The idea that it is a wing or scale appears to me to be based on the idea that the object is flat. The bad focus on the lower left and upper right indicate to me that this is more of a tube than a flat object. It isn't really even a round tube. It has rather sharp corners, making it rectangular. The roundness of the top isn't very round if you follow the edges up to it. It is nearly squared off as well - just appearing rounded due to the lost focus. -- Kainaw(what?) 13:22, 30 August 2007 (UTC)[reply]
Could it be a thread or a bit of gauze? Some type of cotton fiber?--69.118.235.97 15:43, 30 August 2007 (UTC)[reply]
If you look very close it seems that it is woven, and that threads approach the edge from behind wrap around and continue to be woven. Could this possble be some new kind of blown celluloce cigarete filter???--Aaron hart 03:50, 31 August 2007 (UTC)[reply]
You know you need to quit...when you have cigarettes floating in your blood. Someguy1221 03:53, 31 August 2007 (UTC)[reply]

August 29

Chill

Hi, what is actually (ie biologically) happening when one feels a chill running through one's body, or back or whatever? Thanks —Preceding unsigned comment added by 218.250.156.40 (talk) 01:55, August 29, 2007 (UTC)

First, what causes it... This is caused by the sympathetic nervous system, which triggers the fight-or-flight response. It is normally caused by the perception of something requiring the body to prepare for a fight-or-flight response, but can also be triggered by a minor chemical imbalance. The imbalance is normally corrected immediately, but the sudden rush of endorphins and/or adrenalin can cause chills, sweats, and goose bumps.
Now, the chill... As explained, the body is in fight-or-flight mode. One of the responses is to constrict blood flow through much of the body (preparing to need a burst of oxygen-rich blood in a moment's notice). That constriction flows across the skin and is felt as a "chill". This is normally accompanied by the other responses, but can be a chill with no other responses when caused by an imbalance. -- Kainaw(what?) 02:13, 29 August 2007 (UTC)[reply]

Thanks a lot! —Preceding unsigned comment added by 218.250.157.77 (talk) 01:41, August 30, 2007 (UTC)

Hollow moon

Is it true that the moon is actually hollow? --124.254.77.148 02:33, 29 August 2007 (UTC)[reply]

See Hollow moon. There are various advocates of this theory - eg. [6]. But I doubt that most terrestrial scientists take it seriously. -- JackofOz 02:39, 29 August 2007 (UTC)[reply]
Yes, Mighty Mouse ate all that delicious green cheese. Clarityfiend 03:50, 29 August 2007 (UTC)[reply]
If the moon were hollow, then:
  1. The moon would long ago have collapsed into the hole in the middle - and we'd be looking at a solid (albeit smaller) moon.
  2. There is no known mechanism for a large hollow body to form. What we know of planetary formation would not permit that.
  3. We have measured the vibrations of the moon from the impact of discarded a lunar lander crashing into it. The resulting seismic signals show a solid moon.
  4. The moment of inertia of a hollow sphere is wildly different from a solid sphere. Careful measurement of the moon's moment of inertia clearly prove that the moon is quite solid.
  5. The mass of the moon is easily measured from it's gravity. If it were hollow, the material which the skin of the moon would have to be made of would have to be insanely dense.
This is a really obvious nonsense theory. Why do people have to keep coming up with these things? Are the mysteries that we REALLY have not interesting enough for them?
SteveBaker 15:24, 29 August 2007 (UTC)[reply]
Perhaps it's because the real mysteries take the form of "Why doesn't the zero-point energy of the vacuum cause a large cosmological constant?". You can see how "the moon is hollow!" could hog the limelight. --Sean 19:35, 29 August 2007 (UTC)[reply]

Moon's moment of inertia

So, how has the Moon's moment of inertia been determined? It's not as if we could just apply some known force and measure what happens. --Anonymous, August 29, 2007, 20:08 (UTC).

Well...that's not exactly true.
I only know that the rotational inertia of the moon has been measured because hollow moon says so. But if I was an astronomer given the task of doing this I'd note that the moon is almost exactly tidally locked to the earth (ie the same face of the moon faces the earth all the time). That means that theoretically, the length of a lunar day and a lunar month ought to be identical. Yet we know that the duration of the moon's orbit is slowly increasing. As the orbital speed changes then the moon's rotation has to change to compensate for the orbital speed change. We know the size if the force that's doing that (the gravitational forces of earth and moon) - so we could measure the rate at which the orbital speed and rotational speed are changing (which ought to be fairly easy to measure since Apollo dropped laser reflectors onto the moon). So a known force (the tidal force) is being applied and we can measure what happens. But that's speculation. I don't know how they actually did that - I'm relying on Wikipedia (Dangerous!!) SteveBaker 21:38, 29 August 2007 (UTC)[reply]
Here is how they proposed to do it in 1977. SteveBaker 21:44, 29 August 2007 (UTC)[reply]
Well... it is exactly true. The tidal force isn't something "we" apply.
Regarding the first answer, you're talking about measuring the extent by which the change in rate of rotation speed lags behind very small changes in the orbit. I find it hard to believe that enough precision would be available th atway. As to the second answer, only the abstract of that article is available online free. It apparently refers to measurements based on effects of the orbit on a lunar orbiter. But that ought to be impossible; because the gravitational fields of a hollow sphere and a solid one of the same mass are identical, it would seem that such an experiment could only detect departures from spherical symmetry, not the distribution of mass by depth. I must admit that to having no idea what is meant by "C 22 gravity harmonics", though. --Anonymous, August 29, 22:18 (UTC).
The Moon is is tidally locked, but it still oscillates with a motion called libration. This oscillation would depend, I think, on the Moon's moment of inertia. -Arch dude 23:52, 29 August 2007 (UTC)[reply]
(Update) Ah, yes. A quick google for moon libration inertia gives a lot of papers that show that libration is used to measure inertis, and that are measuring things that are several orders of magnitude more subtle than a hollw moon would be. -Arch dude 00:00, 30 August 2007 (UTC)[reply]
Aha. I was thinking of libration only in terms of the apparent motion due to the difference between the essentially constant speed of revolution and the varying speed of the moon in its orbit, but that also causes a change to the angle of the tidal force, and that causes a real motion, and there we go. Thanks! --Anon, August 30, 02:15 (UTC).
I also wondered how the moment of inertia is measured by fine observations of satellites in lunar orbits. Surely a hollow shell would be indistinguishable from a solid moon as far as Lunar Prospector is concerned! Without having done the homework, I think it must work something like this: the moon's librations depend on its mass distribution in several ways. The amount of torque depends on how asymmetric the moon is, and the angular acceleration is the torque divided by the moment of inertia. You can see the librations from Earth and make very fine measurements without leaving home. But if you don't know how much torque is causing the librations, you can't calculate the moment of inertia. So you need the satellites to tell you how lopsided the moon is, and therefore how much torque is acting on it. That probably dominates the uncertainty on moment of inertia. (Just to repeat, this is an educated guess - my university doesn't seem to have an online subscription for reading those papers either!) --Reuben 02:40, 30 August 2007 (UTC)[reply]
The moment of inertia of a hollow sphere is quite different from a solid one - even if their masses are identical. For a uniform solid sphere it's 2mr2/5 and for an infinitely thin spherical shell it would be 2mr2/3 - for something which is hollow but not infinitely thin, it's somewhere between those two limits. So there is a huge difference in the two - even with fairly poor precision one would be able to tell the difference between the two. As noted in NASA's 'moonfacts' the measured moment of inertia is about 1% less than theory would suggest for a uniformly solid sphere. (A hollow sphere would have more inertia than a uniformly solid sphere. That means that not only is it solid - but it's also denser in the center than it is at the surface. SteveBaker 04:00, 30 August 2007 (UTC)[reply]
Are you responding to my post? I don't disagree, but I don't see how that relates to the question of how satellite tracking is sensitive to moment of inertia. --Reuben 06:51, 30 August 2007 (UTC)[reply]

Vision improved?

Something really weird happend yesterday. I went to the eye doctor and he told me my vision improved and its rare but he's seen it before in people that have improved their health (I went from 205lbs (30% bodyfat) to 165lbs (about 11% body fat) but I have never heard of this, is it really possible? I didnt believe it either at first but he gave me new contacts and I see very well with them. —Preceding unsigned comment added by 76.167.145.55 (talk) 04:03, August 29, 2007 (UTC)

Note that nothing said here may be taken as medical advice. Sudden worsening of vision has sometimes been found due to a high blood sugar from adult onset Diabetes [7] , [8],[9] , and with improved blood sugar levels, the vision has sometimes improved, if the vision changes were due to the effect of high sugar causing swelling of the clear tissues in the front of the eye (as opposed to the more permanent effects on the retina). Weight loss due to diet and exercise can lead to improved blood sugar levels. In any event, congratulations on having vision improvement rather than vision detriment. Edison 17:31, 29 August 2007 (UTC)[reply]
(to Anon) My vision has improved slightly in one eye over the past number of years enough that my prescription is off in an old pair of glasses. So you aren't unique here! Flyguy649 talk contribs 21:44, 29 August 2007 (UTC)[reply]
Near-sightedness (myopia) is common in the young, and far-sightedness (presbyopia) is common in older people. Thus, it's fairly common for those who are near-sighted in their youth to see their vision improve slightly with age. This happened to my brother, he was slightly near-sighted, then his vision was perfect for maybe a month, now he is slightly far-sighted.
Another possibility is that it's just an inability to measure your vision accurately. Since they rely on the "which one is better" method, and I usually can't really tell between two adjacent settings, I might pick the worse setting of the two one time and the better setting of the two the next time, even though there was no real change. StuRat 04:18, 30 August 2007 (UTC)[reply]
In that situation, I tell the optician that I can't tell the difference. In fact, they usually phrase it as 'Which is better? 1, or 2? Or are they the same?' at my place. After all, no point in giving a potentially wrong answer to that sort of question. Skittle 14:31, 30 August 2007 (UTC)[reply]
My optometrist measures my prescription with a machine (that you look in with a picture of hot-air balloon in it) and then confirms it with the "which is better" - which is also for astigmatism correction. Flyguy649 talk contribs 13:27, 30 August 2007 (UTC)[reply]

Germans then and now

This is more of a historical-biological question. Are the physical characteristics of today's German people any different than that of their 11th century ancestors? I know some Germans that have dark hair, brown eyes, and very tan scan. I realize that not all Germans are tall with light skin and blue eyes, but I'm sure some sort of change had to take place over the course of a millennia. I guess it also depends on what area of Germany you choose and what the staple diet of the people was at that time. If they were underfed for example, they would be shorter because of a lack of protein and vital nutrients required for healthy growth. However, this is just a generalized question. --Ghostexorcist 07:25, 29 August 2007 (UTC)[reply]

((Probably an archeologist would be a good person to ask (or an anthropologist) - ie someone who studies skeletons of 15th century germans - maybe such people hang out on the humantities desk - I suggest you ask there as well..87.102.18.14 11:50, 29 August 2007 (UTC)))[reply]
I think geography is important more because of interbreeding with neighboring people. Those in northern Germany would tend to resemble Swedes, Danes, Finns, and Norwegians (blonde hair and blue eyes), while those towards the south would tend to resemble the French and Italians (dark hair and eyes). StuRat 04:08, 30 August 2007 (UTC)[reply]
I don't know much, but it's a fact that they were shorter. I'm not sure these are facts, but at least rumor has it that Frederic the Great, when recruiting his famous soldiers (the "Potsdam Giants" - in German simply "lange Kerls" = tall guys) only from people that were 1.80m (5"11') or taller, had some trouble finding enough (capable) people. Well, the population was also smaller, but still I don't think you'd have any trouble finding men of +1.80m in Germany nowadays. (No guarantee that this story is true, it's just hearsay; but the different size of people in the Middle Ages is an often talked-about fact; also see the linked article.)
What you're mentioning about dark hair, brown eyes, and tan scan, however, seems more of a genetics question. So apart from emigration of specific types and from immigration (not in extreme numbers since the 11th century... if you disregard the 20th century which has greatly changed the German population), the regional differences that you mentioned are important. Northern Germans come generally closer to the stereotype (tall, blond, blue-eyed - think of stereotypes about the Dutch or Scandinavians) than do South Germans, and I would speculate that this was even a bit more true before mobility increased dramatically in more recent decades and through WWII - though one can hardly stress enough that even in the North, the stereotype is far from being true in general. (For current numbers, this article might tell more, but it's not for free... But google tells us that "In Germany, among 6000000 school children only 31.80 percent had both blond hair and blue eyes..., while of the 75377 Jewish children 42 percent were of the ..... As the percentage of blond types among the Germans in Prussia is very high ..." - so, judging by the context, these may be figures from the 1930s). Happy googling... --Ibn Battuta 23:57, 30 August 2007 (UTC)[reply]
Comment. In my experience the germans aren't that tall..83.100.249.228 15:58, 1 September 2007 (UTC)[reply]

Biros!

Here's a stumper for you all. At work I enjoy using many different colours of biro. I have found that while the blue and black ink biros write well, the red ones don't write quite as well, and the green ones very rarely work. Why is this? Is it a different viscosity of the ink? Out of 4 green biros from 2 different batches, I have only just got 1 to write at all, and it doesnt write very well. Capuchin 09:08, 29 August 2007 (UTC)[reply]

Could it be that green is not a popular coulour, and that the biros sit around for ages on the shelf before you use it? Perhaps exceeding the shelf life. The same could apply for red. I noticed that no one steals a red pen, but the blue and black pens disappear fast. Blue and black are far more popular and would have a faster stock turnover rate. Graeme Bartlett 09:13, 29 August 2007 (UTC)[reply]
I'll snoop around and check tomorrow morning! :) Capuchin 09:56, 29 August 2007 (UTC)[reply]
I vote for Graeme Bartlett's explanation. The other point I'll note is that water-based pens (such as Biros) often write poorly on xerographic copies or printouts, owing to an overall fine coating of hydrophobic plastic toner materal on the printout. But I've never known that malfunction to be color-sensitive.
Atlant 11:39, 29 August 2007 (UTC)[reply]

This problem varies from make to make surely?87.102.18.14 11:41, 29 August 2007 (UTC)[reply]

Yes there are some good quality pens around that work. Also some workplaces have more problematic pens than others. The kind of workplace that buys 10000 biros at a time can have problem with aging biros. And the fine point pens seem to blockup more easily than the coarse tipped pens. Graeme Bartlett 14:49, 29 August 2007 (UTC)[reply]
I assume you refer to Ballpoint pen rather than the felt-tip variety. Except for the most expensive name-brand ones, they always seem to stop writing long before the ink is used up. I have tried the time-tested method of scribbling them back and forth on scrap paper while muttering (usually ineffective), using centrifugal force by whirling/flipping them like an old mercury fever thermometer (often effective) and applying heat to the tip (rarely effective but often satisfying). The centrufugal force method might be more effective if the pen were attached to a motor to allow more effective centrifuging of the ink, forcing fresh ink around the ball to flush out the dried ink, but of course there is the likelihood of the attempt producing a line of sprayed out ink on anything within range (been there, done that), and/or a pen/projectile flying loose and causing injuries or damage. This has been discussed at Answerbag.com: [10]. [11] suggests storing biros vertically with the cap on, and running the tip across an eraser to force the stuck ball to roll and to clean the dried ink off the ball. Edison 17:18, 29 August 2007 (UTC)[reply]
erm had you tried licking the tip? - re wetting the dry ink? (I'm finding it difficult to resist advertising my favoured biro brand - which of course work down to the last mm of ink)87.102.18.14 18:17, 29 August 2007 (UTC)[reply]
(US readers may not appreciate that 'Biro' is the name of a company that makes cheap ballpoint pens. They have had such a longstanding dominance of the market that many people in the UK call all ballpoint pens 'biros' no matter who makes them.) SteveBaker 21:22, 29 August 2007 (UTC)[reply]
Except there is no such 'biro' firm, which an internet search confirms, perhaps you meant...87.102.14.233 08:22, 30 August 2007 (UTC)[reply]
In other words, a genericized trademark. Titoxd(?!? - cool stuff) 06:48, 30 August 2007 (UTC)[reply]
Not a company Steve, but the inventor, László Bíró. The company you're thinking of is Bic, but is not where the name comes from. I forgot to look this morning. I will try to remember tomorrow! Capuchin 08:32, 30 August 2007 (UTC)[reply]

Could it be the reason red and green pens "are shit" is because nobody needs red and green pens. It's a sort of reverse reasoning which perhaps someone could tell me the name of..87.102.14.233 09:55, 30 August 2007 (UTC)[reply]

I'm not sure this works for "biros", but at least for regular fountain pens (which are much better for the environment anyways, if you excuse me saying so) black and blue ink are in fact different, with the black ink eventually jamming the pen (I think the culprit was soot, but may be completely wrong). I don't think it would happen in the short lifetime of a "biro", but since we're not talking about the difference between black and blue anyways - there's at least a slim chance that some other ingredient would just jam the pens faster. I lean towards the simple explication of the expiry, though. --Ibn Battuta 00:04, 31 August 2007 (UTC)[reply]

Speedy Than Light

I read somewhere that Hawkings had discovered thermal radiations thats coming from Black Holes. Black Holes are huge masses that wont even let light go away from it. So does it mean that the particles in "Hawking's Radiations" has velocity more than of light ?

Oasa 11:33, 29 August 2007 (UTC)[reply]

No, the photons are generated exterior to the event horizon from vacuum fluctuations (This is a simplified explanation, but is an intuitive way to look at it). See Hawking Radiation. Capuchin 11:43, 29 August 2007 (UTC)[reply]
If you are interested in faster than light phenomena, check out faster-than-light and the theoretical tachyon. Capuchin 11:57, 29 August 2007 (UTC)[reply]
These particles are not travelling faster than light. The black hole's event horizon does prevent things that are moving no faster than light (that is to say: "everything") from escaping. What's happening here is that there is a peculiar process happening throughout the universe where (even in a complete vacuum) a particle and it's anti-particle may spontaneously pop into existance for no particularly good reason (See Pair production). Generally, this is irrelevent because they collide and cancel each other out shortly afterwards. However, if one of those particle pairs happens to pop into existance close to the event horizon of a black hole then before they get to annihilate each other, one of the two particles may stray across the event horizon while the other does not. This prevents them from cancelling each other out. Hence, one particle (like maybe an electron) shoots off into space (which it can do without travelling faster than light since it's already outside of the event horizon) - while a negative particle (a positron perhaps) falls into the black hole. This effectively results in the black hole "emitting a particle" - whilst simultaneously losing a small amount of mass. Seen from a distance, it's just as if the black hole is emitting random particles while gradually 'evaporating'. This is explained quite well in Hawkin Radiation. It's only a theory though - it's not proven and many cosmologists are skeptical about it.
(What I don't understand is why we don't get a random mix of (say) electrons and positrons being emitted - which would still cancel out. Also, if the black hole is absorbing a random mix of particles and antiparticles - how come it 'evaporates'?)
SteveBaker 14:16, 29 August 2007 (UTC)[reply]
For the evaporation, the article explains it quite well (although oversimplified): "In order to preserve total energy, the particle which fell into the black hole must have had a negative energy (with respect to an observer far away from the black hole). By this process the black hole loses mass, and to an outside observer it would appear that the black hole has just emitted a particle.". As for the still cancelling thing, i'm not sure that there's any reason for the emitted particles to come into contact and annihilate is there? Capuchin 14:21, 29 August 2007 (UTC)[reply]
Yeah - I already read that. It says it's true - but I don't understand why. Why couldn't it be the particle with the positive energy that fell into the black hole (resulting in the hole gaining mass and Hawking radiation being negative energy)? This would allow a black hole to grow by 'devouring the vacuum'! Energy would still be conserved. Personally, I'd expect it to be entirely random whether the particle or the antiparticle that fell into the hole - resulting in (on average) no net loss or gain to the mass of the hole - and the resulting radiation would be a random mix of positive and negative particles - which would (presumably) annihilate each other almost immediately. The result of which would be: No Hawking radiation - no black hole evaporation. If those particles and antiparticles didn't annihilate each other - then Hawking radiation would be a random mix of particles and antiparticles - but the black hole would not evaporate.
So why the asymmetry between particles and antiparticles? Why does the antiparticle get swallowed by the black hole in preference to the positive particle? SteveBaker 15:15, 29 August 2007 (UTC)[reply]
There's no asymmetry between particles and antiparticles: both come out. The asymmetry is between positive-energy and negative-energy particles of all types, including both electrons and positrons. Positive-energy electrons and positrons (and everything else) tend to escape, while negative-energy ones tend to fall in. There's no connection between having a positive or negative energy, and being a particle or antiparticle. --Reuben 16:54, 29 August 2007 (UTC)[reply]
OK, so Someguy1221 made a good point (and then self-censored it!): that story about a pair of particles, one with positive and one with negative energy, is itself quite suspect as an explanation for how Hawking radiation works. In fact, it might even be totally bogus. It is the most common non-technical way of explaining Hawking radiation, and if I remember correctly, Hawking himself uses it in A brief history of time. I can't claim to understand the "real" version, but I think I know enough to answer Steve's questions.
  • The possibly-bogus explanation involves negative-energy particles going in and positive-energy particles coming out.
  • There's no particular correlation between negative/positive energy and particles/antiparticles.
  • Particles and antiparticles come out in equal numbers (it's a perfectly thermal distribution, so you can write a partition function).
  • This last point remains true even in the "real" treatment.
Here's a paper if you want the "real" version: [12]. I only skimmed part of it; it's good, but this is not light reading. --Reuben 17:49, 29 August 2007 (UTC)[reply]


Surely the positive/negative energy particle explanation is an outdated one from Dirac hole theory. They can only exist for a short time due to the Uncertainty Principle. The reason we may see more particles than antiparticles coming out is that antiparticles will annihilate with the many particles that make up the universe, giving off photons. As to why the universe has more matter than antimatter, that is another, much more difficult, question. Cyta 09:28, 30 August 2007 (UTC)[reply]
That might be a reasonable explanation for why Hawking radiation would consist almost entirely of regular particles and photons - but it doesn't explain why the black hole would gradually 'evaporate'. Ingestion of more 'negative energy' than positive seems like the only way that could happen and I don't understand why that asymmetry would exist. SteveBaker 14:13, 30 August 2007 (UTC)[reply]

Well Thanks to All of you I simply cant understand everything you told. I am just in 12th Std.Please explain to me this in that way I can understand. I understand that a particle and an anti-particle is formed (like the way in which a neutron splits into positron and an electron) at the event horizon. So why dont the black hole attract both of them before one of them escapes. One More thing: Black Holes gets destroyed(evaporated), is there any proof ? I have read about special relativity but i still cant take to my mind that why cant the newtonian and gallilean principles be applied. Once More Thank you!! Oasa 03:27, 2 September 2007 (UTC)[reply]

The black hole can attract both particles in, before either escapes. But sometimes, one of them just very luckily gets out. Remember, even if it's only a teeny itsy bitsy bit from the event horizon, it can still get away if it's going fast enough. There is no experimental proof that black holes actually evaporate, as we have not yet devised a way to monitor the very dim emissions caused by hawking radiation from the distance to the nearest black hole (I'm not sure if it's even possible at that distance). Someguy1221 03:19, 2 September 2007 (UTC)[reply]
Thanks, yet the word "Fast Enough" it must fastr than C right ? and what about Special Relativity Does the White Holes Really exist. Has anyone seen White Holes ? White Hole and Black Holes are Part of wormholes. So does it mean that All the matter that Black holes absorb is released through white holes (may be absurd)? One of my friend said that it is theorotically proved that we can travel to future ? WHat is the theory called so that i can look for it ? Oasa 03:39, 2 September 2007 (UTC)[reply]
Fast enough doesn't mean faster than C, it just means really darn close. Remember, light can still escape a black hole if it's going the right direction, right up until it hits the event horizon. So something going just a little bit slower can too. Someguy1221 03:35, 2 September 2007 (UTC)[reply]
Well to start with, you can certainly travel to the future, all you have to do is...nothing. You're traveling into the future right now! As for white holes, we have an article on that, although I can't promise any of it will make sense. But no, those have never been experimentally proven to exist either, as none have ever been found. There is large uncertainty as to wether they exist, mainly from the inability of quantum physics and relativity to describe what happens at a singularity, like that in the center of a black hole. Some physicists interpret thegeneral relativistic analysis of a black hole's singularity as suggesting that the future of a particle within a black hole always lies towards the singularity, and once it reaches that singularity, the future of the particle must lie yet somewhere else, leading to the suggestion of white holes and wormholes. But again, relativity can't actually explain what happens in a singularity with certainty, so we can only conjecture, and you can read white hol at your lesiure. Someguy1221 03:51, 2 September 2007 (UTC)[reply]
Sorry! I didnt get you . What do you mean by saying that by travelling in right direction we can escape from the black hole? The question about Relativity is that why can the general principles be applied. If a person travelling (with a velocity v) in the opposite direction to light doesnt that mean the velocity of the light is (V+C) to the observer. I reread the article but still cant make-out.Can you help me in the Future-Travel I mentioned above. Oasa 03:49, 2 September 2007 (UTC)[reply]
Ah, it seems my response to your latter question landed above you in the database lockup and subsequent edit conflict. Anywho, when I say in the right direction, I mean that if a photon is headed straight for a black hole, it's going in. If it's headed straight out, it'll leave (I'm not sure of the best angle for escaping a spinning black hole, though, might not be straight out). For those angles inbetween, depends on how close the photon is to the event horizon and how big the black hole is. As for relativistic velocity addition, I'm not sure I can give you any better explanation than that it is derivable mathematically from the assumption that light's velocity has the same measure for all observers. You might want to start a new thread on that at the bottom, and maybe someone else can offer a nicer explanation. Someguy1221 03:56, 2 September 2007 (UTC)[reply]


I meant to a distant future or a distant past. So as per you, if we head in a particular direction if we head then we will be able to escape less than the escape velocity. Is that possible in Earth too (less than 11.2 KM/s). What is the possible explanation behind it ? For Relativity I shall start a new topic. Once again Thanks .... Oasa 04:10, 2 September 2007 (UTC)[reply]
Well, no one has ever formed a convincing theory of time travel, just lots of conjecture. Read time travel for more. As for the escape velocity...I find it instructive to ask people to think in terms of extremes...For example, you state that the escape velocity from the Earth is 11.2 KM/s, now, this would be the same whether the Earth is right where it is right now, or alone in some distant part of the universe, away from all other objects. Now, imagine you are one light year from the Earth. You are still affected by its gravity to a degree, but do you think you need to reach the same velocity to get away? The answer is no. Think of it this way: You take off from the Earth's surface at escape velocity, and as you get further and further, the Earth's gravity slows you down below what you thought was escape velocity, but you're still escaping! Escape velocity only applies to a particular distance from an object. So, while the escape velocity from a black hole may mathematically be C at the event horizon, the escape velocity goes down as you get further away from it. Someguy1221 04:18, 2 September 2007 (UTC)[reply]
Thanks and a lot of thanks. Now my doubt is almost cleared,actually I meant it from earth's surface, but still I cant understand by "direction".

One More thing if we are attracted by earth, we also attract earth and hence both gets an acceleration.but the acceleration of earth is very very small because of its huge mass. My question there are so many persons,objects etc so does earth accelerate in that direction in which the vector sum of all these gets.Another thing because of the attraction of sun and earth, earth accelerates but why doesnt earth fall on sun like we do by the atraction of earth. i.e. why doesnt earth accelerate in a st.line towards sun and fall on it. instead it does on eliptical orbits.And how does the orbiting prevent the fall.

Well, I'll make one more attempt to explain "direction." Imagine you take off from Earth's surface with escape velocity. Except, you accidentally point your rocket straight at the Earth? Gonna escape? You have escape velocity. Now, yes, quite correct, the net gravitational force is the vector sum of all the little gravitational forces from everything in the observable universe (and the big gravitational force from the sun). So, as surprising as it might be, the Earth is actually accelerating more or less directly into the sun. But why doesn't it get any closer!?! Try reading centripetal acceleration, and come back if you still have more questions, always welcome! Someguy1221 04:47, 2 September 2007 (UTC)[reply]


Tanks again. I appreciate your patience. Actually I was trying to say that does the escape velocity depends upon direction. i.e. Consider A rocket projected at 30 degree and another with 90 degree. Will the velcoity they need to escape from earth differ ? I read but still cant understand "why doesn't it get any closer!?! " Oasa 05:19, 2 September 2007 (UTC)[reply]

(de-indenting for readability) Well, for normal examples, it shouldn't. As long as you're not heading into whatever you're trying to escape, you'll escape. You'll just go in an ever largening spiral instead of going straight out. I'm not sure if this is true for black holes, if maybe there is a general relativistic effect that might make it work out differently (i'm very sketchy on GR, so I wouldn't know anyway). Now, since I think we answered your original question here, I think the other two should also get new sections. But to try and answer that last one, the result of a force is to accelerate the recipient of that force. To accelerate means to change one's velocity. In the case of centripetal acceleration, since the direction of acceleration is always perpendicular to the direction of motion, this never imparts energy (and therefore, change in magnitude of velocity) onto the recipient of the force, by the work formula: . In fact, this is the same deal with spinning a tennis ball attached to a string about your head; once it's up and spinning, all you're actually doing is pulling it towards yourself, but as in the previous case, it never gets close. When accelerating an object perfectly perpendicular to its direction of motion, all you succeed in doing is changing its direction of motion. As a visual aid, imagine dropping a ball onto the ground, but from 1000 miles up. It falls straight down. If you throw it a bit, it'll land some distance from where you were hovering over. As you can imagine, the faster you throw it, the further it will land. If you throw it really really fast, it'll just take off and never come back. Now, try to imagine that there might be some inbetween, where the ball is just balanced between trying to fall and trying to get away. This is where things enter orbit (it doesn't have to be perfectly balanced. If it is not perfectly balanced, the orbit will be elliptical instead of spherical, and alternate between getting closer and farther from the Earth). I am appending to this section a nice picture of this. Someguy1221 05:39, 2 September 2007 (UTC) [reply]

Thank You all of you here ends my doubt. Truly I understood everything.Oasa 03:29, 3 September 2007 (UTC)[reply]

What effect do creatine supplements have on insulin levels?

What effect do creatine supplements have on insulin levels? Creatine affects insulin levels but I'm not sure exactly how. Up or down. Can it cause diabetes or other problems? Also what impact does this have on blood sugar level? --Gosplan 11:55, 29 August 2007 (UTC)[reply]

Creatine supplements#Safety states that there are no known problems associated with the supplements, except possibly muscle cramping. Algebraist 13:03, 29 August 2007 (UTC)[reply]
Note that if you are looking for medical advice, people on Wikipedia are not qualified to give it: Wikipedia:Medical disclaimer -- 21:34, 29 August 2007 (UTC) —Preceding unsigned comment added by 72.33.121.200 (talk)

Network subscribers in South Africa

Hey Wikis, i was wondering if any one could tell me how many subscribers are listed on the mobile networks in SA(Vodacom, Cell C,MTN,Virgin Mobile etc)I have already searched on wiki so thanks for that info....Also what would you consider to be luxury features on a cellphone and which are a basic needs. Any added info on this topic would be appreciated! Crazypinkster 12:32, 29 August 2007 (UTC)[reply]

For me, I consider these to be basic features:
1) Having a directory of names and phone numbers with the ability to both call those numbers and display those names when a call is received (a version of Caller ID).
2) Phone should display the number of minutes/days remaining before it goes dead.
3) Should save a list of missed, received, and sent calls (dates, times, and names/numbers).
4) Should hold a charge for a long time and recharge at home or with the cigarette lighter in a car.
5) Cell phone should take and store phone messages. Ideally these should be stored right in the phone, as opposed to making you call some number to get your messages, but that would only work if you had free incoming calls.
6) Good reception everywhere I need to use it.
7) No "roaming" charge.
8) Silent ring (vibrate) option.
9) No ability for the government to track you down by your cell phone (it should be anonymous).
10) Callers should have to enter a code number (which I've previously given them) after dialing the phone number, or it should ignore the call. This would prevent drunken bastards from calling me at 3 AM unless I actually know them. (Alas, my phone lacks this feature.)
Unimportant to me:
1) Games.
2) Multiple ring tones.
3) Being small enough to fit into a wallet.
4) Color screen with fancy graphics.
5) Camera (I have a real digital camera).
6) Speaker phone (nice idea, but it's going to drain those batteries too quickly).
7) Not having any buttons (the iPhone).
8) Internet access (I don't want to squint at the Internet through a 2 inch screen.)
9) Text messaging (how is getting sore fingers by typing in hieroglyphics better than talking ?).
Unfortunately, to get most of the important things, I had to accept some of the unimportant things. I have a Motorola V170 with a Tracfone plan; not perfect, but fits my needs fairly well. I live in the US and don't know if either of these is available in South Africa. StuRat 03:41, 30 August 2007 (UTC)[reply]
  • Although the two are related, there is a difference between what your cell phone is able to offer and what your cellular network is able to offer. This is what confuses me about the OP's question: Are you asking about basic/luxury needs on a cell phone with regard to the networks, or in general? For example I don't know of any cell phone that takes a voice message on the phone itself, usually voicemails / messages happen when the call diverts back to the network (because your phone is off / out of range / you didn't answer it) and then the network puts the call through to your voicemail bank, plays your customised message (if applicable) and records a message, then, according to your account's settings will notify you of a message by sending an SMS/text.


  • Another example is the storing of SMSs/texts on the phone instead of the SIM card - some phones do this, others don't, and the phone's that do will usually have a setting.


  • South Africa's networks are actually great. There is a bit of a fuss that it's "So expensive" - and yes, when comparing the costs around the world South Africa's telecoms don't fair very well; but having compared the networks in South Africa and the networks in Australia, I'd have to say South Africa's are absolutely gold. Almost none of the networks in Australia have free voicemail (compare with South Africa where they all have free voicemail). In South Africa on Cell C you can recharge your credit for as little as R5 (approx. $USD0.65), on Vodacom for R12 (approx. $USD1.80) and the credit doesn't expire for at least a month - compared with Australia where almost all minimum recharges are $AUD29.00 (approx $USD24.00) (There are some exceptions, such as a $AUD10.00 (approx $USD7.00 recharge with Optus where the credit lasts for 7 days!). As far as I know, all networks in South Africa don't charge you to check your balance, and it's always easy to do such as punching in '#101*' - compare with Australia where you get charged for this on certain networks -- and you have to call a hotline to get the information


  • Strictly speaking, originally, "basic" functions of a cellular network and cellular phone was to simply be able to make and receive calls much like you would on a landline phone - it "rings" when somebody calls it, and you can also pick it up and dial a number. Of course this evolved, and you could store contacts on it. This evolved to being able to store more information on contacts, and of course ring tones evolved. ... Flash-forward to the future, and now phones do a whole host of functions: GPS navigation, graphics, games, internet functionality, cameras (something I'm very critical of with phones), etc. I would still rate all of those as "luxuries", but as technology develops the market will become to dictate that a phone should have certain things as basic functionality - for example it would be 'bad' if a phone would only allow you to store 7-8 SMS/texts (consider the Nokia 3310 that used only store about a dozen) - now phones store hundreds of SMS/texts. There also used to be phones that were only compatable with certain 'bands' of network range, and now most phones are 'dual-band' and are more compatable with reception.


  • But to answer what network capabilities are considered "basic" and which are considered "luxury", I'd like to think that checking your balance for free should be considered basic, voicemail should be considered basic (I'll compromise and say "even if you have to pay for it"), SMSs should be considered basic, call diverts are basic, etc.<br /.


  • I hope this has been of some help
  • Rfwoolf 04:11, 30 August 2007 (UTC)[reply]
I haven't managed to find anything recent on Vodacom and Cell C or Virgin. Rfwoolf 04:30, 30 August 2007 (UTC)[reply]
I'm a bit of a minimalist when it comes to phones. The number one feature I looked for in my present phone was the shape of the charger socket! All of my previous three or four phones - along with those of my family - have failed because the weird multi-pin charger socket wore out and the phone wouldn't reliably recharge. My present phone has lasted longer than any previous phone I've owned because it has a 'conventional' Barrel connector. So - the nature of the power connector has become my #1 requirement on a new phone! SteveBaker 14:03, 30 August 2007 (UTC)[reply]

Helium Baloons

How many helium baloons would i need to take off or glide with, if i had them attached in clusters around my armpits and the back of my heel? (kinda the same pose in the cartoon where that boy flies with the snow man )(You know the one....) —Preceding unsigned comment added by 81.76.125.193 (talk) 15:17, August 29, 2007 (UTC)

42 weather ballons full Gzuckier 15:57, 29 August 2007 (UTC)[reply]


Well considering im a 16 year old weighing no more than 10 stone ( unlike the 33 year old, truck driver) it should be significantly less... also how can i avoid air fines (bearing in mind i live about 10 miles from cottesmore airport) where would i get the weather baloons, and where would i get a parachute, in england *cough* nanny state *cough*? answers appreciated ... —Preceding unsigned comment added by 78.144.55.130 (talk) 17:24, August 29, 2007 (UTC)

If you seek manned flight, I recommend the safer, slower process of becoming a certified pilot. This way, you will learn the appropriate regulations that will prevent harm to yourself and others. The process can take as little as a few weeks or months, depending on how well you progress. Remember - aviation rules are not only there for your safety, but for the safety of others. Imagine if your lawn-chair-balloon contraption interfered with a commercial aircraft flight - you would be personally responsible for endangering other people. Do not violate local laws by operating an unregulated aircraft. Nimur 17:32, 29 August 2007 (UTC)[reply]
See Cluster ballooning. It's more dangerous than you think, so you're pretty much on your own figuring out how to do it. —Keenan Pepper 17:43, 29 August 2007 (UTC)[reply]
Ah, you see, the nanny state is everywhere, not just the UK. But seriously this sounds like a bad idea. A fall from 20 feet could kill you as would a strike from a Greater Black-Backed Gull (see below)Richard Avery 17:59, 30 August 2007 (UTC)[reply]
Bear in mind if you do this in the US in particular in or near a no fly zone you might find some fighter jets heading your way. And if you're near commercial aircraft, particularly if you're wearing a rucksack or you look foreign well let's just say you may find you trip rather short and the landing far faster then you had planned (of course you may already be in pieces so you won't care) Nil Einne 22:04, 30 August 2007 (UTC)[reply]


"The movie The Red Balloon, made in France in 1956, ends with a simulated cluster balloon ride as a young boy, having lost his magical red balloon to a gang of vandals, then finds all the other balloons in Paris coming to him." Hmm........ A gang of vandals? I'm not sure what that would be like outside of Wikipedia. Juanita Hodges 17:08, 31 August 2007 (UTC)[reply]

odd question - spin inversion

If a proton, neutron, or electron is isolated in space (ie no magnetic or electric fields) then the energy of the two spin states is equal (I'm assuming the answer to this is yes)

1.Is the spin here considered to be undefined (eg a 'supposition of states' or whatever language term used to describe 'unknown until tested') or is a particles spin fixed even though the energy between states is zero.

2.If spin inversion occurs the energy change is zero, but the spin change is 1 - does this mean that a photon of energy 0 is emitted - does this count as a non event?

3.(ignore depending on Q.1) If the spin IS fixed - what rules govern spin inversion here - is there a rate of inversion, is it allowed or forbidden.87.102.18.14 15:42, 29 August 2007 (UTC)[reply]

It would be in a superposition of states, and if there is no field, then the distribution should be equal. In order to check the spin, you would need to probe the particle, which would typically require an applied magnetic field. The spin can't really invert until it has been determined in the first place (so what you describe in (2) is a "non-event"); detection of any emitted energy would mandate that the spin is first measured (i.e. an applied field). Nimur 17:28, 29 August 2007 (UTC)[reply]
You have to be careful here. A spin-1/2 system can't be in an equal superposition of all spin states; every possible state vector describes some definite spin axis (see Bloch sphere). On the other hand, a spin-1/2 system described by a density matrix can be in a spherically symmetric state, namely 1/2 times the identity matrix.
Regarding the original poster's second question, a free electron or proton can't emit a photon (or any other particle) without violating the conservation of energy-momentum. As you say, the energy of the emitted particle would have to be zero. -- BenRG 21:24, 29 August 2007 (UTC)[reply]
You think there is a spin axis?? (like a direction in space?)?87.102.14.233 08:59, 30 August 2007 (UTC)[reply]
Hm? Yes, there's a spin axis. Electron spin is described by a spinor, which points in a particular direction like a vector does. -- BenRG 21:34, 30 August 2007 (UTC)[reply]
I thought it merely had value - are you sure? I mean it's easy to quote articles.213.249.232.202 06:16, 31 August 2007 (UTC)[reply]
What I mean to say is that given that the spin can be either 'alligned' or 'opposed' to the field (since it is quantised) a concept of direction is irrelevent. It is the field itself which has a direction. Do you agree?213.249.232.202 07:21, 31 August 2007 (UTC)[reply]
It's tricky to talk about this because of the lack of a good ontology for quantum mechanics, but the following is true: Any wavefunction describing an electron spin system can be interpreted as a direction (ray) in space. If you do a Stern-Gerlach measurement along the axis of the ray, you'll find the spin to be pointing in the direction of the ray with probability 1. If you measure along a different axis, you'll find one of the two axis-aligned states with probabilities related to the angle between the measurement axis and the ray. To put it in purely operational terms, when you prepare a spin system you choose a particular spatial direction, and when you measure it you choose an axis, and the transition probabilities are a monotonic function of the angle between the two. -- BenRG 11:22, 31 August 2007 (UTC)[reply]
Though I appreciate what you are saying.. Isn't it impossible to measure the spin 'off axis' - you need a field to observe it - and there is only one field. (and the percentage of spins aligned/unaligned would depend on their thermal energy, but could be affected by light - causing low to high spin energy transitions).87.102.88.202 14:03, 31 August 2007 (UTC)[reply]
Yes, you only get one chance to measure the spin of any particular electron, and you have to choose a particular axis before you do the measurement. What I said above has to be taken as a statistical property of an ensemble of systems or as a Bayesian confidence level. And I ignored all the nasty experimental details that real physicists have to worry about. -- BenRG 23:46, 31 August 2007 (UTC)[reply]

Discouraging collared doves from my bird table?

I got a bird table for my garden a few months ago (I don't only just feed the gulls any more!). One problem though - I have way too many collared doves coming down and eating way too much birdseed. As well as being easily capable of devouring a 10lb bag of seed in two days, they also chase the other garden birds away if they approach the table. These critters look cute, sweet and innocent but they have an aggressive streak towards smaller birds - and they'll even drive the magpies and crows away. Just about the only bird that isn't phased by them are my great black-backed gulls, which have absolutely zero interest in eating seed in the first place. They're seriously worse than the starlings when it comes to being mob-handed, winged hogs.

Does anyone know of a way that I might discourage the doves from feeding in my garden? I don't want to shoot, poison or otherwise kill anything - and I don't want to stop feeding the birds. Is there such a thing as a 'dove proof' feeder I can hang on the table? --Kurt Shaped Box 17:37, 29 August 2007 (UTC)[reply]

You can certainly invent complicated feeders that birds have to hang upside down from (or something difficult like that) which would discourage birds like doves that can't do that. You could also devise something where the perch that the bird stands on to get at the food is a counter-balanced lever that closes a door and shuts off access to the food to any bird that weighs more than the counterweight - this could be used to shut out large birds and let the smaller ones have access to the food. You could make a large clear plastic cube and put the food inside - then cut holes as entrances and exits that are too small for a dove to get into. SteveBaker 19:38, 29 August 2007 (UTC)[reply]
You have black backed gulls? Why not stop feeding those FEARSOME PREDATORS, let them get hungry and angry, then let them deal with the doves for you? —Preceding unsigned comment added by 84.65.105.7 (talk) 20:32, August 29, 2007 (UTC)
No! It's worse! These are GREAT black-backed gulls. SteveBaker 00:29, 30 August 2007 (UTC)[reply]
That's quite funny - the thought of these gulls actually motivating themselves to hunt, that is. If I stopped feeding them, they'd just go and hang out near the house of the next 'gull friendly' person on the street. Or they'd rip open my rubbish bags. A lot of people on my street like the seeing the GBBGs feeding up close - just look at the general size and impressiveness of them!, so they tend to be somewhat 'pampered'. --Kurt Shaped Box 09:27, 30 August 2007 (UTC)[reply]
Is this a very roundabout seagull question? >:|      :) --frotht 22:05, 29 August 2007 (UTC)[reply]
(It has seagull in it...I think it counts.) SteveBaker 00:29, 30 August 2007 (UTC)[reply]
or you could make a protective cover out of what we call 'chicken wire' or 'wire netting' in the UK. You can buy it in several different sizes. I would think 2 inch holes would allow the little birds in and keep the bigger birds out....sorry for stating the obvious. On a slightly related ornithological note, we had no collared doves in the UK in the 50s then a few were brought over from the US for a private aviary and now we are similarly plagued with the damn things. Do they taste good? Richard Avery 17:53, 30 August 2007 (UTC)[reply]
I'm in the UK too, dude! :) While cute, the collared dove is very, very annoying. I don't mind the screeching of nesting gulls outside my bedroom window, the chattering of magpies or the warbling of an overzealous song thrush early in the morning - but that constant, repetitive 'coooo coooo cuk' from the doves goes right through me like a dripping tap. It's worst in the summer when it starts getting light at 4am - there's no way I can sleep through that racket. I figure that if they were good to eat, they'd be finding their way into pigeon pies (wot? no article???) - which they don't seem to be. --Kurt Shaped Box 18:08, 30 August 2007 (UTC)[reply]

I have had neighbors with birdfeeders or who just throw food out to feed birds. The result is noisy birds when I try to read or watch TV and lots of bird crap all over. I say don't use bird feeders. Juanita Hodges 17:10, 31 August 2007 (UTC)[reply]

You could put a roof over the feeder so birds have to duck to get to the food. A balcony over the entrance is used here in Australia to deter some birds from using nesting boxes. Polypipe Wrangler 07:47, 1 September 2007 (UTC)[reply]

The American Dental Association made a vaccine to Streptococcus mutans and then patented it and sat on the patent so ensure dentists stayed in business???

My dentist told me that in dental school a professor claimed that in the 1950s, the American Dental Association made a vaccine to Streptococcus mutans and then patented it and sat on the patent so ensure dentists stayed in business. I've asked him about it and tried to find information confirming if this was true or not. Google only finds that some people are working on a vaccine. The Streptococcus mutans article's is vague on the idea of a vaccine and it's one source on that is some website that's unviewable unless you pay them a lot of month first. Juanita Hodges 18:30, 29 August 2007 (UTC)[reply]

If they patented it in the 1950s, that patent would be expired now. The patent would have always been public and whatever it covered would have long since become unprotected as well, so anyone (drug company, private or university researcher, etc) could work on making it. That's the whole model of the generic-drugs business—patent protection is quite limited and once something is patented it soon becomes free-for-all. DMacks 18:40, 29 August 2007 (UTC)[reply]
I would add that taking out a patent on something is in no way proof that it actually works. You can patent the most outrageous concepts whether true or not. People have FAR too much respect for the patent system. It's perfectly possible that this guy did indeed patent this - but that doesn't prove that a usable vaccine has ever existed. SteveBaker 19:32, 29 August 2007 (UTC)[reply]
A shame, since otherwise I'd be getting to work in a British Rail flying saucer. GeeJo (t)(c) • 19:03, 30 August 2007 (UTC)[reply]
A patent is a bargain that society makes: you tell us how to do something neat, and we'll give you a temporary monopoly on making money off of it. There is no way to sit on a patent indefinitely, since that would break the inventor's side of the deal. You can try to get an extension, but not for 50+ years. For that, you need copyright. --Sean 19:46, 29 August 2007 (UTC)[reply]
And even then, copyright is not infinite. Well, at least not yet. Titoxd(?!? - cool stuff) 19:49, 29 August 2007 (UTC)[reply]
Trademarks, on the other hand, are infinite as long as they are still being used. But we digress. And please do not take this as legal advice! --Anonymous, August 29, 20:08 (UTC).
The pitiful article, Caries vaccine, points out that researchers are still working on a vaccine against tooth decay. It would be important to point out that although decay is still a very common problem treated in dental offices, you still have gum disease, root canal issues, trauma, cancer, and replacement of teeth that dentistry would still need to address even in the event of a 100% effective vaccine. ;) And those are the first things that come to mind. There are still many other problems. Thus, I, myself, am not too worried about dentistry staying in business. - Dozenist talk 00:05, 30 August 2007 (UTC)[reply]
Along that same line, there is actually a vaccine out for dogs now for periodontitis. It covers three different Porphyromonas species (see Porphyromonas gingivalis for the human version of the bacteria). --Joelmills 02:48, 30 August 2007 (UTC)[reply]
There is this article. As I recall, the biggest hurdle was the transfer of the bacteria by sharing glasses/kissing etc. Since you'd need a prescription to get the bacteria, you'd be giving it out illegally, or some other nonsense. In any case that is a place to start. I also did a Google search for "tooth decay bacteria genetic modification", and got some hits about modified sugar cane and other things along those lines. --Cody.Pope 10:19, 30 August 2007 (UTC)[reply]

Wind data

I'm looking for any sites with quantitative data I can use to show relationships between geographic features and/or seasonal variation with changes in wind speed. Any links would be appreciated. --Sopoforic 20:21, 29 August 2007 (UTC)[reply]

Google Search turns up some geographic sites on wind erosion? Might be of help SGGH speak! 21:14, 29 August 2007 (UTC)[reply]
Yes, but those are more geographical features created by wind than wind created (or channeled) by geographic features, although they both effect each other, I suppose. StuRat 03:05, 30 August 2007 (UTC)[reply]
this pdf also seems to have soem figures SGGH speak! 21:14, 29 August 2007 (UTC)[reply]
Well, I don't know about the World, but for the Great State of California, here's a place I drive by every now and again, their site with wind reports, http://www.energy.ca.gov/wind/overview.html --Jacobi am the kwisatz haderach 21:28, 29 August 2007 (UTC)

August 30

EFFECT OF ATMOSPHERIC PRESSURE ON BODY WEIGHT

I'M ON A DIET. SOMEDAYS I KNOW THAT I HAVE EATEN NOTHING AND YET MY WEIGHT HAS STILL GONE UP --ONE TIME IT WAS 5 LBS OVERNIGHT. A FRIEND SAID THAT ATMOSPHERIC PRESSURE OR BAROMETRIC PRESSURE CHANGES CAN AFFECT THE RESULTS OF MY STEPPING ON THE BATHROOM SCALE. IS THIS TRUE? AND IF SO, WHY? —Preceding unsigned comment added by Gtigue (talkcontribs) 00:37, August 30, 2007 (UTC)

If atmospheric pressure goes up, your apparent weight will be slightly less, because of buoyancy. Emphasis on slightly -- there's no way this is the real issue you're seeing. But your scale will be somewhat variable, so it could show your weight as higher when it's not. Or you might have drunk water. Or you might have forgotten about a snack. --Trovatore 00:47, 30 August 2007 (UTC)[reply]
Slight differences in foot positioning can affect cheap scales. Try weighing yourself 5 times per session (just step off and back on), and take the average. If you are still seeing a significant overnight increase in your weight, you might be a somnambulist.  :) --Sean 01:19, 30 August 2007 (UTC)[reply]
There are a lot of factors that can change the reading on scales. You can weigh different at different times of day, depending on how much water you've drunk recently, whether the scale is placed on a hard or soft surface, what you're wearing, whether the planets are aligned (ok that's only going to have an effect of a fraction of a gram, no matter what the astrologers say), and - this might be what your friend was thinking of - humidity. If you have a mechanical scale, especially a really cheap one, it may be possible for high humidity to affect the inner workings in such a way that the reading changes. So, the best way to get reliable weighings is to (a) buy a good quality, possibly digital scale, that is fully enclosed, (b) weigh yourself at the same time of day every time, wearing a similar amount of clothing, (c) try to make sure your eating and drinking habits are regular enough that you won't have drunk a litre of water before your weighing one day and not the next. Confusing Manifestation 01:41, 30 August 2007 (UTC)[reply]
I agree with humidity. I had just such a cheap spring scale that would vary by 5 lbs depending on the humidity. One clue that it was "sticking" was that the needle would go back and forth maybe 10 times before it settled on a reading when the humidity was low, but only once or twice when humid. Also, a change in barometric (atmospheric) pressure frequently goes along with a change in humidity, so your friend wasn't wrong. StuRat 02:03, 30 August 2007 (UTC)[reply]
The weight of air pushing down on the top of the scale is absolutely huge, 14.7 pounds on every square inch - that's probably more than a ton on the whole top surface of the scale! However, that same force is pushing on the bottom of the scale's cover too. And it's there even when you're not standing on the scale, so any effect from air pressure should simply be zeroed out. Even though you may not often think about it, the air weighing down on you from above is actually very heavy! --Reuben 02:16, 30 August 2007 (UTC)[reply]
No, not quite zeroed out. The air around your feet is under slightly higher pressure than the air around your head. This creates a slight net upward force, equal to the weight of the chunk of air that would have occupied the space where your body is, if your body weren't there. See Archimedes' principle for details -- haven't looked at that article but that's where it should be. --Trovatore 02:21, 30 August 2007 (UTC)[reply]
Right - that's a small but nonzero effect too. According to atmospheric pressure, air at sea level is about 0.08 lb / cubic foot. A human is a few cubic feet [13], so each of us displaces a weight of air of order 1/4 pound. Not big, but that's far more than I expected! --Reuben 02:33, 30 August 2007 (UTC)[reply]
According to this site: [14] "The lowest sea level air pressure ever recorded was 870 mb (25.69 in.) in the eye of Typhoon (Tip) over the Pacific Ocean, whereas the highest sea level air pressure ever recorded was 1084 mb (32.01 in.) at Siberia associated with an extremely cold air mass." That's about 24.6% higher at the maximum air pressure using the min air pressure as a base. If we multiply that by Reuben's 1/4 pound figure, we get around 1/16 pound differential in your weight from the highest atmospheric pressure to the lowest. The air pressure in your bathroom won't vary nearly as much, so air pressure alone can't explain a 5 lb diff. It's off by well over a factor of 80. StuRat 02:47, 30 August 2007 (UTC)[reply]
On another note, eating nothing at all for a day is not generally considered an effective way of losing weight, for a variety of reasons. Skittle 14:11, 30 August 2007 (UTC)[reply]

Gecko feet

Just read about Geckos and their sticky feet. Since their feet are so sticky, why don't they get totaly clogged up with dirt and lose their stickyness? -OOPSIE- 02:40, 30 August 2007 (UTC)[reply]

Perhaps they use the slug method and constantly ooze more slime, leaving the old "clogged up" slime in a trail behind them. StuRat 02:53, 30 August 2007 (UTC)[reply]
Gecko feet do not use a sticky slime to generate their stickiness, instead they have "a network of tiny hairs and pads on their feet which produce electrical attractions that literally glue the animals down. With millions of the hairs on each foot, the combined attraction of the weak electrical forces allow the gecko to stick to virtually any surface." [15] [16] I expect small dirt particles do not interfere with the Van der Waals forces between the gecko's seta and the surface of adhesion. Rockpocket 06:12, 30 August 2007 (UTC)[reply]
The setae somehow self-clean, but the mechanism is still a subject of current research [17]. --mglg(talk) 21:08, 30 August 2007 (UTC)[reply]
Thank you! -OOPSIE- 03:48, 31 August 2007 (UTC)[reply]

Physics

What is Radio Frequency Identification? —Preceding unsigned comment added by Beta alpha (talkcontribs) 07:32, August 30, 2007 (UTC)

Just look it up! Radio-frequency identification. —Bromskloss 07:52, 30 August 2007 (UTC)[reply]

Hunger related

What is the scientific term used to describe the condition when a person chooses not to eat and their body comes to a point where it no longer triggers us to let u snow we are hungry? - Pastor Jon--JacksonParkBC 12:18, 30 August 2007 (UTC)[reply]

symptomatologically, this is known as anorexia. Tuckerekcut 12:46, 30 August 2007 (UTC)[reply]
That's a more general term, though. I'm more familiar with what the OP's asking in terms of thirst. At first, you experience sensations of thirst, but after a certain level of dehydration, your body "gives up" on trying to notify you that you need water. I was always taught that if you're not thirsty (and circumstances suggest you should be), that can mean that you're extremely dehydrated. Anyway, I can imagine a similar situation with hunger. jeffjon 13:28, 30 August 2007 (UTC)[reply]

In regards to this link,

http://www.cnn.com/2007/US/08/30/spider.web.ap/index.html

What species of spider is this? Are there any spider species that work together with other species? --WonderFran 13:52, 30 August 2007 (UTC)[reply]

If the entomologist in the article can't identify them through the pictures, I doubt anyone here can. However, there is a social spiders section in the Spider article. -- JSBillings 17:07, 30 August 2007 (UTC)[reply]
This past spring there were quite a number of smaller versions of this phenomenon covering hedges alongside roads to the north of Salisbury, (UK). Those webs were made by the larvae of some species of moth that lived on the leaves of the plant it had covered as a form of protection from predators. I think they might have been tent moths, Lasiocampidae, although several other species spin webs. But in whatever case that is one huge infestation. Richard Avery 17:39, 30 August 2007 (UTC)[reply]

Time period for the deposition of calcium carbonate in Italy.

I have written the following footnote for a book I'm working on using information gleaned from Wikepedia.

The pure-white stone mostly associated with the word “marble” is the result of the metamorphism of pure limestone (calcium carbonate, whose origin was the deposition of the shells of marine organisms 65–100 million years ago). The heat and pressure in the metamorphic process usually destroys the fossil remains. This marble was used in Roman interiors and works of art. The most common Roman “marble” was travertine, a precipitate of carbonate minerals often aragonite but at times calcium carbonate. It has not undergone metamorphism and is often yellowish with visible embedded fossils. The stone blocks of the Coliseum are of travertine. The Latin name for the rock was lapis tiburtinus because of the large deposit quarried at Tibur (Tivoli).

The info for the time period of the deposit of calcium carbonate was from an article on the formation of chalk in England. Would this period also include the deposition of calcium carbonate in Italy? 69.201.141.45 14:04, 30 August 2007 (UTC)Linnaeus Shecut[reply]

(I just wanted to pick you up on a sentence you gave above - "..travertine, a precipitate of carbonate minerals often aragonite but at times calcium carbonate.." - aragonite is a form of calcium carbonate so.. you could just omit the "..but at times calcium carbonate.." - did that make sense.?
as for your main question - until someone else answers - I think it's best to say not neccessarily - the cretacious period is named after the chalk that formed at that time - but that doesn't mean that carbonates weren't produced in earlier aeons - that said - I've no idea.87.102.14.233 15:15, 30 August 2007 (UTC)[reply]
HOWEVER from the article "The Cretaceous is justly famous for its chalk; indeed, more chalk formed in the Cretaceous than in any other period in the Phanerozoic" - so it's a good bet that the chalk is cretacian - but I suppose it depends where it's found and at what depth...213.249.232.26 19:14, 30 August 2007 (UTC)[reply]
NOTE a web search for "marble formation italy" suggests that liassic is a common age quoted for the date of formation. —Preceding unsigned comment added by 213.249.232.26 (talk) 19:02, August 30, 2007 (UTC) UK chalk I believe formed in the cretacious - however it's up to you to discover whether the liassic date refers to the laying down of the sediment OR the partial metamorphosis to marble (disclaimer I'm not a profesional geologist)213.249.232.26 19:05, 30 August 2007 (UTC)[reply]
As suggested, the age range of various marbles and limestones in Italy can be large. The Carrara marbles are Jurassic but there are probably plenty of other ages as well. The lapis tiburtinus travertines are quite young - around 100,000 years [18]. The mineral name for the other crystalline habit of calcium carbonate (apart from Aragonite which as mentioned is also calcium carbonate) is calcite. Also note the correct spelling of Cretaceous. Cheers Geologyguy 19:37, 30 August 2007 (UTC)[reply]

geography/cartography issue, example included

I needed coordinates of the topmost easternmost, etc., points of a country, I used those specified in the article on the respective country. Here's what I get.

OK, there's an offset or something of some 40 km.

But, I get that it's not uncommon to specify this (i.e., coordinates of the northernmost esternmost, etc., points of a particular country) in literature dealing with this. I was wondering if anyone knows of some website or something that would have reliable information. I'm actually not sure how do I google this. Alternative methods of putting territories into boxes and getting the coordinates of the sides of the box would be even greater appreciated. 354d 16:09, 30 August 2007 (UTC)[reply]

Is it possible that one source is including the terratorial waters around the country and the other isn't? If the numbers in the article included the oceanic terratory and the map that you laid into that box did not - then there would be an error of around this much Perhaps there is some teeny-tiny island that belongs to Latvia that you didn't include in your map. This seems a very 'iffy' way of getting things to line up right. SteveBaker 19:01, 30 August 2007 (UTC)[reply]
I doubt there are territorial waters included cause they're like 200 km or something idk and it has no islands. Actually as I look at it there's some distortion - the top part's offset less than bottom - the overlain map appears to be somehow bigger although google maps is supposed to scale it.
The solution might be pulling the contours of the country from a snapshot of google maps (since it apparently has different projection) and somehow fitting the original map in there. Actually yea I thought earth is round and google maps have cylindrical projection - there has to be distortion.
No I'm wrong - there has to be a built-in "google-maps-pptimization" cause specifying a polyline for instance involves specifying real life coordinates and google maps dispalys it correctly so that's def not relevant. 354d 06:31, 31 August 2007 (UTC)[reply]
but anyway the coordinates in the wikipedia article are very crude or something. It would be cool if there was some google-maps-optimization function somewhere, but guess I have to ask this on google earth forums.
anyway any authoritative source on "country lies between ... and..." coordinates would be appreciated. 354d 05:01, 31 August 2007 (UTC)[reply]
Tell me - is the error much worse for a country further from the 0 meridian? If you pick a place like (say) India, is the error in the result worse? What about for a country closer to the equator? This could be to do with the map projection that Google uses - or because you are failing to correct for WGS84. SteveBaker 14:29, 31 August 2007 (UTC)[reply]

I just made a test (could've done it earlier :D) I specified coordinates that I took myself manually (arbitrarily or whatever) and the map gets positioned pretty much where it should be except for the eastern part where it is clearly visible that there's something wrong with the overlay map (and I'm pretty sure there is something wrong with it) and the second conclusion is that the coordinates in the article are very robust or whatever. Coordinates they (Google) use in their demos work pretty much to a zoom level where you can see cars so I'm really not sure that it's a projection issue or something. I should review sources rather, it might be a very error prone process using something they made back in USSR, I'm pretty sure they didn't abide by any WGS84 rules. 354d 10:20, 2 September 2007 (UTC)[reply]

Bachelor of Science in Engineering vs Bachelor of Engineering

I have a BSc in Engineering, my work colleagues have a BEng. We are trying to figure out what is the difference between the two and if one is better than the other. —Preceding unsigned comment added by Pebbles82 (talkcontribs) 16:12, August 30, 2007 (UTC)

If the degrees are from different schools, there may be no direct comparison. Schools may choose their own nomenclature. You might look under accreditation guidelines to see if both schools are accredited - if so, they must meet certain minimum requirements. If it's boasting rights you seek, why don't you compare number of classes you had to take? That will clearly settle which of you had more requirements in college; but of course there is always room for subjective interpretation. You might also look into Philosophy of education - our article details some of the different viewpoints about quantity and quality of coursework. Nimur 16:33, 30 August 2007 (UTC)[reply]
We also have Bachelor of Engineering and Bachelor of Science articles, but you should be aware that regional and school variations may supersede information in those articles. Nimur 16:35, 30 August 2007 (UTC)[reply]
I don't think there is any kind of formal distinction. Lots of British universities award 'BA' (Bachelor of the Arts) degrees for science subjects - whilst others award the 'BSc'. There is no significance in that beyond tradition. I would expect the same to be true of a BEng versus a BSc. To know which is 'best', I think you'd have to look at the quality of the institution that awarded it. In the minds of most employers, a 'B-anything' from Harvard, Yale, MIT or whatever trumps a 'B-whatever' from some of the lesser colleges - even if the name is the same. SteveBaker 18:35, 30 August 2007 (UTC)[reply]
Not certain here, but I think that by 'lots of British universities' you mean 'Oxford and Cambridge': certainly that isn't common practice here in Britain. Algebraist 23:28, 30 August 2007 (UTC)[reply]
Are those really the only ones? Well - I guess it could be - my two friends who have BA's in science subjects both went to Oxford and my BSc comes from Kent - so that fits the facts. But anyway - there is no special significance to that beyond tradition. SteveBaker 01:20, 31 August 2007 (UTC)[reply]

SOAP Notes

HI,

I am a speech-language pathologist with the current position of instructor/clinical supervisor at Southern Illinois University. I am teaching our graduate students how to write a SOAP note note this semester. I found your definition great for doctors, however, SOAP notes for other professions (ours, counseling, etc.) are a little different as to what goes into each section. It would be great to see different examples pertaining to other fields that utilize SOAP notes.


Thank You, Shawna Pope

—Preceding unsigned comment added by 131.230.22.145 (talk) 16:34, August 30, 2007 (UTC)

Email removed to prevent spam - Nimur 16:36, 30 August 2007 (UTC)[reply]

Simple Object Access Protocol? Sorry, I guess you meant a SOAP note. I have to say, in the engineering fields I work with, there's not much room for subjectivity. If, for example, I am diagnosing a circuit error, I simply record the appropriate characteristics (voltage, current, pin diagrams, etc. for example). This pure objectivity is, by design, a part of the philosophy of engineering (with good reason, in my opinion - though this point may be debated). I've often wondered if biology and the social scientists could improve dramatically if they would work a little harder to remove subjectivity from their world-view; but I recognize that those disciplines address very different types of problems. Maybe this question should defer to someone in a different area... Nimur 16:44, 30 August 2007 (UTC)[reply]
The article should be enough for any provider-patient relationship. First, let the patient explain his or her problem. Then, the provider gathers information on the patient through any tests the providers deems necessary. Then, the provider assesses the problem(s) that need treatment. Finally, the provider prescribes a plan for treatment. -- Kainaw(what?) 16:58, 30 August 2007 (UTC)[reply]
Shawna: Are you saying that you understand what is needed for a SOAP Note in your profession and that you wish our article covered that? Or are you perhaps saying that you don't understand how a SOAP Note would look for your profession? From the way you phrased your question and the fact that you are an instructor - I presume that it's the former and that you are simply pointing out a flaw in our article. If that's the case, then could I plead for you to write up whatever is missing and add it to the article yourself. Wikipedia only exists because people who know the subject material write about it - the contributions of a professional in the field (such as yourself) would be greatly welcomed. SteveBaker 18:27, 30 August 2007 (UTC)[reply]

Time, space and the universe.

Hi there, my question is simply this :- Assuming the universe is exactly 13.7 billion years old, if you had a telescope capable of looking back 13.7 billion yrs and 1 minute, would you actually see the big bang happen? ( Also assuming you were pointing it at the right point in space ). I can fully relate to " looking back 10 billion yrs " but seeing the big bang happen ( all be it with a super telescope ) is perplexing me. I hope someone can answer my question. Many thanks. Symon. —Preceding unsigned comment added by Gun dog 1 (talkcontribs) 17:20, August 30, 2007 (UTC)

The problem is that the big bang wasn't just a teeny-tiny dot somewhere in an infinitely empty space that kinda exploded and tossed out matter in all directions. It was the fabric of space and time itself that was a dot (a 'singularity' to be more exact). So the curvature of the universe was infinite and there was literally nothing other than the singularity itself...no 'outside'. So you couldn't be looking from 'outside' of the big bang and watch it going off like a stick of dynamite. No matter which way you look, you are looking towards the big bang. Worse still, time itself started with the big bang - so there is no 'before'. You could look back to (say) one nanosecond after the Big Bang - but space would be so screwed up that I'm not at all sure there would be things like photons flying out from it. Using specialised instruments mounted on satellites, we can see the Cosmic microwave background radiation which formed about 300,000 years after the Big Bang - that's pretty amazingly close to the beginning of time and space (compared to 14 billion years anyway). We could theoretically look back to within 2 seconds of the Big Bang by examining the Cosmic neutrino background - but neutrino's are insanely difficult to detect and a typical neutrino "telescope" is something that lives in a deep mineshaft and has a couple of swimming pools worth of dry cleaning fluid in it and can detect a couple of neutrino's every month. This is not the kind of instrument that's going to give you much in the way of a photograph! So, no - you can't watch the Big Bang going off. SteveBaker 18:17, 30 August 2007 (UTC)[reply]
Further to SteveBaker's response, the reason we need neutrinos or something to see further back is that before c. 380,000 years after the big bang, the universe was so hot and dense that it was opaque to photons (see Cosmic background radiation#Features) Algebraist 18:22, 30 August 2007 (UTC)[reply]
We can see artifacts of the expansion and the inhomogeneity of the early universe - see W map. Nimur 19:19, 30 August 2007 (UTC)[reply]
A question as this, you may enjoy an authors take on timeline and understanding of beginnings and ends. Gabriel García Márquez' One Hundred Years of Solitude. Note the studying of the Melquíades Parchments. --i am the kwisatz haderach 19:50, 30 August 2007 (UTC)[reply]
Stephen Hawking wrote 'A Brief History of Time' - it is a very approachable book on this subject and was actually a best seller when it was first published. SteveBaker 01:17, 31 August 2007 (UTC)[reply]

Colored water on plants

What is the effect of colored water on plants such as daisies or roses? —Preceding unsigned comment added by 71.166.60.75 (talk) 19:35, August 30, 2007 (UTC)

This is commonly used to give artificial color to flowers. Some dyes will be absorbed by the plant and show up in the flowers, others will not. -- Kainaw(what?) 22:59, 30 August 2007 (UTC)[reply]
And some dyes spread relatively evenly through the flowers while other remain concentrated around the veins. StuRat 03:28, 31 August 2007 (UTC)[reply]

Looking back in time

Hi, i wanted to know that if you had a powerful enough telescope could it look back in time to places on Earth, more specificly could it be programed to look back to a certain time period on Earth, e.g may 2000. If so how long would it take and how would it work.

Thanks Bailster2k7 20:30, 30 August 2007 (UTC)[reply]

If you had a sufficiently powerful telescope, yes, you could look at Earth circa 2000, although you would have to be 7 light years from Earth. It would take at least 7 years to get 7 light years from Earth, so even if you left right now, by the time you got there you'd only be able to look at right now. Further, the telescope would only be able to see the light coming in, "at-the-moment." So, if you have situated it 7 light years from Earth, it will always be seeing what happened on Earth exactly 7 years prior. Someguy1221 20:47, 30 August 2007 (UTC)[reply]
Check out the discussion on the space-time continuum just up the page. — Lomn 21:15, 30 August 2007 (UTC)[reply]
Of course, if there was a mirror 3.5 light years away from Earth, sufficiently large and facing the right way, then you could use the super-telescope to view things in the mirror. --Anonymous, 21:57 UTC, August 30, 2007.
I suspect that nicely polished large mirrors in space are unlikely...although I suppose that it wouldn't be beyond the bounds of possibility to find some light path that passed close to two or more massive objects, say neutron stars or black holes, whereby you could receive some photons in your telescope that originally came from the Earth in the past. The problem being that this path would likely be short-lived due to the changing relative positions of these objects and the Earth - and I suspect that you'd receive only a very tiny proportion of the photons due to the accuracy needed to get any to return back to the Earth. Also, I suspect the nearest things massive enough to deflect the light sufficiently would give a very long light-path of at least several thousand years.Richard B 23:07, 30 August 2007 (UTC)[reply]
We had this in answer to another question a few days ago. Basically, there is no way to see further back in time than the day you launch your mirror - and even then, you'd have to launch it at the speed of light. Of course if you launched your mirror - waited a few hundred years to get a few lightyears out there - then you'd be able to look back to a point a few years in the past...but not before the day you launched the mirror. There would doubtless be practical problems with making a large enough mirror and steering it so it would still be looking at the earth as the earth goes around the sun...but I guess this is a thought experiment. It would be cheaper and easier to stick a bunch of cameras into orbit with video recorders hooked up to them...the result would be pretty similar. SteveBaker 01:12, 31 August 2007 (UTC)[reply]
Think about this - "seeing into the past" is going to be a lot like using Google Maps (many of those photos are a decade old). SteveBaker 01:14, 31 August 2007 (UTC)[reply]
You are separated from your past self on earth by a time-like interval, so there is no mechanism consistent with modern theory that would allow you to view your past self (regardless of where you travel to before setting up your hypothetical telescope). Nimur 01:31, 31 August 2007 (UTC)[reply]
Hmm, aren't I looking at my past self when a look in the bathroom mirror (albeit very recent past self). Richard B 13:25, 31 August 2007 (UTC)[reply]
Oops! SteveBaker mentions an important correction below (which I thought was implicit, but should have been clearly stated). Nimur 16:21, 31 August 2007 (UTC)[reply]
Yes, you are...and you could set up a mirror on Alpha Centauri (4 lightyears away) and see yourself as you looked 8 years ago. What you can't do (even with a brand new mirror) is look back at yourself at a time in the past before you placed the mirror there. But as I've pointed out - cameras and VCR's make for an easier-to-understand version of this problem. You don't need mirrors the size of a planet and telescopes of mythalogical power in order to explore this concept - it's actually very familiar to us. When you watch a TV show that you recorded on your VCR - you are watching something from the past. SteveBaker 14:22, 31 August 2007 (UTC)[reply]

Flies in the dark

What happens if you switch off the light while a fly (or some other insect which is usually active during daylight) is still flying around? Do they crash into the nearest wall? Can they instantly switch to some type of "night vision" (if so, how?)? At least some insects do seem to see rather badly at night. Or let's just assume it's really pitch black. Do they fly more cautiously? Do they have a way of "extending their legs, so they'll land instantly on whatever they touch"? Or what happens? ... Thanks, Ibn Battuta 23:02, 30 August 2007 (UTC)[reply]

  • It's an interesting question, but I note that houseflies will routinely fly at full speed into a glass window pane with no obvious ill effect. --Sean 01:19, 31 August 2007 (UTC)[reply]
  • Agreed, I see no sign that they watch where they're going even during the day, they just bounce off the walls until they find something they can eat or something eats them. Sort of Roomba logic. StuRat 03:22, 31 August 2007 (UTC)[reply]
I can imagine that they could sense a difference in air movement near a surface. So maybe they slow down very quickly (flies can change course extremely fast) and don't quite hit the wall/window at full speed. However, being very small makes them light, which means the momentum and therefore the force exerted on their bodies can't be very high. Being small also means they can withstand more force. So maybe they wouldn't even need to slow down. DirkvdM 06:40, 31 August 2007 (UTC)[reply]
Houseflies are naturally diurnal and do most of their foraging during the daytime. They have apposition eyes and navigate by using polarized skylight. The ommatidia of apposition eyes have a light-absorbing screening pigment, so the only light that reaches the rhabdom enters through the small lens. Thus by dusk their eyes capture insufficient light to allow foraging, and most of their activity ceases for the day. In contrast, nocturnal insects such as moths, beetles, cockroaches and locusts have superposition eyes, a design based on the superposition of light rays entering hundreds, if not thousands, of ommatidia. This permits them "to see in the dark" and forage at night.
The bees with poor vision which you refer to have apposition eyes, yet despite this, tropical sweat bees (Megalopta genalis) are nocturnal and appear to see rather well at night. Scientists have found unusual interneurons in the optic ganglion, of these nocturnal bees, that have significantly wider lateral branching. The scientists hypothesize these neurons permit "a strategy of photon summation in time and in space" thereby enhancing night vision.
As it happens houseflies also have an very interesting neural modification: each ommatidia has seven rhabdomeres. This permits signals from adjacent ommatidia to interact within the neurons of the fly’s brain system. The result is fast directionally selective motion detection, which enables the fly to maneuver perfectly in three-dimensional space. This type of adaptation is called a "neural superposition eye" (even though it is a type of apposition eye). There is some debate over whether having a neural superposition eye provides significantly better night vision also [19]. It is generally thought that neural superposition is better than a regular apposition eye, but not as good as a regular superposition eye. [20] So its likely flies are not completely nightblind, but are still restricted. I don't know what a fly would do if the lights were suddenly turned off, but I expect they would probably settle on a nearby safe surface, in a similar manner as we navigate in the dark (relatively gingerly, with "arms" extended), until their eyes have suitably adjusted. Then they may fly around if need be, but because their natural behaviour does not support the requirement for night foraging, I expect they would probably take it relatively easy until the lights came back on. Rockpocket 08:42, 1 September 2007 (UTC)[reply]
Ahem. --Sean 12:43, 1 September 2007 (UTC)[reply]
Ah. DirkvdM 08:08, 2 September 2007 (UTC)[reply]

Titration and pH meter/indicator

What did chemists use to determine pH of chemicals before the electric pH meter was invented? Seeing how determining the pH of chemicals are very important, the instrument that measures it has to be accurate. —Preceding unsigned comment added by 128.163.224.103 (talk) 23:30, 30 August 2007 (UTC)[reply]

Why, litmus paper! I remember it well :D. I didn't even know there was an electric pH meter SGGH speak! 23:48, 30 August 2007 (UTC)[reply]
I've never seen an electric pH meter, but when I did A-level chemistry, most titrations were done with phenolphthalein: it doesn't have the range of litmus, but it's not bad for locating the acid/base changeover fairly precisely. Algebraist 00:19, 31 August 2007 (UTC)[reply]
See pH indicator for more useful chemicals. Algebraist 00:22, 31 August 2007 (UTC)[reply]
But aren't those methods not accurate in terms of getting the exact number? In other words, they're merely approximations. In acid/base calculations, such as the Henderson Hasselbach equation, and in doing titrations doesn't the exact pH need to be known? 128.163.224.103 00:54, 31 August 2007 (UTC)[reply]
There was no way to get an exact (accurate) number, before the electronic pH meter. All you could do was approximate based on calculations. Even using the most sophisticated derivations of the Henderson Hasselbach Eqn, there is a level of error that can't be accounted for. Ionic shielding and activity coefficients try and compensate, but there is no replacement for using a pH meter.Mrdeath5493 01:25, 31 August 2007 (UTC)[reply]

Mythbusters

On Mythbusters last night they tested using a grappling hook shot from a moving car onto a stable structure to make fast 90 degree turns. When they used a steel cable rated for 15,000 pounds, it broke. How much pressure would actually be on a cable going from a car to a structure at 30 miles per hour making a 90 degree turn? 68.231.151.161 23:40, 30 August 2007 (UTC)[reply]

By pressure I assume you me force? F=M*A?? At the time the cable became taught, acceleration was zero to thirty miles an hour (I think) which is 48 280.32m/s/s if the cable goes taught over one second. But without the mass of the car, and the time it took for the cable to get caught, I don't know if it can be worked out. I predict that I am about to be proved wrong though! :D SGGH speak! 23:47, 30 August 2007 (UTC)[reply]
I think they said the weight of the car was 7000 pounds. —Preceding unsigned comment added by 68.231.151.161 (talk) 00:41, 31 August 2007 (UTC)[reply]
We'd also need to know the length of the cable. Someguy1221 00:46, 31 August 2007 (UTC)[reply]
Yep - we sure would. We need the centrifugal force - but to calculate that, we need the length of the rope...which we don't know. F=mw2r - but we don't know 'r' (the radius) and to calculate w (the rotational velocity) given only the 30mph linear velocity, we also need the radius. Since w is inversely proportional to r, the final force is proportional to 1/r. So if we guess the radius wrong by a factor of two, we'll be off by a factor of two when we work out the force on the rope. Can't calculate it...sorry! I'm kinda surprised the force was that great though - if a 7,000lb car broke a 15,000lb rope - that's a lateral force of 2g's...this surprises me because my tricked out MINI Cooper takes turns like that at around 30mph without resorting to ropes and it can only manage about 1g of lateral. I guess that turn was tighter than it looked. SteveBaker 01:03, 31 August 2007 (UTC)[reply]
No need for angular momentum formulas; for circular motion a = v²/r. If a 7,000-pound car (Is that right? What was it, an SUV?) exerts 15,000 pounds force on the rope, we have a = 15/7 gees = 21 m/s². And v = 30 mph = 13.4 m/s, which gives r = 13.4²/21 m = 8.55 m = 28 feet. The radius needed to produce the same force in the rope is proportional to the car's mass, so if a the car weighed say 3,000 pounds they'd have to have tried a rope only 12 feet long.
I haven't seen the episode; could the turn they tried have been that tight? If not, I'd guess that the rope was not capable of supporting its rated force when it was applied suddenly. That seems possible intuitively but I don't know if it's likely in terms of materials science.
--Anonymous, 01:34 UTC, August 31, 2007.
It was a tight turn, probably well under 2 car lengths, meant to be tighter than one could do with a car alone. Also, since they were doing it without the aid of the steering system, there would be an additional substantial force created by the friction of trying to drag the tires sideways. 76.225.157.167 07:28, 31 August 2007 (UTC)[reply]

A critical factor that you have to know is the elasticity of the cable. The cable must stretch as it goes from no load to full load, lest the loading (and change in the car's velocity) take place instantly and the force on the cable rises to infinity (owing to F=MA). In a successful experiment, the load on the cable as it stretches won't exceed the breaking strain of the cable.

Atlant 12:44, 31 August 2007 (UTC)[reply]

The force is not infinite, and neither is the acceleration; they are mv²/r and v²/r respectively. The quantities that do become infinite if there is no stretching of the cable are the yank and the jerk. As I said, it seems possible intuitively that the cable could break under a big yank, but I don't know if it's likely in terms of materials science. --Anon, 22:30 UTC, August 31, 2007.
And that's precisely why the cable must stretch. Consider: The car is speeding away and the slack in the cable is being taken up. Suddenly, the cable draws taut. What happens? If the cable is not elastic, either it instantly stops the car (which, by the way, did represent infinite acceleration and exerts infinite force on the car and so is unlikely) or it instantly breaks (also unlikely). In the real world, the cable is elastic and stress and strain both begin building in the cable. Eventually, either the cable exerts enough force to constrain the car's motion or the cable snaps.
Atlant 00:02, 2 September 2007 (UTC)[reply]
I was assuming that setup had the car running tangentially to the circular arc at the moment the cable was attached — in other words, that the cable was attached while extending at right angles to the car's initial path. In that case a non-stretching cable would produce an infinite jerk but not an infinite acceleration. But of course that would be difficult to arrange in practice; it'd be much easier to make it at least a bit diagonal, and no doubt they did. My apologies for missing that point. --Anonymous, 05:30 UTC, September 3, 2007.

August 31

Have just been reading this article, and am finding it really difficult to wrap my head around. I can understand all the other universe expansion, theory of relativity things (was always very good at physics) but I'm hoping someone can give me a simple analogy for the Fourth dimension.I'm looking at my monitor, it has an "arrow" for up (height) and arrow away from me (depth) and an arrow running across my (width). With time as the Fourth dimension, does this mean it has another arrow pointing somewhere else representing this progress of the monitor through time (progressing along with me)? If so, where is this arrow pointing? The "time-vectors" of all objects must be the same length otherwise we would all be jumping through time at different speeds? what?! huh? Help! SGGH speak! 00:17, 31 August 2007 (UTC)[reply]

Thinking of time as a fourth dimension is very hard to wrap one's mind around. Instead, try to imagine time as the third dimension in some imaginary 2 dimensional universe. In this universe, everything is only described as width and depth, there is no height. Imagine also that this universe is finite in size, shaped like a square. You are looking at it now. Any given moment in this universe can be captured in a 2 dimensional image of the universe, like individual frames of a video. But instead of playing the frames continuously to observe the universe in motion, stack the frames on top of one another. You have now introduced height, where height is time. Any particle that remains motionless is a straight, vertical line in in your 3D space-time. If an object is in motion, it will trace out a more complex curve through this 3D space-time. If you slice out any cross section of this space-time, you have captured for yourself a freeze-frame of the universe in a given moment. I hope that gives you some guidance. Someguy1221 00:31, 31 August 2007 (UTC)[reply]
that is actually really helpful, thankyou! So in essence, time's 4th dimension is the 3D universe "stacked" ontop of itself for each frame? And the movement of these complex 3D shapes would produce some very complex curves indeed if you managed to link them all together dot-to-dot and view the universe kind of... "side on" if we use that 3D square model of yours. So in theory, as height in your 3D model has a vector and a direction, time has a vector and a direction? Does this mean time has a direction of travel that is relative to length width and depth? If an object could somehow move through the frames quicker than other objects, would it have a larger vector? Or have I extended your model too far? I'm basially asking if time as a front, like your length and width and depth has finite ends, is an object at the end of its "time" at all times because it is in the present, just as the edge of my tv screen is at the end of its length/width/depth? SGGH speak! 00:48, 31 August 2007 (UTC)[reply]
I'm not sure how to answer all of those, but I'm sure someone else will come along soon. Time is always pointing straight up in the example I provided. The directional change is in the total "space-time" vector. As for the "ends" of a particle's extent through time, the only ends would be wherever the particle is created or destroyed, otherwise it will continue on through the time dimension (unless time itself ends at some point). You may be interested in reading Minkowski diagram, a scientifically rigorous method of diagraming 2D space-time. Someguy1221 01:06, 31 August 2007 (UTC)[reply]
The traces of a 4-dimensional object in three-space are solids. It's possible for a single 4-dimensional object to create multiple solid objects as the traces in 3D, just as a single 3D object (say an octopus) can create multiple 2D curves (cross sections) in any 2D plane. StuRat 03:12, 31 August 2007 (UTC)[reply]
It's worth noting that there are different types of "4 dimensional space" - spacetime mentioned above is one, another is simply the addition of a fourth orthogonal axis to the 3 (length )dimensions most people experience - see Fourth dimension. These are often called 'einsteinian' and 'spatial' fourth dimensions respectively.213.249.232.202 06:06, 31 August 2007 (UTC)[reply]
"Most people"? Are there cases of people who think they live in a 4D world? Does WP have articles on them? Aaadddaaammm 09:05, 31 August 2007 (UTC)[reply]
"...the 3 (length )dimensions most people experience " 3, three...????????213.249.232.202 09:46, 31 August 2007 (UTC)[reply]
Sadly, almost everyone seems to fail to understand what 4 dimensional space means. Maybe an elementary course in linear algebra could help, but, in the meantime, don't panic: 4 dimensions means having "4 variables" in which to store information, nothing more, nothing less. Living in a world of 4 dimensions means we have 4 variables attached to every object in our universe, 3 for what we call "space" altogether and one for time. We could imagine a world of 12 dimensions too, just think of having 3 for space, one for time, one for its angular momentum, another for its favourite football team... --Taraborn 10:38, 31 August 2007 (UTC)[reply]
Actually the title article Fourth dimension describes four dimensional space (not einsteins space time) 'quite well'?87.102.88.202 13:57, 31 August 2007 (UTC) I can't find copies of equations for rotations etc in four spatial dimensions on wikipedia - maybe are expected to work those out for ourselfs?87.102.88.202 14:07, 31 August 2007 (UTC)[reply]
It's better to use linear algebra for this. Represent positions as vectors, represent rotations as vectors. The 'equation' for N-dimensional rotation then becomes nothing more than an N-dimensional matrix multiplication which is easy to understand for any value of 'N'. So take some time to soak up some 3D linear algebra. Once you've seen how to make a rotation matrix in 3D and how to multiply matrices in 3D, the extension to 4D is obvious and natural. Incidentally, in the world of computer graphics, 4D matrix arithmetic is what we use all the time for doing 3D graphics because it allows translations and perspective calculations to happen naturally and at the same time as rotations. Hence computer graphics hardware already works in 4D! SteveBaker 15:16, 31 August 2007 (UTC)[reply]
Ooops sorry - I alreagy had that (interesting to consider rotating about planes etc - but impossible for me to visualise) - I've caused you unneccessary typing - I meant that wikipedia doesn't have the answers worked out for us for higher dimensional transformations (can't find the page and imagine it doesn't exist) - sort of rhetorical question but not very well worded. Perhaps such a page should exist?
(Thinking about it aren't modern GPU's set up specifically for 3d - in terms of their parallelism - for quickly performing projections and rotations etc - maybe they wouldn't work so well with more parameters because the instr set doesn't support it - trivia)

87.102.88.202 16:03, 31 August 2007 (UTC)[reply]

(Oh I guess you meant the VMX type instructions with 4 32 bit values per register - yes that could be useful for 4D)87.102.88.202 16:06, 31 August 2007 (UTC)[reply]
Firstly, it is not at all clear that time should be considered to be a 'fourth dimension' that can be treated as space is. There are many time-effects (such as entropy and our human perception of time) that are quite unlike any effect we see happening in the three space-like dimensions. So we should really consider 'time-like dimensions' and 'space-like dimensions' separately. When you do that, the question of how the world would look if it had four space-like dimensions is the real question - and it is a tricky one.
An important first part of the question is: "How does 3D space look?" - you may think you know the answer but you don't. Our eyes are only 2D devices - we see images projected onto a 2D retina and our brains somehow manage to interpret this as a 3D world. (People will argue that we have two eyes and stereo effects let us see in 3D but that's bogus for a lot of reasons - close one eye and the world still looks pretty much the same). So what we actually see is a PROJECTION of the 3D world onto our 2D retinas. If you want to know what a 4D world would look like when projected onto our 2D retinas - just write a computer program to do that. It's easy. There are a bunch of them out on the Internet. Here is a game that you can play in 2, 3 or 4 dimensions.
Are there people who think they live in a 4D world? Oh yes! There are string theorists who believe that we live in a 26 dimensional world. Do these people SEE in more than 3 dimensions? No - they still have 2D retinas just like you and me. If the universe had more than three 'ordinary' space-like dimensions then photons would move in all four dimensions and the world would look like it does in that 4D video game - and we'd see that and would have evolved to understand it too.
Since that's not the case, then any 'extra' dimensions have to be in some way different from the three that we're familiar with. Well, we have that one 'time-like' dimension - and that's definitely different. The string theorists claim that ever since the big bang, their twenty-odd extra dimensions have been 'curled up' tightly - so (for example) the 4th dimension is such that if you move along it for a short distance, you end up back where you started. (Kinda like flying around the world and coming back to where you started). But that 'short distance' is insanely short - much MUCH shorter than the diameter of an atom. That being the case, anything that was moving or placed 'off to one side' in that extra dimension would look exactly like it was right here in the same place as us along that fourth axis. You can't turn your head by 90 degrees and face along the 4th dimension because there isn't enough room! Extend that out to 20-some extra dimensions and things still look pretty much like a 3D world.
So if you believe the string theorists (and I'm not sure that I do) - then you are ALREADY seeing perfectly well in a 26 dimensional world - and it looks like you think it does!
SteveBaker 14:13, 31 August 2007 (UTC)[reply]
Is it possible that a 4 or higher dimensional (spatial) being could 'project' themselves into 3 dimensional space?87.102.88.202 14:45, 31 August 2007 (UTC)[reply]
That question is meaningless. If there were four (or more) 'unwrapped' space-like dimensions then we'd see in 4D and we'd be 4-dimensional-beings ourselves. You can't have 4-dimensional beings without there being 4-dimensional space...and there isn't, so there isn't. You can't have some things in the universe having a different number of dimensions than others because the 'dimensionality' of the universe is a property of the laws of physics - not the nature of a particular object. However, you can write computer software to 'project' a 4D object onto a 2D computer screen - and (as I said) there are plenty of places on the net where you can see that. It's mostly just confusing - not particularly mind-blowing at all. But check it out for yourself here, for example. SteveBaker 14:59, 31 August 2007 (UTC)[reply]
Those are confusing in part because they are projecting a 4D idea into a 3D space represented on a 2D monitor. An easier projection is the one we actually see: things change over time, though we only see one discrete moment of it at a time. It is easy to describe an ice cube in 4 dimensions. The 3 spatial dimensions are simple — it is a cube, after all — and then 4-D description is just its decay from time t to time t'. What we can't easily do is see the entire "lenght" of the fourth dimension at any single time. We could superimpose all images of the ice cube on top of itself, but that would look like a jumble, in the same way a hypercube does. We are 4-dimensional beings — we age, we move through spacetime. If we were only 3-dimensional beings it would look frozen, static. --24.147.86.187 23:05, 31 August 2007 (UTC)[reply]
Yes - but our eyes ARE 2D image receptors - just like the computer monitor. The question is about what a 4D world would look like to human eyes - so a projection down to 2D is precisely the right answer. Now, you might ask what a 4D world would look like to people with 3D optical recptors in their 4D eyes - and that's something we can't know. Mapping the extra dimension into time (as you advocate) doesn't help at all! If you map a common 3D object into a series of time-wise 2D cross-sections, you can't grasp the shape of the darned thing at all! That technique gives you no idea whatever what a 4D world would look like. So I'll absolutely defend my previous answer - projecting 4D into 2D lets you see EXACTLY what things would look like when viewed with human eyes in a 4D world. This isn't a speculative or approximate thing at all - we can easily simulate the paths of photons in a 4D world and figure out where each one would intercept a human retina...that's an exact thing and it's what the examples I gave are (essentially) doing. SteveBaker 21:04, 1 September 2007 (UTC)[reply]
What I find odd is that there is no good reason why we only have 3 dimensions - 4 or more is possible - as you know. But the universe appears to only have three spatial dimensions - am I missing something obvious as a reason - or is this a good example of an unanswered question in physics/philosophy - "why 3 spatial dimensions" (I release that answers such as "because it is - that's what we see etc" are meaningfull - but I'd like to go beyond 'it is').Your comments appreciated.87.102.88.202 15:19, 31 August 2007 (UTC)[reply]
Why 3 dimensions? Well, it's not clear that there are only three. String theorists postulate many more than that. However, if the number is three then I suggest that it's because if there were fewer, then things like atoms could not exist and we wouldn't be here to observe it. This is an appeal to the anthropic principle - but it's the best explanation we have for that. Perhaps (as string theorists suggest) the unnecessary dimensions 'curled up' and collapsed in the first nanoseconds of the big bang and three is some kind of minimum energy configuration or some such. Asking this is a bit like asking why the charge on the electron is what it is - it has to have some value and this happens to be it. SteveBaker 20:55, 1 September 2007 (UTC)[reply]
An appeal to the anthropic principle works for me, as does common sense! but it's still a mystery on a 'cosmoslogical scale' - but then everything else is as well.. Maybe it's a better question for philosophers than scientists - I think it's an unfair question to ask a scientist as they/we have no evidence any other way (or even the ability to check!)..83.100.249.228 21:11, 1 September 2007 (UTC)[reply]
I suggest you read Flatland, which is both entertaining and educational for thinking about the spacial dimensions. -- JSBillings 16:47, 31 August 2007 (UTC)[reply]
I don't think Flatland is any good at all. It talks very little about the nature of spatial dimensions and gets things wrong MANY times (it's description of a 2D house confuses whether it's a sideways cross-section or a plan-view of a 3D building. It's mostly a painfully sexist diatriabe against the nature of class in our society...nothing enlightening about that at all! There are better books about 2D worlds - and the one that rises head-and-shoulders about the others is The Planiverse. SteveBaker 20:55, 1 September 2007 (UTC)[reply]
Hypercube has been mentioned, but not linked to yet. So here you go. :) And here's a more elaborate visualisation of it. DirkvdM 05:17, 1 September 2007 (UTC)[reply]

Blood Sugar Dropping

Why does blood sugar drop in normal people sometimes and cause them to get dizzy and sometimes faint? I thought when you blood sugar drops and you dont eat, you start breaking down fat. How come the same people dont have the same problem when they exercise? —Preceding unsigned comment added by 76.167.145.55 (talk) 03:13, 31 August 2007 (UTC)[reply]

Please see hypoglycemia, Neuroglycopenia, and maybe even Idiopathic postprandial syndrome. In short, while most of your body can burn other fuel sources, your brain really doesn't like to (I haven't figured this one out yet, but Wikipedia claims it is not fully understood). Someguy1221 03:39, 31 August 2007 (UTC)[reply]
Blood sugars below 80 are usually a signal of some underlying condition. The body's normal response to low blood sugar is to release more sugar into the blood (mainly from the liver I believe). The system that controls storage is usually activated when blood sugar rises above 120. So normal bodies store sugar in the blood when it is more concentrated then 120 and release sugar when the concentration falls below 80. If the sugar level gets low enough, cells start breaking down energy sources other than blood sugar (like the fat you mentioned). However, there is not much fat in and around the brain. There is not much energy storage period above the neck (no protein stores or fat stores). So, blood is the brain's only source of energy. This is why low blood sugar immediately causes symptoms in the brain. Not to mention the brain uses more sugar than than most parts of the body. So when something goes wrong with the release mechanism, the brain suffers because it needs that sugar. Fat, protein, or carbohydrates call all be catabolized into molecules the brain can use for energy. However, this process does not happen in the brain itself.
As for why problems don't happen when a person with chronic hypoglycemia exercises, I'm pretty sure that the major cause of hypoglycemia is related to self-injected insulin. That is a patient who's storage mechanism is broken injects insulin to trigger the storage mechanism and it stores to much (dropping the level below 80). Or, the second most common cause is that a patient's storage mechanism is very sensitive and overrides the release mechanism in certain circumstances. So, a person who normally gets hypoglycemic goes to exercise, their release mechanism works fine and they don't pass out. However, that same person eats only a candy bar and this sends their storage mechanism into overdrive and their blood sugar plummets because even though their release mechanism may start working, their storage mechanism overpowers it. Mrdeath5493 23:10, 31 August 2007 (UTC)[reply]

Gauss's law

Another person seems to think that the field due a point charge Q at distance r (due to coloumb inverse square law) is exactly the same as the field due to a evenly distributed charge of Q on the surface of a sphere (of radius R) when r>R. Exactly the same! Can someone else help persuade them that gauss's law is only an approximation to the field when r>R, (and maybe at the same time show that the approximation becomes better as r/R increase..). (Or maybe you'd like to do the opposite!)213.249.232.202 06:37, 31 August 2007 (UTC) ((Comment added after: I'm assuming here that Gauss's law should be consistent and derivable from Coloumb's law))213.249.232.202 07:29, 31 August 2007 (UTC)[reply]

Have you seen the article Gauss's law? No approximations there. It can be derived exactly from a starting point of an inverse square field strength function. And yes, the electric field outside a spherically symmetric system is identical to that of a point charge located at the origin. (In fact, this can occur even when an inverse square law doesn't apply, such as in general relativity.) Confusing Manifestation 06:56, 31 August 2007 (UTC)[reply]
Have you considered that the charge on the outside of the sphere (relative to the line joining the centre of the sphere and point of measurement) is at an angle to that line - and therefor (due to the circular symmetry about that line) the effective field (due to the charge on that circular band of the sphere) is reduced by a factor of cos(angle between line and points on sphere)? - this means that the true field is always less than the field given by gauss's law (except at infintity). That is the approximation.213.249.232.202 07:19, 31 August 2007 (UTC)[reply]
If you want to prove the equality of the predictions using Coulomb's law, be prepared to write out pages and pages of calculus (you also neglect that some charges are closer or further than the center. A charge being closer than it should has a greater effect on its field at your location than its being further away). But please note that Gauss's law was not derived from Coulomb's law, it was derived from the divergence theorem (proof here (not the original!)). This is pure math, based on arbitrary vector fields. Coulomb's law describes a vector field, so Gauss's law applies. If you seriously doubt this, I truly suggest you pick up an introductory vector calculus book for a more detailed proof (I have been seperated from my own temporarily, or else I would read to you from that). Someguy1221 07:52, 31 August 2007 (UTC)[reply]
Actually, I see that Shell theorem has a nice proof (that Gauss's law and Coulomb's law agree) of this for gravity starting from Newton's law of gravitation (just swap the constants to turn it into coulomb's law). (Oh, and it definately took me a lot more calculus to do my own proof! I think it never occured to me to use law of cosines...) Someguy1221 07:57, 31 August 2007 (UTC)[reply]
Well, well, well I really owe you a debt of gratitude for finding me shell theory. Excellent! I haven't checked it thoroughly but it looks like it simplifies the integration a lot - MANY THANKS! (To you and Newton)
However looking at the integration

There seems to be a problem since

ʃ r2+s2-R2/s2 ds = ʃ (r2-R2/s2 + 1) ds

Which equals [s - (r2-R)2/3s3] and evaluates to:

between r+R and r-R

2R - (r-R)/3(r+R)2 + (r+R)/3(r-R)2 ?

Not giving the same result as in the text, have I made a mistake?213.249.232.202 08:33, 31 August 2007 (UTC)[reply]

Your notation is unclear but:
=
=
=
=
=
=
Which matches the quoted result. Dragons flight 09:00, 31 August 2007 (UTC)[reply]

Ah yes thanks. (made a mistake)213.249.232.202 09:05, 31 August 2007 (UTC)[reply]

Moths.

Kind of similar to the fly question above. At night, moths esspecially more than any other bug are attracted into my room with the light on. They are attracted directly to the light and fly around it and try to get as close to the light source as possible. Do they do the same during the day? Just try to get as close to the sun as possible? Or is the sky luminous enough to turn off this behavior? What exactly is the advantage of seeking out light? Capuchin 07:06, 31 August 2007 (UTC)[reply]

See Moth, especially Moth#Attraction to light. Moths are typically nocturnal, so they don't see the Sun, and they navigate by the Moon, so artificial lights confuse them. Confusing Manifestation 07:10, 31 August 2007 (UTC)[reply]

Quote " It is immediately apparent that for a spherical Gaussian surface of radius r < R the enclosed charge is zero: hence the net flux is zero and the magnitude of the electric field on the Gaussian surface is also 0"

I think this is not quite right - especially in terms of wording

To my thinking it is immediately apparent that the field at r=0 is zero. However if r<>0 but r<R then the field is non-zero - this is not what the article says. Should there be a change of wording or a disclaimer of the variation from reality?213.249.232.202 07:36, 31 August 2007 (UTC)[reply]

Quite right, actually. See above, and see Shell theorem for proof (the same as Coulomb's law, just swap the constants). Someguy1221 07:57, 31 August 2007 (UTC)[reply]
While "immediately apparent" might be a bit generous, the field IS exactly zero everywhere inside of a spherically symmetric distribution. This can be proven mathematically and is the whole point of the shell theorem. 76.225.157.167 08:05, 31 August 2007 (UTC)[reply]
See talk 2 sections above for my questions about shell theorum.213.249.232.202 08:40, 31 August 2007 (UTC)[reply]
The claim is not that it is apparent that the field is zero inside the shell, but that the enclosed charge is zero, which is obvious: all the charge is concentrated on the spherical shell, that's the point. Gauss's law then allows us to reach the nonobvious conclusion that the field is zero. Algebraist 16:20, 31 August 2007 (UTC)[reply]
I couldn't understand the result from gauss's law - but now I've read the proof at shell theorem everything is clear, but thanks anyway..87.102.88.202 16:46, 31 August 2007 (UTC)[reply]

phenomena-defying gravity

I visited a unique place near Nairobi,a town in Athi River area,we were taken to a hill by locals where they poured water on the steep road.Instead of water going down,it went upward.I put our car in netural,instead of going down it went up the hill.It is a phenomena.

Where can I get the explanation.How can I search Wikipedia?

Best regards.

Nizam Khalfan —Preceding unsigned comment added by 202.163.98.109 (talk) 08:45, 31 August 2007 (UTC)[reply]

It's usually explained as being an optical illusion - ie it looks 'up' but that's just due to the lie of the land. Many places like this exist - see Gravity hill for more details.213.249.232.202 08:59, 31 August 2007 (UTC)[reply]
You could add your place to the list now under Gravity hill#Locations.213.249.232.202 09:02, 31 August 2007 (UTC)[reply]

Really Odd Insect

My husband and I drive a truck over the road and have the opportunity to see some really odd and amazing things sometimes. This trip was no different.

While at the shipper this past Tuesday afternoon in West Valley City, Utah, I saw this "thing" crawling around under one of the customers vans. Upon closer examination, this "thing" was an insect of some kind and I would very much like for someone to give me some help on finding out what it was.

It was about 2 1/2 to 3 inches long, had an orange head, black pinchers or jaws (whatever you want to call them), six legs and its body was black and white striped.

I have been all over the internet looking for this thing and while I refused to allow it in the truck with me, I would still like to know what it was and whether or not it was dangerous. (yes we let it live)

Is there anyone who can help me with this and tell me where i can find the information on it and what it is?

Thank you

--Dyanna1 09:05, 31 August 2007 (UTC)[reply]

Did you take a picture we can see (but note, I have no idea, but don't know much about bugs, sorry)? --Cody.Pope 09:57, 31 August 2007 (UTC)[reply]
Did it have wings? Was it similar to anything you've seen before? --Ukdan999 13:13, 31 August 2007 (UTC)[reply]
The folks at whatsthatbug.com know their stuff. A lot of potato beetles have black and white stripes, but not large jaws. --Sean 14:52, 31 August 2007 (UTC)[reply]



No, unfortunately I did not get to the camera to get a picture of it. It resembled an ant, but it only had its head and body. It was not a "three part body" like any other ant, it was only two parts. No wings.

As I said earlier, I have been all over the net and have found nothing in the ant, beetle, or spider families that even come close. Only its body was black and white striped, its head was orange.

--Dyanna1 17:11, 31 August 2007 (UTC)[reply]

It sounds to me like a type of arachnid called a Solpugid (Order Solfugae). It has a number of common names: sun spider, wind scorpion, and camel spider. It has huge, very powerful jaws that can inflict a very painful bite, but is is not venomous. It usually has a reddish or orange head and a body that is not really striped, but is banded. Try an image search on google and see it that's your critter.--Eriastrum 18:50, 1 September 2007 (UTC)[reply]

Black and white spinner

Do we have an article on those discs colored with a black and white pattern and when you spin it you see colors? I just got a similar effect while scrolling through a pdf file. Would like more info on the effect - I can't remember how it works. Also, would the two effects be related, I have a clumsy old CRT monitor on this PC and wouldnt be surprised if the effect I just saw was down to some sort of moiré patterning and difference in response times. Capuchin 10:50, 31 August 2007 (UTC)[reply]

We have an article on Benham's top - is that what you mean? DuncanHill 10:58, 31 August 2007 (UTC)[reply]
That's the one I was thinking of, but maybe a version a little less complicated than the picture here. Fechner color seems to describe exactly what I was seeing with the text. Very strange. Capuchin 11:03, 31 August 2007 (UTC)[reply]
I strongly recommend following the link to an interactive version from the Benham's Top page - more optical illusions than you can shake a stick at! DuncanHill 11:23, 31 August 2007 (UTC)[reply]
That's the one I was thinking of! The one with the sets of three lines! Capuchin 11:25, 31 August 2007 (UTC)[reply]
Hmm, on second thought, playing with that interactive one, the black lines appear colored, whereas the effect I was seeing was far more blurry, more of a kind of faint rainbow behind the text. Capuchin 11:28, 31 August 2007 (UTC)[reply]
Wow that is an amazing site. THanks!Phgao 12:22, 31 August 2007 (UTC)[reply]
I believe that the reason is that the red and blue sensors in your eyes are a bit slower than your green sensors. So when white light changes abruptly to black (or vice-versa), your green sensor recognises the change a little ahead of red/blue. So when the pattern moves but your eyes stay still, the red/blue part of the white bars lags a little behind the green part. Since red+blue=magenta, you get magenta down the edges of the strips moving one way and green down the edges of the other. But we are all unique individuals and for some of us the red sensor is a little faster than the blue - so then you get red+green=yellow in one direction and blue in the other. Yet other people don't see the illusion at all. Probably our brains compensate for this effect in normal vision - so it takes some kind of weird-shaped artificial pattern to make the illusion really work well. Anything that flips from black to white quickly ought to exhibit the same effect - so the spinning black/white disks and all of those other effects are showing the same artifact. SteveBaker 14:50, 31 August 2007 (UTC)[reply]

An excellent analysis of this effect is "Cohen, Jozef and Gordon, D. A. "The Prevost-Fechner-Benham Subjective Colors." Psychological Bulletin, Volume 46, pages 97-136, 1949. The 1960's black and white TV show "Thriller" hosted by Boris Karloff had an episode in which a woman wearing a dress with vertical black and white stripes was dancing, and when she spun around, the sequence of black and white movement caused all of us viewing it to see red streaks- on the black and white set. Quite unintentional but amazing. If it works on B&W TV it should work on a B&W monitor, but seeing color effects on a color monitor is never going to be very impressive. The disc can be found on the web and in textbooks, and can be printed out in a convenient size, like 8 inches, glued to cardboard, then attached to a machine screw with 2 washers and a bolt(after drilling a hole in the center of the disk). Then simply put the bolt in the chick of a variable speed drill and try spinning it at various speeds forward and backward, to get a variety of effects. Edison 00:57, 2 September 2007 (UTC)[reply]

The Membrane Cell for NaOH Production

Hello! I read that in the Membrane Cell there is a "Semipermeable Plastic Membrane" which seperates the solutions of ions and products. It allows positive Na+ ions to migrate through to react with OH- to form NaOH, yet it prevents the OH- and Cl- ions from passing through. ie. It allows +ve charged ions to pass, yet blocks negative charged ions. I am wondering what material this is, how it works and whether wikipedia has any detailed infomation on it. Thanks! —Preceding unsigned comment added by Phgao (talkcontribs) 11:44, 31 August 2007 (UTC)[reply]

It partially depends on the concentration gradient. You might check out active transport versus passive transport, and things like sodium potassium pumps. --slakr(talk) 11:48, 31 August 2007 (UTC)[reply]
Another one: osmosis, which is basically passive transport/diffusion but with movement of only water. Also, if you're interested in an all-encompassing, practical example, you might check out action potentials, as they're how neurons in your brain communicate. --slakr(talk) 11:52, 31 August 2007 (UTC)[reply]
You know, on third thought, I completely misread what you originally posted, LOL. You were asking what the semi-permeable membrane could be made of. A practical example is dialysis tubing, which is, in essence, a semi-permeable membrane used in real-world kidney dialysis. Sorry 'bout that. --slakr(talk) 12:02, 31 August 2007 (UTC)[reply]
No problem, when I first read what you wrote, I was wondering... So these tubings allow +ve ions to pass while blocking -ve ions? Can you explain why that occurs? —Preceding unsigned comment added by Phgao (talkcontribs) 12:05, 31 August 2007 (UTC)[reply]
Well, it's not really that they're normally specific to the ions-- after all, Na+, Ca2+, and OH-, for example, are extremely small and can easily pass through a cell membrane (this is the basis for diffusion). Alternatively, they can use ion channels. This is most likely too complex for the every day person (and/or biology/chemistry student to have to worry about), though. When someone says that an arbitrary membrane is/isn't permeable to a specific ion, it's usually done in the context of a U-tube problem or something similar, since most biological membranes are always permeable to most small ions-- especially elemental ions (like Na+ and Cl-). As far as I know, the overwhelming majority of the time, the only real way for a membrane to not be permeable to an elemental ion would be if the membrane actually reacted with an ion, thus trapping it in the membrane in some way, but that's probably more complex than what you're dealing with.
In any case, the main thing you must grasp is the concept of diffusion, as it applies in all circumstances. Say, for example, a membrane happens to be permeable to sodium ions (Na+) but not to starch-- the duodenum of the small intestine is a perfect example. If the concentration of sodium ions on one side of the membrane (say, the blood stream side) is low, and the membrane is permeable to sodium, sodium will pass through the membrane due to the concentration gradient. That is, there are fewer Na+ on the blood side as opposed to the digestive side, so in order to reach chemical equilibrium, Na+ diffuses to the area of lower Na+ concentration. However, regardless of the starch content of the blood stream, since the membrane isn't permeable to starch, starch cannot simply pass through the intestinal wall-- it needs to be broken down into something smaller. That doesn't mean, however, that starch is completely out of the equation.
Let's instead assume that the membrane were only permeable to water and that everything else was equal. If there's a bunch of starch on one side of the membrane (that is, the starchy side of the membrane has less water in proportion to the all-water side), then water, like sodium, will move across the membrane from the environment of tons of water to the environment with less water-- again, in an attempt to establish equilibrium. This is where terms like hypotonic, hypertonic, and isotonic come in to play. --slakr(talk) 13:40, 31 August 2007 (UTC)[reply]
Thanks for the super info! So it is ion channels which prevent negative ions passing through while allowing positive ions? As these ions would be similar size and the concentration of these would be zero on the other side of the cell, so equilibirum doesnt come into effect here i dont think. Phgao 17:49, 31 August 2007 (UTC)[reply]
Also, I dont mean to say this membrane is specific to these ions, (namely Cl- and Na+), but that since the cell only has Cl-, OH-, Na+, those are the ions in question. Furthermore it is described as some kind of polymer "form of a specially designed plastic sheet that only allows sodium ions to move from one chamber to another i.e. the chloride ions in the anode chamber cannot move into and contaminate the sodium hydrozide produced at the cathode. Furthermore the OH- ions formed at the cathode are prevented from moving to the anode). Phgao 17:52, 31 August 2007 (UTC)[reply]

Blueprints

Is the origin of the term "blueprint" from the fact that photocopiers cannot copy blue colours very well, hence the plans could not be stolen? 82.198.250.8 12:41, 31 August 2007 (UTC)[reply]

This is an encyclopedia. Type blueprint in the search box and click on "Go". -- Kainaw(what?) 12:43, 31 August 2007 (UTC)[reply]
Blueprints predate photocopiers by quite some time, i'm sure :) Capuchin 12:44, 31 August 2007 (UTC)[reply]
Then, later (but still before Xerographic copiers), diazo copiers came into use. While technically producing whiteprints, the output of these copiers was a bluish image on a white background, allowing the continued casual use of the term "blueprint". Ahh, the reek of ammonia!
Atlant 13:01, 31 August 2007 (UTC)[reply]
I am reminded of a bit in the novel A Canticle for Leibowitz. Civilization has collapsed and there is this monk who has somehow come into possession of a blueprint without knowing what it means. He goes to great effort to copy it with a pen, carefully inking in all the blue areas and leaving the white lines and lettering un-inked, because he assumes that the blue is significant and redrawing it the way we would would not produce an equivalent. --Anonymous, 22:28 UTC, August 31, 2007.
Blueprints also came in a version with blue lines on a white background, which would have made life easier for the scribe in the excellent book "A Canticle for Liebowitz." Edison 00:52, 2 September 2007 (UTC)[reply]

eyesight

who is the best eye surgoen in leeds —Preceding unsigned comment added by 82.2.218.57 (talk) 13:11, 31 August 2007 (UTC)[reply]

"Best" in this case is a matter of opinion. What are you basing your standards on? -- Kainaw(what?) 14:31, 31 August 2007 (UTC)[reply]
Is Google your friend? We hardly can advise on this matter. Conscious 18:06, 31 August 2007 (UTC)[reply]
Where? --h2g2bob (talk) 00:45, 1 September 2007 (UTC)[reply]
Oh come on. That's like asking where?? when someone asks about something in London. Unless there's some reason to think otherwise, it's safe to assume he means Leeds. -Elmer Clark 21:19, 2 September 2007 (UTC)[reply]

Cobalt weight vs Nickel weight

By 1891, the order of Nickel and Cobalt were correct in the periodic table, although it appears their masses may have been fudged. Nimur 16:56, 31 August 2007 (UTC)[reply]

Why is Ni, succeeding Co on the periodic table, heavier than the one before it? 81.93.102.185 13:57, 31 August 2007 (UTC)[reply]

The periodic table is ordered by the number of protons in the nucleus. Ni has 28 and Co has 27. However, for the most common isotopes: Ni has only 31 neutrons while Co has 32. Since neutrons are a teeny bit heavier than protons, Co ends up having a slightly higher atomic weight whilst having one less proton. SteveBaker 14:39, 31 August 2007 (UTC)[reply]
Thank you for that speedy reply. Am I to understand that 'most common' means the same as occuring most often in nature, percentagewise? Is there a version of the periodic table which states the most normal isotopes for them? 81.93.102.185 14:46, 31 August 2007 (UTC)[reply]
Probably. But you can see this information on the Wikipedia pages for each element. Go to Nickel and Cobalt and you'll see tables of isotopes for each one. Many isotopes are radioactive - lots of them don't exist at all in nature and have to be manufactured in a nuclear reactor or something. Some of them decay rapidly into the more common isotopes. So, for example, there is Cobalt-56 which has two less neutrons than the naturally occuring Cobalt-58, and there is Nickel-61 which has three more neutrons than Nickel-58. If you happened to pick those two (weird) isotopes instead of the common ones, you'd find that nickel was heavier than cobalt. Since chemical properties are most often related to the number of electrons (which in turn matches the number of protons), it is more useful to have the table of the elements arranged by number of protons than by atomic weight because you get pleasing groupings of elements that behave similarly and predictably from one row to the next. Hence the slightly strange ordering of Nickel and Cobalt. SteveBaker 15:06, 31 August 2007 (UTC)[reply]
(EC) Yes, that's what most common means, and yes, many extended periodic tables will list isotope percentages as part of their data. The excellent online one at www.webelements.com makes it easy to see the percentages (click an element, then "Naturally occurring isotopes" on the left), but it's not all on one page like in some paper ones. That's tough on the web. --Sean 15:10, 31 August 2007 (UTC)[reply]
It may be worth noting that Mendeleev had nickel and cobalt backwards for this reason - his arrangement of the elements was based on mass (since the theory of nuclei was still in development). Nimur 16:46, 31 August 2007 (UTC)[reply]

wbc

is it possible to seperate W.B.C. from the blood & inject into the blood of the needy? how can it be done? where the facility available? --121.247.221.158 14:26, 31 August 2007 (UTC)lalitha.[reply]

Yes, it is possible. Type "white blood cell separation" into Google and you'll find many methods. You're asking for non-destructive methods. As for injecting it back, there is an issue with deterioration of the white blood cells. Is there a facility available? I do not believe so. Where's the profit in storing white blood cells for people? -- Kainaw(what?) 14:31, 31 August 2007 (UTC)[reply]
This would not work, actually. White blood cells are created to target completely random antigens. Each person's own body has methods of filtering immature white blood cells of the ones that target "self" antigens, so that only those white blood cells that target foreign antigens make it into maturity. Mature white blood cells harvested from one individual's blood stream would inevitably target normal, human cells in any other individual's body (except an identical twin). The only way to do this properly is to perform a bone marrow transplant, which transfers the stem cells that create white blood cells (among others). Once transplanted, the transplantee's body can still perform the normal function of filtering out self-targetting white blood cells (usually). Someguy1221 14:44, 31 August 2007 (UTC)[reply]

The E

What does E mean in E=mc^2? —Preceding unsigned comment added by 66.213.124.227 (talk) 15:04, 31 August 2007 (UTC)[reply]

Wikipedia has articles on such things, so need to wait for an answer here: just type "E=mc^2" into the search box, press "Go", and all will be revealed. Or click here: E=mc^2. --Sean 15:13, 31 August 2007 (UTC)[reply]
It stands for 'Energy'. In words, E=Mc2 means: Energy equals Mass multiplied by the square of the Speed Of Light. SteveBaker 15:18, 31 August 2007 (UTC)[reply]
Specifically, it's the rest energy, a particular form of the energy of a particle any object with mass. Other kinds of energy (such as kinetic energy and electric potential energy) have other equations of description. Nimur 16:39, 31 August 2007 (UTC)[reply]

Burning gas indoors

Why is it safe to burn natural gas indoors (as with a gas stove or oven), but not to use a gas grill indoors? Doesn't burning natural gas put off carbon monoxide? Thanks. --Sean 17:55, 31 August 2007 (UTC)[reply]

1) Methane almost always burns "clean" as water and carbon dioxide. Propane can sometimes produce carbon monoxide.

2) A natural gas (methane) leak will shoot gas straight up. If you get lucky, it will wander out of your house very quickly. A liquid petroleum gas (propane) leak will lurk on the floor, so it's harder to get out of the house.

But I should add a third thing -- it's also that gas grills are designed to be used outside, and stoves are designed to be used inside. The technology exists to build indoor propane grills and outdoor methane grills. There's just not a market for them. --M@rēino 19:46, 31 August 2007 (UTC)[reply]

In my previous residence, I had an indoor gas range that used LP (although it could also use natural gas, there was no natural gas supply in my neighborhood). I recall having had to call plumber to do the installation, which partially involved adjusting the incoming gas pressure. So I guess you can use LP inside, but again, it probably has a lot to do with the design of the appliance. --LarryMac | Talk 20:20, 31 August 2007 (UTC)[reply]

I moved into a house once with a natural gas furnace that, after a renovation by the previous owner, was in a small room with an ordinary door and no window. A house inspector told me that this was dangerous and should be corrected at once by putting ventilators in the door. (Yes, I did.) Otherwise the natural gas could burn incompletely and it then would indeed produce carbon monoxide. --Anonymous, 23:18 UTC, August 31, 2007.

I have a gas oven and it always builds up this black oily residue on the floor under and adjacent to it. I had always assumed the natural gas had crap in it. Juanita Hodges 00:34, 1 September 2007 (UTC)[reply]

Juanita - please get your oven checked by a qualified gas-fitter. This is not normal behaviour for a gas cooker. DuncanHill 00:37, 1 September 2007 (UTC)[reply]
I think it's fine. It's extremely slow and I've rented the place for 5 years and nothing's happened. So I'm pretty sure it's safe. I just thought somebody might know what the stuff was. Juanita Hodges 00:45, 1 September 2007 (UTC)[reply]
I'd say that's evaporated grease from the food you cooked in there, which then condenses on the floor. In other words, it has nothing to do with the combustion gas. StuRat 01:14, 1 September 2007 (UTC)[reply]
We're not allowed to answer medical questions because readers could be stupid enough to not realise that this ref desk is no substitute for a doctor. I find that very annoying. But in this case I'm inclined to say that a proper diagnosis of Juanita's situation is not possible with the skimpy info provided. People are notoriously unaware of how dangerous a bad gas installation can be. I wouldn't advise to 'go see a doctor' (a 'gas doctor' in this case), but I certainly wouldn't say everything's fine, based on the info at hand. DirkvdM 05:50, 1 September 2007 (UTC)[reply]
Well, regardless of the source, we can say that the greasy spots should be kept clean, since they're possibly flammable and near an ignition source. StuRat 06:15, 1 September 2007 (UTC)[reply]
I believe the answer to the heart of the question is that Natural Gas has a specific Density that varies between .6 and .7 (since it is not all composed of the same molecule) While Propaine on the other hand has a specific density of 1.52; Im sure that you can easily see the dangers of this if there is even a small leak over enough time inside of a confined space. Expecially if there is a basement, etc...--Aaron hart 21:10, 2 September 2007 (UTC)[reply]

Young snakes

How easy is it to distinguish between the young of an adder and of a grass snake? -- 82.26.220.137 19:08, 31 August 2007 (UTC)[reply]

Seriosuly - read about "adder young" and "grass snake young" using wikipedia or a library or search the internet - then compare the differences between the two.
Before asking a question on the reference desk please do you own research.87.102.87.15 10:43, 1 September 2007 (UTC)[reply]
Please do not WP:BITE persons who ask very reasonable questions on the Ref Desk. Edison 00:48, 2 September 2007 (UTC)[reply]
Then why am I expected to answer it then - why don't you do that,87.102.42.128 08:42, 2 September 2007 (UTC)[reply]
Not everyone spots questions as fast as other people. x42bn6 Talk Mess 09:59, 2 September 2007 (UTC)[reply]

any thoughs were I can source this —Preceding unsigned comment added by Tobiasatkinson (talkcontribs) 19:13, 31 August 2007 (UTC)[reply]

  • To clarify -- I'm pretty sure the question is where one would find sources to confirm the info in the riverstone pebble tiles article. (I wish I knew; I'd do it myself.) --M@rēino 19:48, 31 August 2007 (UTC)[reply]
If that is the case the trade descriptions should do in absense of manufacturer information - if the aim is to prove that they exist and are what the article says what they are. (See previous links)83.100.249.228 15:53, 1 September 2007 (UTC)[reply]
I think this was addressed above - Wikipedia:Reference Desk/Science#Riverstone pebble tiles --LarryMac | Talk 20:24, 31 August 2007 (UTC)[reply]

In an aqueous solution, why is [H][OH] always equal to a constant?

I understand that autoionization of water will result in the same [H] and [OH] because as a hydronium ion forms, a hydroxide ion is left behind. But I don't understand what happens if an acid is added. For example, in my chem book, it shows that if .1 M of HCl is added, then there will be 10-12 of [OH]. That's because [H][OH] is always equal to 10-14. Substituting [H] for .1 would yield [OH] to the aforementioned number. I get how to solve the simple equation, but what I need to understand is what actually happens in the molecular level. From my intuition, adding HCl, thus adding protons, will make [H] a lot higher, but I don't know why the [OH] would become lower so that [H][OH] stays constant.128.163.160.121 19:34, 31 August 2007 (UTC)[reply]

Well, you can look at it this way...Whenever you have hydrogen ions and hydroxide ions floating in solution (I'm ignoring that hydronium bit), there's always a probability of the two binding together and becoming water. And, similarly, there's a chance of water spontaneously splitting into those two ions again. When you dump hydrogen ions into an aqueous solution, you greatly increase the chance of a hydrogen ion and hydroxide ion bumping into eachother and forming a water molecule. The chance of a water molecule spontaneously splitting hasn't changed though. Thus, buy dumping hydrogen ions into the solution, you decrease the concentration of hydroxide ions. The familiar equation to us all, Ka=[X+][Y-]/[XY], is what relates all of these probabilities. I forget how to prove this is the case though, I'm sure someone else around here can explain that, or point to a good article. Someguy1221 20:15, 31 August 2007 (UTC)[reply]
To expand on the above, in a water solution you always have the reaction H+ + OH- <-> H2O going on. This is a fast reaction, so it rapidly reaches chemical equilibrium. Due to thermodynamic principles I don't quite fully understand, when a reaction is at equilibrium, you satisfy the equation given above, e.g. Keq = [H+][OH-]/[H2O], where Keq is the equilibrium constant, (and since the reaction is an acid-base reaction, it also happens to be an acid dissociation constant). Since in aqueous solution the concentration of molecular water is practically constant, and Keq is a constant you can move those to the left side of the equation, giving <constant> = [H+][OH-]. Normally, this constant is denoted Kw, and is called the self ionization constant of water. -- 21:27, 31 August 2007 (UTC) —Preceding unsigned comment added by 72.33.121.200 (talk)
Well, Someguy1221's microkinetic explanation does pretty much explain those "thermodynamic principles". Another way of putting it is that, for H+ + OH- <-> H2O to be in equilibrium, the rates of H2O -> H+ + OH- and H+ + OH- -> H2O must obviously be equal. The first reaction describes the spontaneous breakup of a water molecule, which happens at a constant rate per molecule. Its overall rate therefore depends only on the concentration of water, which, in an aqueous solution, can be treated as a constant. The second reaction, however, involves the collision of a hydroxide ion and a hydrogen ion, an event that happens with a frequency proportional to the respective concentrations of each ion. Thus, when you increase the number of hydrogen ions by adding acid into the solution, that reaction will run faster until the number of hydroxide ions has gone down enough to make the rates equal again. (The actual numerical value of the equilibrium constant Kw, which relates the concentrations needed to achieve this equilibrium, comes from both the rate of spontaneous breakup of water and from the probability of hydroxide and hydrogen ions reacting when they do happen bump into each other.) —Ilmari Karonen (talk) 18:26, 1 September 2007 (UTC)[reply]
The explanations make sense so far, but I don't know why Kw applies to non pure solutions. I thought Kw, the ion product, only applies for pure water. So when HCl or any other substances is added, the solution isn't pure water anymore, thus my confusion as to why Kw applies in the equation. —Preceding unsigned comment added by 128.163.224.199 (talk) 18:59, 2 September 2007 (UTC)[reply]
Fist off H+ is actually H3O+ and [H3O+][OH-]=Kw O.K. now that that is straight, we make the assumption that in an aqueous solution the concentration of water is large compared with the concentration of its ions, thus it is an invarient. This is all you need to be concerned with for now. If you wish to think about the concentrations of a mixture of solutions, whose equations get rather complicated, basically you solve them with a calculator, etc.. But remember the equations are dynamic, for example the chemical ratio formulas on the microscopic level are moving back and forth keeping the ratio. In other words in a complex aqueous solution, no matter what you add to the soulution all the ratios of the ions will remain the same! They stay in eqiuilibrium if you can relate to this you are on your way--Aaron hart 22:25, 2 September 2007 (UTC)[reply]
I forgot to mention with solutions each compound has a Ksp for solubility product constants, there is one for each solution and a K for keeping each compund that ionizes to a constant ratio between the two, these are independent of the other compounds added as far as the ratios go, but it will change the amounts, depending on what you add. The important part is all the ratios of the ions for each chemicals K stays the same regardless of if you have a solution with five compounds, and add a little more of a certain compound the amounts change but all the ratios stay the same.--Aaron hart 22:35, 2 September 2007 (UTC)[reply]

Hydrophobic vs. Hydrophilic

Hydrophilic molecules like water because they're polar. Polar - polar interaction betw. the hydrophilic molecule and water is why they "stick together". But what does it mean when a molecule is hydrophobic? In the wiki page, it says a molecule is h-phobic because it "prefers" non polar molecules. That doesn't sound right to me. Isn't something hydrophobic because it doesn't prefer polar molecules? The wiki page implies that there's an interaction betw non polar molecules. There's no physical attraction between non polar molecules as there are between polar molecules. Am I getting this right?128.163.160.121 19:46, 31 August 2007 (UTC)[reply]

You can have Van der Waals forces between non-polar molecules, but these are weaker than polar-polar interactions. Non-polar molecules can interact with polar ones, but highly polar molecules like water will bind to eachother so strongly as to exclude non-polar molecules except in very low concentrations. Someguy1221 20:09, 31 August 2007 (UTC)[reply]
Right. Each non-polar molecule feels a stronger attraction to the polar molecules than to its own kind, but the polar molecules have such a strong attraction to each other that they form a kind of "clique", and no (or very few) non-polar molecules can break in, even though they'd "like" to. —Keenan Pepper 20:50, 31 August 2007 (UTC)[reply]

It's simple -

Technically a molecule is hydrophobic if the bonding (read 'preference') is stronger with itself than with water - this typically equates to lack of functionality that can hydrogen bond/no charge on the molecule.
The physical attraction between non polar molecules (see Wan der Waals force) is much weaker than that between polar molecules.87.102.87.15 13:14, 1 September 2007 (UTC)[reply]
Effectively you compare the 'preference'/bonding between the molecules you are looking at.87.102.87.15 13:14, 1 September 2007 (UTC)[reply]

If you think about it you will see that "prefers non polar molecules" is the same as "doesn't prefer polar molecules" - molecules are either polar or non polar right?83.100.249.228 15:50, 1 September 2007 (UTC)[reply]

They can also be amphipathic. Someguy1221 22:25, 1 September 2007 (UTC)[reply]

Escape velocity of the solar system for the sun

If the sun wanted to leave the solar system, would its required escape velocity be any less than the escape velocity of a deep space probe due to the difference in mass? --frotht 21:42, 31 August 2007 (UTC)[reply]

I'm not sure I understand the question you're asking. I'm guessing you're asking what speed of acceleration/overall speed the sun would require in order to break the gravitional forces that keep the planets etc in orbit? Exxolon 22:01, 31 August 2007 (UTC)[reply]
The escape velocity doesn't depend on the mass of the escaping object, but the sun would have a much lower escape velocity anyway because it doesn't have to escape from itself. It only has to escape from the rest of the mass of the solar system, which totals something like 0.2% of the sun's mass. It's not as simple as a ratio of because you also have to factor in the varying distance of the mass from the sun. I suppose for a first approximation we can just take Jupiter, for which I get an escape velocity of 570 m/s, about 70 times smaller than the escape velocity from the solar system from earth orbit. (This is wrong; see Anonymous below for the right answer) -- BenRG 22:03, 31 August 2007 (UTC)[reply]
Would the direction of the sun in relation to the Ecliptic make any difference? i.e. if the sun were to be accelerated directly "up" on a Z-axis in relation to the planets orbiting would that change the velocity required as opposed to be accelerated out on the X-axis "through" the planetary orbits? Exxolon 22:30, 31 August 2007 (UTC)[reply]
It would make a slight difference, in that the distance from most of the non-Sun mass of the solar system would increase more quickly going perpendicular to the plane in which most objects orbit. While the gravitational attraction as the Sun approached those objects would help speed it up, it's faster speed than after it passed those objects would mean the drag would have more time to act than the attraction, sort of a reverse slingshot effect. StuRat 22:40, 31 August 2007 (UTC)[reply]

The statement that "the escape velocity doesn't depend on the mass of the escaping object" only works where the primary is massive enough that you can treat it as stationary. Let's consider the solar system as simplified all the way to "Jupiter plus debris". What speed would have to be imparted to Jupiter (mass for it to escape from the Sun (mass )? What speed would have to be imparted to the Sun for it to escape from Jupiter? You can't just apply the formula and say that it's in the first case and in the second, giving two grossly different answers; we're talking about the exact same situation viewed from two different frames of reference. Either the bodies are moving fast enough with respect to each other than they never fall back together, or they aren't.

The correct frame of reference is, I believe, the one where the center of mass of the Sun and Jupiter is stationary — which is almost the same as the one where the Sun is stationary. This means that is the correct answer for either body to escape from the other. In SI units and rounding to 2 significant digits, Wikipedia (at Gravitational constant, Sun, and Jupiter) gives , , and (based on the semi-major axis). From which I compute that the correct escape velocity is about 13,000 m/s or 29,000 mph. The other planets would add to that, but not a whole lot due to their greater distance and/or much, much lower mass, except for Saturn.

The direction makes no difference, so long as it isn't one that would cause a collision. It's purely a matter of kinetic energy. Escape velocity, as it says in the article, would be better referred to as "escape speed".

--Anonymous, waiting now for 00:00 UTC, September 1, 2007.

Yes, you're quite right. Sorry. -- BenRG 00:44, 1 September 2007 (UTC)[reply]
So was I right? The sun doesn't have to escape from its own mass so its escape velocity is much lower than a probe's, right? --frotht 02:20, 1 September 2007 (UTC)[reply]
Wrong -- the situation is symmetrical. You can think of it this way. The Sun doesn't have to "escape from its own mass", but it has to go fast enough to keep its own gravity from dragging everything else after it (which wouldn't be escaping).
But I made a major error too. I said that the other planets wouldn't have much effect because of their relatively small mass, but as I've explained, it's the Sun's mass and the distance that matter. In order for the Sun to "escape" from the entire solar system, it has to escape from the closest body to it, otherwise that body will be dragged along with it. So that's Mercury, whose distance from the Sun varies from to m.
Which means that the relevant escape velocity is anywhere from 44,000 m/s (98,000 mph) to 54,000 m/s (120,000 mph), depending on where in its orbit Mercury is at the time that the Sun makes its giant leap for freedom.
--Anonymous, 04:17 UTC, September 1, 2007.
Is there a rate of acceleration involved here? Or does this calculation assume that the sun jumps from 0 m/s relative to mercury to the 44,000m/s / 54,0000 m/s you've quoted. In laymans terms, how hard does the sun have to accerlerate to snap the "gravitational elastic". If it takes 1 day to reach to 44/54k figure would that be fast enough? Exxolon 16:41, 1 September 2007 (UTC)[reply]
Correct me if I'm wrong anonymous, but the only condition that must be met is that the relative velocity between the sun and mercury has to be ~50km/s. Intuition tells me that basically any acceleration at all will eventually build up that difference, even though "gravity is trying to keep up" by pulling mercury behind it. --frotht 18:00, 1 September 2007 (UTC)[reply]
The formula for escape velocity applies to a direct jump from zero to full speed. If prolonged acceleration is allowed, there is no requirement for any particular speed. As the acceleration continues, the distance will keep increasing, changing the R term in the formula and thus reducing the escape velocity. At some time the increasing actual velocity will overtake the reduced escape velocity and there you are. However, the energy required to accelerate for enough time to achieve escape at low speed is much greater than the energy required to accelerate rapidly to the usual escape velocity. (Oddly enough, this point came up in a comedy movie once: The Mouse on the Moon, which is not based on any sort of serious physics. They were talking about going to the Moon rather than escaping Earth altogether, but the issue is the same. Professor Kokintz in the movie says something like "They have to go fast because they are using rockets. With my device providing unlimited power, we can take our time getting to the Moon." Which is right.) --Anonymous, 11:46 UTC, September 2, 2007.
If we think of a simple two-body problem - a rocket and the sun, say - then the answer is very easy. Does the rocket need more velocity than the sun in order to ultimately put an arbitarily large distance between them? The answer has to be "no" because relativity says that the laws of physics are the same no matter your frame of reference. Whether the rocket considers itself stationary and the sun is moving away at escape velocity - or whether it is moving at escape velocity and the sun is stationary - it's the same thing. But if you try to turn that argument into something about the sun escaping the solar system - then you get into trouble because all of the planets, moons, rocks, asteroids (and Pluto) that make up the solar system already have a lot of relative motion. If the sun started moving off, then some of those objects are going to be heading in the same direction - others in the opposite direction. To escape from all of them requires the sun to have sufficient velocity to escape from all of them. That depends a lot on which direction the sun is moving in - and on complicated interactions between the gravitation of the planets themselves. It's a really complicated (and ultimately meaningless) question. 66.137.234.217 19:43, 1 September 2007 (UTC)[reply]

Is my Sun Conure in pain??

She's going through a major moult at the moment and seems to be really irritable. She's scratching, beating her raggedy wings to get the loose feathers in order and fidgeting all the time. She also gets very angry if I try to stroke her and accidentally touch her new pin feathers. Will she be in actual pain from all this? I've seem how deep the roots on some feathers are and the new ones coming through her skin are sharp. --62.136.214.30 22:48, 31 August 2007 (UTC)[reply]

We cannot give veterinary advice on Wikipedia - please consult your usual vet for advice. I do hope that your pet is back to her normal self soon. DuncanHill 23:24, 31 August 2007 (UTC)[reply]
Perhaps more itchy than pain, just like ingrown hair in people. StuRat 01:08, 1 September 2007 (UTC)[reply]
If you are at all concerned about your bird, take her to the veterinarian, even if only to put your mind at rest. My budgies seem to suffer a lot during the biannual 'big moults'. If I accidentally stroke a blood feather (i.e. a growing pin feather that still has a blood supply), they flinch and squeak (then bite me) - which does lead me to believe that there is some level of discomfort there. --Kurt Shaped Box 13:59, 1 September 2007 (UTC)[reply]

The gloowwww, the wonderful glowwww!

Here's a stumpter for you (I hope), I was fiddling around with the settings on my camera. Hue set to 0, saturation to full, gamma middle, white balance full, contrast full. Everything in the picture, including me, went a rich purple, except my eyes which glowed bright green! On close inspection, it was my contact lenses!! Without a doubt, I can see that that the green disc encompases my iris and goes slightly beyond just like my contacts, and i can move them around. Why do my contacts come up green when they dont even show on the camera normally? SGGH speak! 23:35, 31 August 2007 (UTC)[reply]

come on guys, I look like vincent price with this goatee.... SGGH speak! 23:58, 31 August 2007 (UTC)[reply]
I could tell you, but I'd have to run a few spectrophotometric tests on your lenses. — Kieff | Talk 00:49, 1 September 2007 (UTC)[reply]
Flash on? Flash off? --Reuben 00:50, 1 September 2007 (UTC)[reply]
I'm thinking your camera has some type of automatic red-eye adjustment, whereby it adds green/reduces red to what it determines to be your eyes, in order to compensate for this. If you can find such a setting, turn it off and see if the green eyes go away. StuRat 01:02, 1 September 2007 (UTC)[reply]
Did you try it with your contacts out? —Keenan Pepper 01:48, 1 September 2007 (UTC)[reply]


Can you give us a picture of something (probably not yourself for privacy reasons but a chair maybe and then contacts inside their case) to analyze better with the unusual setting you did? Juanita Hodges 14:54, 1 September 2007 (UTC)[reply]

The original
Oversaturated
Over-saturation tends to pull more or less random colours from greyish areas. For example, these two pictures (original from today's front-page). The smallest hint of colour in the gray parts of the image is enough to throw the math off into generating wild colours that are essentially random. I can explain the math behind it if you care...but it's not very interesting. The answer is "don't do that"! SteveBaker 16:07, 1 September 2007 (UTC)[reply]
That's a great example, Steve. Our reader might also find Saturation, Hue, and HSV color space useful. TenOfAllTrades(talk) 17:40, 1 September 2007 (UTC)[reply]
Was a webcamera, so had no flash or red-eye issues. I suspect the answer involving those nice mountain shots is the best guess. Cheers guys. SGGH speak! 21:44, 1 September 2007 (UTC)[reply]
With a webcam the effect would take effect (...) sooner because overall there is less basic colour info to go by (as is the case in grey areas). But I don't think the effect is random - what little colour info there is is grossly exaggerated for lack of anything else to go by. DirkvdM 09:14, 2 September 2007 (UTC)[reply]
I stand by what I said - the colour you get isn't truly random (I explicitly said "essentially random") - but a one bit difference from an 8-bit RGB value of (128,128,128) (mid grey) to (128,127,128) (mid grey but very, very slightly pinkish) is enough to cause the magenta in those mountain tops of the photo on the right, above. The least-significant bit of the value the camera captures in it's CCD is 'essentially' random - it's just not 100% accurate - nothing in nature ever is. So with enough of a saturation boost, the hue of the colours you get out are indeed essentially random because super-saturation magnifies small errors in measured brightness into violent colour changes. SteveBaker 16:15, 2 September 2007 (UTC)[reply]
Our disagreeing is essentially a difference in interpretation of the word 'essential', but you're the native English speaker, so I'll yield (although not before pointing out you misspelled 'its CCD' :) ). But back to the point itself. If a colour going one way or another depends on one lsb, then 'another quantum fluctuation' (I probably use the wrong words here) may have caused another value for that bit and then it would have been truly random effect. Still not sure what you mean by 'essentially', though. And while we're talking language, you're from Texas but use EE spelling - how come? DirkvdM 18:27, 2 September 2007 (UTC)[reply]
I think SteveBaker is correct, but he's being a stickler for the definition of random. Since it's a photovoltaic process, the bits are subject to noise, etc, etc, etc. Long integration times should yield the true value, which is due to the deterministic details of the source of the photons. This point, I think, is tangential to the original poster's question. Nimur 18:52, 2 September 2007 (UTC)[reply]

September 1

Solubility of Tri n_Butyl Amine in Di Chloro Methane (MDC)

Tri n-Butyl Amine.HCl remains dissolved in xylene at 50 degrees celsius, when used as an acid binder for HCl gas. What is the solubility in non aromatic chlorinated solvents like Di Chloro Methane/methylene di chloride (MDC), 1,2 dichloro ethane (EDC), chloroform etc, below their boiling points? —Preceding unsigned comment added by Mmbk (talkcontribs) 08:28, 1 September 2007 (UTC)[reply]

It will be soluble, but I haven't got a data to say how much, it's possible that the crc handbook (CRC Handbook of Chemistry and Physics) may have data like this (there should be one in a library) otherwise the searcj may be more difficult.87.102.87.15 11:34, 1 September 2007 (UTC)[reply]

Flight in young birds - instinct or acquired skill?

I've been watching the newly-fledged gull chicks flapping around awkwardly (as their parents effortlessly soar) and struggling to fly and maintain balance whilst carrying food in their beaks today. The question was raised in my head - can a young gull instinctively fly to some degree, or does it have to learn everything from scratch by trial and error and observing its parents? Watching chicks in the nest, I see them beating their tiny, downy wings and jumping up and down from a very early age, as though they are trying to grasp the basics ASAP. Different species, I know - but as I understand it, macaws raised by humans in an environment that precludes wing exercising and 'test flights' whilst young will never learn to fly properly. --Kurt Shaped Box 15:03, 1 September 2007 (UTC)[reply]

I'd say a combo of both. Wing flapping is probably instinct, but there's a lot more to flight than just flapping wings, and much of that needs to be learned. By comparison, kittens seem to have the instinct to scratch the ground after they urinate or defecate, but don't actually put this instinct to good use to bury the urine or feces (and thus hide their scent from predators), until taught by momcat. StuRat 16:03, 1 September 2007 (UTC)[reply]
Think of how humans learn to walk. We're obviously "hard wired" for walking bipedally, in the sense that our brain and bodies are arranged so that it is an easy and instictive way of getting around, but that doesn't make it an easy thing to learn how to do from the get-go. It takes a lot of trial and error to get all of those "pre-packaged" routines in sync with our legs and our eyes and our inner ear, and to get comfortable enough with them so that we can do very complex things like walking up and down stairs, dancing, etc. --24.147.86.187 16:07, 1 September 2007 (UTC)[reply]
Thanks, guys. I figured that it may have been something along those lines. It really hit home when I noticed that while the adult gulls were capable of circling (seemingly) effortlessly and barely beating their wings, the youngsters were fluttering in ragged circles and struggling to maintain level flight. I realized that the parent gulls most have an utterly *phenomenal* intuitive understanding of aerodynamics, thrust, wing loading, lift, drag, etc. The sort of subjects that might take a human years to learn, gulls can figure out in a matter of weeks by just getting out there and doing it (the older juvies fly just fine). There *must* be a hard-wired aspect to it. Amazing. --Kurt Shaped Box 19:34, 1 September 2007 (UTC)[reply]
I don't think flight is all that much more complex than other common tasks animals do. Consider people catching a ball. Seems simple enough, but we know the ball would follow a parabolic path in a vacuum but is also affected by air resistance and wind. So, somewhere in our minds we are taking sample points, calculating distance, estimating position and velocity vectors, and changing our location and hand position to catch the ball. If we asked a robot to do it there would be an astounding amount of math involved. But, for us, it's child's play, we don't even have to think about it. It's not all instinct though, it does take practice to learn the proper method. StuRat 02:11, 2 September 2007 (UTC)[reply]
I was just considering the relative risk of flight compared to other activities that animals partake in. You fail to walk, you fall down on your backside - you fail to catch something and you fail to catch something. Get the mechanics of flight wrong and you drop like a stone, with the risk of *severe* injury unless you can recover it in time. Misjudge the landing and you can very easily hit something very hard and unyielding behind your intended destination (think of birds landing on cliff-faces - or the 'pet bird and wall behind the cage' situation), again with the risk of *severe* injury. The consequences of 'failing to fly' can be far greater than the consequences of 'failing to walk'. I suppose that's a strong evolutionary incentive to get it right pretty quickly. :) --Kurt Shaped Box 08:15, 2 September 2007 (UTC)[reply]
I strongly disagree, If you are a newborn Thomson's Gazelle out on the plains of East Africa - you'd better be up and walking within a very few minutes - and running at 80mph with the rest of the herd not long afterwards - or you're cheetah-food. Since half of all baby thomsons die that way, the evolutionary pressure to be able to walk (and indeed run faster than my 1963 Mini!) is pretty much up there with learning to fly. At least birds get the chance to get their eyesight sorted out - learn who their mommy is, get some food in them, flap about a bit in the nest to get a feel for how the air feels in their feathers, watch how other birds fly. The gazelle gets one chance and one chance only - and it's priority #1 immediately it's born. Baby whales have to swim the moment they are born so they can get to the surface to take their first breath - if they don't learn - they drown for sure! Baby birds often survive that first fall from the tree when they mess up flying. SteveBaker 16:05, 2 September 2007 (UTC)[reply]
Fair points there, Steve. Consider me proven wrong. :) WRT to the baby whales - don't their mothers nudge them to the surface after birth to catch their first breaths? --Kurt Shaped Box 05:27, 3 September 2007 (UTC)[reply]

Difference between a transactivator and an transcription factor?

What's the difference between a transactivator and an (upregulating) transcription factor? --Seans Potato Business 18:28, 1 September 2007 (UTC)[reply]

See the basic definitions at Transactivation and transcription factor. The basic difference is that a transcription factor usually descibes an endogenous protein whereas a transactivator is a term that can mean anything that activates genes in trans, including upregulating transcription factors that are both endogenous and exogenous (such as viral in origin). Rockpocket 19:36, 1 September 2007 (UTC)[reply]

Rockhounding

I am new to rockhounding and recently asked someone to identify some rocks for me. He identified one and I wrote down the name as I heard him say it. Then he corrected me saying that "It begins with a p." I wrote down psyomaline or phyomaline. Two questions: 1. Do you know the rock or how can I find out waht the ending of the word means so I can look up other words with that "maline" ending and maybe gain a little insite. 2. The object was black, felt metalic, relativly flat, shiny on top with small bumps, bottom was probably lying in contact with some other object when it was formed as the bottom is totally different from the top. It is relatively smooth. Do you know what kind of mineral it is? Can you be of any help with either question? Thanks, Bill Rockhounder 18:35, 1 September 2007 (UTC)[reply]

ok think I've got it - here's what I did..
First I scratched my head and tried somethings that didn't work..
Then I searched for "minerals list" eg http://ww.google.co.uk/search?q=minerals+list&hl=en
from that I got a list of minerals beginning with p eg http://www.galleries.com/minerals/byname.htm#P
From that I found "PSILOMELANE" which I'm almost certain is your mineral..
This mineral is a transition metal oxide - for similar minerals you could look up 'oxide mineral' - similar minerals might be pyrolusite haematite.83.100.249.228 20:36, 1 September 2007 (UTC)[reply]
Was is the right name??83.100.249.228 20:36, 1 September 2007 (UTC)[reply]

We have a page on it - see Psilomelane also check the links or do an image search for 'psilomelan' as it can take different forms which don't all look exactly the same.83.100.249.228 20:37, 1 September 2007 (UTC)[reply]

By the way 'psilo' means bald as in no hair, or naked, or smooth - because one form looks like a bald head. 'melane' means black/dark as in 'melanin' the skin pigment - this picture will explain why...http://www.mineralatlas.com/mineral%20photos/R/romanechitecp.htm (scroll down 4), however this form http://www.mindat.org/photo-73188.html shows it doing the total opposite - those hairs are the mineral as well, as is the base in this example.83.100.249.228 20:58, 1 September 2007 (UTC) Note these are special examples - mostly it will look a lot more 'normal' for a rock.83.100.249.228 21:00, 1 September 2007 (UTC)[reply]

horsepower

My understanding is that one horsepower is determined by the speed at which an average horse can lift a specified load to a certain height. Is it possible that the same horse can exceed this value when running at a full gallop and carrying a heavier load such that the value determined for each situation is different, producing an error if the value determined for one is applied to the other? Clem 19:45, 1 September 2007 (UTC)[reply]

The value of 1hp is defined as an absolute value unrelated to what horses can actually do.
However the value of 1hp is roughly equal to the amount of work a horse can do.
See Horsepower#Current_Definitions and the article in general.83.100.249.228 20:24, 1 September 2007 (UTC)[reply]
So unfortunately the capabilities of a horse nowadays under standard or other conditions isn't even related to the figure of power that is described by 1hp.83.100.249.228 20:26, 1 September 2007 (UTC)[reply]
Ack, and apparently Candlepower isn't related to actual candles anymore either.. Pfly 01:44, 2 September 2007 (UTC)[reply]
Which is good because not all candles and not all horses are created equal. We're lucky we can specify these things more precisely. ←BenB4 02:09, 2 September 2007 (UTC)[reply]
Most of our systems of measurement come about through similar approximations. Even fairly recent ones like the meter have changed several times over the years. The meter started out as one millionth of the distance from the equator to the north pole...but that wasn't very convenient - so they made a chunk of metal to the exact length they thought represented that distance and kept it very carefully in a vault someplace. However, they were wrong in their measurement of the earth - because they didn't realise that it was not a perfect sphere. So the new metal meter was not one millionth of the distance from the equator to the pole - but it was still the legal definition of what a meter is. Subsequently, we've realised that the exact length of this bar depends on too many other things like how it is supported, what the air pressure is and where precisely you measure the rod if the ends are not perfectly perpendicular to it's long axis...and all sorts of other annoying things. So we redefined the meter as some number of wavelengths of the light produced by some particular substance under some special conditions...but that's a messy description because it depends on too many other things - if you specify the exact temperature at which the object emits this light then the meter ends up depending on the definition of the Kelvin - which in turn is defined by the boiling of water under one atmosphere of pressure - which is some number of kilograms per cubic meter...which means that you don't know how long a meter is until you know how long a meter is! The modern definition is the distance travelled by light in a vacuum in some accurately specified fraction of a second - but sadly, the second also depends on the kelvin - so we aren't much better off! A similar thing happened to the horsepower. The idea that this was the amount of work one horse could do was kinda useful as a rule of thumb when selling steam engines to replace horses! But which horse? Which way of extracting work from it? It was all a bit of a pain. So we define it in terms of other units like the foot and the pound...both of which used to be defined by chunks of metal buried in vaults - but which are nowadays defined in terms of the meter and the kilogram respectively. SteveBaker 15:39, 2 September 2007 (UTC)[reply]
Is the kilogram the only unit still defined by an artifact? —Tamfang 21:03, 2 September 2007 (UTC)[reply]
Yes. The kilogram is defined by an actual lump of platinum-iridium stored in a vault in Sèvres, France. SteveBaker 21:20, 2 September 2007 (UTC)[reply]

Discussing Evolution with a Friend...

I've been chatting with a friend about evolution; he is somewhat in disagreement with it. However, he's come up with an objection that I can't really come up with a way to phrase a counter-argument to, so I'm coming here for help.

He says that, even if variation could develop from evolution, any group of organisms in the same environment (humans, dogs and deer were the e‌xamples he used) would develop into the same organism; that best suited to its environment. Because this is not the observable case, he says, evolution is wrong.

I think that his definition of environment is wrong somehow, and that it doesn't incorporate the idea of an environmental niche for a particular species, but I don't really know how to phrase that idea very effectively.

Any thoughts?

Thanks, Daniel (‽) 21:18, 1 September 2007 (UTC)[reply]

Well you could say that plants are best suited for an earth enviroment (as an example) - but no plant is perfect - there will always be space for a smaller plant that can live in partial shade - so your counter argument of enviromental niches is a good one - and those niches that can cause the diversity we see - for example very small plants and animals and very big ones..Following on you could evolve one species that can eat all the food available but that means that any plant that can render itself inedible will prosper - producing diversity...83.100.249.228 21:42, 1 September 2007 (UTC)[reply]
If I read the statement correctly, your friend is proposing that all species will evolve in parallel into a single "best" species. The problem, I believe, is your friend's apparently naive definition of "best suited." Evolution can be summed up in two very simple concepts: 1) Evolution builds on and alters that which already exists (on the molecular level. New genes don't pop out of nowhere, for example, they just get modified, which can produce new genes only over long time spans); 2) Evolution acts to improve an organisms's chance of having successful offspring. The first is merely the nature of how quickly anything can change, and the latter is sort of proven in reverse; those organisms who are more likely to have successful offspring become more representative of their species over time. Thus, given a group of organisms sharing some environment, they will only evolve in directions that improve their chance of having successful offspring (this also means improving their own chance of survival, for having offspring generally requires that they be alive). So now, I ask you, if the "goal" of evolution is for organisms to make more babies, better babies, make them faster...why would a bunch of different species go on the long and difficult evolutionary path towards becoming the same species? The mutations necessary to do that are incredibly less likely to occur than simpler mutations to accomplish the goals I just gave you. The "niche" part of all this is simply that many organisms can independently be the "best" organism in the environment, the best at surviving in some slightly unique manner. Someguy1221 21:44, 1 September 2007 (UTC)[reply]
(edit conflict) There isn't a single "best" fit for an environment. Take a look at the animals of Africa. Some of them are best served by eating grasslands. Some are best served by hunting. Of those, some are best served by going fast, some are best served by working in packs. All of these are "fit" adaptations, but none of them is the "most fit" approach to survival as a species. When multiple species exist and independentally evolve, they also end up shaping the entire set of species. Thus as the predators get faster, the prey get faster (or develop better defenses, or whatever), and vice versa. At some point an equilibrium of sorts falls into place, so that the prey do not become overly depleted and the predators do not run out of food.
Additionally the "environment" is not a static thing — it also include the other species within the environment. So for the humans, the deer and the dogs are part of the environment; for the dogs, the deer and the humans are part of it. The environment itself is in a state of flux.
(Additionally, dogs have been largely artificially bred by humans, and are a bad case study for evolution in almost any circumstance.) --24.147.86.187 21:47, 1 September 2007 (UTC)[reply]
On the other hand, you could say "Oh, that's correct; and given a few more billion years, all life everywhere will evolve to pretty much the same state, along with everything else. See entropy death." --jpgordon∇∆∇∆ 21:52, 1 September 2007 (UTC)[reply]
Interesting answer. A.Z. 20:00, 2 September 2007 (UTC)[reply]
(edit conflict) It's also important to note that as a species spreads out into new environments, it might find that its own particular niche is not the same anymore (or alternatively, its existing enviroment can change). This might prompt an immediate alteration in which traits are more or less beneficial to the organism, and can result in evolution given enough time. This is simply an example of how not only can several "bests" exist, but what is best changes over time and location. Someguy1221 21:55, 1 September 2007 (UTC)[reply]
Actually, the entropy bit is interesting. Imagine modeling all DNA in the world as a thermodynamic system. It changes randomly, but there will be some preference for DNA of a particular sequence to change in a particular manner. Now, to say that all organisms will eventually evolve into the same organism is asking all the DNA in this model to randomly change until they are all very similar. For this to happen would require some incredible preference, some incredible force directing all DNA, regardless of its current sequence, towards one particular sequence. Because of the existence of niches, and the subsequent lack of a single "best" set of traits, there is no such force, and asking this to happen would be like asking the entropy of a system to kindly go down. Someguy1221 21:59, 1 September 2007 (UTC)[reply]
Your friend's argument doesn't hold for much. He thinks that evolution says that if the environment is the same for all the organisms in it, the organisms would turn into the same species; he has a very poor understanding of evolution. First of all, since the organisms in the environment are different in the first place, it makes it impossible for them to evolve into the "same species" in the future. Selection pressures in evolution works on what organisms already there; the same evolutionary forces may be at work, but since the base organisms are all different, the variations produced will be different.
For example, let's say the environment is the savanna filled with cheetahs. And let's say that the sole selection pressure (what will make some species thrive and others fail) is how fast a species can go (in order to avoid fast predators. In this simplification, faster gazelles would evolve, faster elephants, giraffes, etc. would evolve. Faster organisms will evolve, but they don't become "one" species, or the same organism.199.76.186.8 22:13, 1 September 2007 (UTC)[reply]
Oh, let me add that one of the most amazing observations of evolution is convergent evolution. This is where relatively unrelated species living in a similar environment and pressure would develop similar features. Take whales, which are of course mammals, fish, and ancient marine reptiles such as Ichthyosaur. They all evolved similar features (streamlined bodies, fins placed in similar places, etc.) because they lived in the same environment (water). Note that there are still subtle differences. The tail fin of whales, on one hand, are horizontal, while fishes are vertical. This is a case where two different solutions work for a problem. So in this sense, different organisms do evolve to similar organisms. But your friend's demand that the animals have to evolve to the same exact species is unrealistic.199.76.186.8 22:20, 1 September 2007 (UTC)[reply]
Convergent evolution can be pretty amazing. The main examples that come to mind are the way lemurs came to dominate Madgascar's ecological niches that elsewhere were long controlled by other species (at least until human-introduced species and logging, etc, put the lemur-ecosystems under much stress). Another example is marsupial dominance of Australia's ecosystems. In some cases, as I understand it, the outward appearance of one species may have come to closely resemble that of another, quite unrelated species -- but that due to the very different origins (lemurs vs. marsupials, vs who knows what else) there remain many important differences beyond a cursory outward appearance. The same idea holds for flora -- Australia's tree species are, as I understand it, dominated by Eucalyptus and a few other types. A shrubby "weed tree" around the time of Australia's isolation, the Eucalyptus adapted to a very wide range of niches, spawning a large number of distinct species in the process. As for eventually merging into a single species, this would seem to require something like interbreeding arises between, say, kangaroos and elk, which seems a long shot. Pfly 02:58, 2 September 2007 (UTC)[reply]
There's a South American rodent that looks like a little deer (about as big as a housecat). I would guess that its native region has no real deer. —Tamfang 21:00, 2 September 2007 (UTC)[reply]
One more consideration is that the environment isn't just an empty landscape with static obstacles for an organism to overcome in its quest to survive and reproduce. The environment is filled with other organisms. If all the organisms evolved towards the same ideal, there would be intense competition for the resources. Imagine if every animal in Africa started trying to eat grass. Grass would be hard to come by, but if some species, instead of eating grass, learned to eat fruit or leaves (or even other animals - predation is a hugely successful strategy), they would have all the food they could handle, since there would be no competition. This is what evolutionary niches are. Evolution pushes organisms in two directions - the first is to be more competitive in your present niche, and the second is the find new niches where there will be less competition. If all the organisms started competing for the same resources, any organisms that found -other- survival techniques would be onto a winner. Maelin (Talk | Contribs) 02:12, 2 September 2007 (UTC)[reply]
Another very interesting concept to keep in mind is disruptive selection. This is the case in which two opposing traits are favorable to possess, but anything inbetween is not. Examples are species of Finch in the Galapagos. Some of these species have access to two primary sources of food, both nuts, large and small. Each type of nut can only be easily broken by a particular sized beek, large or small. Medium sized beaks are greatly inferior at opening nuts, and such finches will be deftly outcompeted by those finches that possess the "extreme" sizes. Someguy1221 03:08, 2 September 2007 (UTC)[reply]
I think there are a few good reasons. Evolution most often comes about because of a change of environment. Consider an uninhabited island - onto which animals are swept by clinging to vegetation or something. Two species of animal - one evolved from a hotter drier climate than our island and the other evolved on a cooler, wetter environment are washed onto the island. In order to better adapt to it's new conditions, each has to evolve some. If the animal that came from the cold, wet place has white fur and dark skin with thick subcutaneous fat deposits (like a polar bear), the other creature may have dark fur and a light coloured skin with little stored fat (like a brown bear maybe). Now, will both species evolve into the exact same creature? Probably not - the white animal may be able to adapt to be perfectly at home in the new warmer location by simply shedding some of it's fur. The dark-furred animal may need to grow slightly more fur to keep its lean body warm in this cooler environment. Both changes would require only small genetic modification - and yet we now have two different solutions to keeping the animal at the right temperature. For the two creatures to turn out to be identical, each would have to change fur colour, skin colour and fat-retention strategies - and that's a much bigger change than a simple change to fur density. So you end up with two quite different animals - both at home in the new climate - one bald and dark-skinned, the other hairy and white-skinned - both perfects well adapted.
Another reason is that there might be more than one possible food source on the island which might split a single species into two. So if the island has nuts lying around everywhere and also ample small fish in the rivers, a species of fruit-eating bird coming to the island might be poorly adapted to either diet. One individual might get a random genetic mutation that strengthened it's beak to the point where it could thrive by cracking open the hardest nuts that the other birds couldn't eat - where another individual might randomly develop a long, thin beak to efficiently spear the more agile of the fish (although in the process become less good at eating nuts). At this point, subsequent generations of birds will adapt to the diet they have chosen and sooner or later, you have two totally different species of bird living in the exact same habitat.
If you toss in a small number of initial species and provide a number of different food supplies, then before you know it, you have diversity. You could imagine other kinds of diversity-causeing differences - but this is enough to knock your friend's argument out of the ballpark. SteveBaker 15:19, 2 September 2007 (UTC)[reply]
Wow. Thank a lot, all of you. I gave up on summarizing this and send the friend to read the whole thing himself. So far I think he's impressed. Daniel (‽) 19:17, 2 September 2007 (UTC)[reply]

Hi, I am the friend to whom Daniel is referring and I thought I'd just like to clarify some of my thoughts on the subject.

My point was that all organsms originated from the same place and then, presumably spread out at some stage to different environments where the "evolutionary mechanism" caused various changes to their makeup. However, there are a limited number of inhabitable environments on the planet each with their own (presumably) unique characteristics and so in each area, one form of the organism would prove most able to survive and, over time, emerge as the predominant species.

One cannot object to this by imagining a situation in which several different animals are present without explaining how they came to exist there in the first place!

It seems plausible to me that different species could live harmoniously as above users have asserted. However, I don't feel this accounts for rather large differences in makeup of species which live in the same environment. To put it bluntly: wouldn't every animal develop desirable characteristics such as the comparitively extrodinary brainpower that humans posess?

Another objection I raised was how complex organs such as the eye and aspects of the circulatory system evolved, as it seems to me that randomly occuring mutations "half a heart" or "half an eye" would be of no use at all without other components in te organism!

Thank you for all of your comments and please correct my ignorance as you see fit (I am as you may gather by no means an expert with regards to evolution). —Preceding unsigned comment added by 86.112.251.136 (talk) 19:43, 2 September 2007 (UTC)[reply]

Brainpower is not an absolute good; it requires extra energy and growth for the brain, and in humans it has required the shape of a woman's hips to change so that the larger head can emerge, making a woman's hips slightly less well-adapted for other things. What would a worm do with extra brainpower?
As to the last objection, remember that the randomly occuring mutations are each generally very small. So you wouldn't randomly be born with half an eye, you might randomly be born with a few light-sensitive cells which allow you to gauge light and shadow. The evolution of the eye is quite studied, so I'll leave the article to explain further! Skittle 20:08, 2 September 2007 (UTC)[reply]
And as for "half a heart" – if I remember right we still have partial hearts, in that our arteries have muscles that participate actively in pushing the blood along (veins are passive). The first proto-heart may have been a specialized node in a system of previously unorganized arteries. (Likely it allowed its owner's descendants to grow to unprecedented sizes, and thus claim empty niches.) I don't know of any animals with one- or two-chamber hearts, but don't some reptiles have three-chamber hearts? —Tamfang 21:00, 2 September 2007 (UTC)[reply]


Yep - the 'what use is half an eye' argument is a classic creationist ploy - but it's only hard to understand if you don't bother to read the scientific explanation - which is quite clear and logical. Creationists don't read the science because they don't particularly want to understand it. Evolution of the eye is a great explanation. It doesn't take much imagination to come up with a similar scenario for development of hearts, ears, brains and any other organ you could come up with. But religion doesn't encourage imagination...with predictable results.
But coming back to the "how does diversity come about" issue - OK, I buy that life started in one place, probably in the oceans and with just one 'organism'. It's clear though that there are other environments than just the ocean. So as competition for resources in the ocean grows, there is an evolutionary pressure to change in order to colonise these other environments. So it shouldn't be hard to understand why there was initially enough diversity to provide at least one species per habitat. But as I explained in my 'island' analogy above, a single species in a single environment may well split into two species - each utilising a different resource within that environment. That's how come the same island has birds that are specialised to eat fish, birds that are specialised for fruit, birds who eat insects and birds that eat seeds. Each one needs some kind of specialism to be best at what it does - that also makes it worst at one of the other specialisms. So that lets us have multiple species in one environment - each one exploiting a different resource. Now apply the argument I made about the two kinds of bears who evolved for different environments who meet to exploit a single resource in a single environment. Each has to evolve to suit that new environment - but they may find it easiest to evolve in different ways. So it should come as no surprise that when (say) the African plains have a lot of animal carcasses lying around, both dogs and birds come in to exploit that resource - and you end up with Hyenas and Vultures who are both finely evolved to exploit that resource - but which (because of the evolutionary paths they used to get here) wind up looking totally differently. This just doesn't seem like much of a difficult puzzle to me. It's obvious.
SteveBaker 21:08, 2 September 2007 (UTC)[reply]
If I may just register my objection to the declaration "...religion doesn't encourage imagination..."? Completely off the point, inaccurate, unsupported and a great way of reducing an interesting scientific discussion to 'They say, We say', alienating people rather than informing. This is not about religion vs science, it is about understanding a scientific theory. That some people co-opt these things for their versions of a particular religion does not change this. A little less dogmatism would be nice. Skittle 23:20, 2 September 2007 (UTC)[reply]
On this idea: there are a limited number of inhabitable environments on the planet each with their own (presumably) unique characteristics and so in each area, one form of the organism would prove most able to survive and, over time, emerge as the predominant species. I'd suggest that yes, where there is intense competition between multiple species for the same exact ecological niche one of the other will ultimately win, with the other going extinct. On the other hand, if one or the other competitors develops a slightly different strategy or "way of life", then in effect they are no longer competing for the exact same niche. Since different species naturally have different "ways of life", I might argue that you never find different species competing for the exact same ecological niche. Each brings unique strengths and weaknesses that alter the playing field, so to speak. Then there is the matter of geography and how space separates species, allowing different species to dominate the same "niche" in different places because the competitors are simply not there. In this sense, there is an important difference between "form" and "species", which is essentially what the idea of convergent evolution addresses. The first example I could quickly find was the Marsupial mole and the "regular" mole. There are numerous similar examples. The point is that similar environments (like, say, the environment of burrowing mammals) will naturally favor certain "forms" (strong digging hands, poor eyesight, good smell, etc), but this doesn't not necessarily mean that marsupial moles and "regular" moles are remotely similar, species-wise, even if they take similar forms. The two forms of "moles" (and there are far more than two such forms) can exist simultaneously because they are geographically isolated. If all critters were given equal access to the entire planet, then we may well see a large number competing for the same environmental niches. No doubt many would go extinct. These days, with humans transporting animals all over the place, something like this may well come to pass. Pfly 06:42, 3 September 2007 (UTC)[reply]

Why do 'flammable' and 'inflammable' mean the same thing?

...and why are both terms still in use interchangably? Isn't this really dangerous? I've heard of people being killed after thinking that inflammable meant 'not flammable'. —Preceding unsigned comment added by 84.68.243.102 (talk) 22:36, 1 September 2007 (UTC)[reply]

Inflammable comes from the word "inflame" which means "to burst into flame." Flammable, obviously, comes from flame. It's merely curious in that "in" can mean "not" as a prefix. Someguy1221 22:45, 1 September 2007 (UTC)[reply]
No one said the English language made sense! -- 68.156.149.62 02:02, 2 September 2007 (UTC)[reply]
The correct antonym for "inflammable" and "flammable" is "non-flammable". Maelin (Talk | Contribs) 02:04, 2 September 2007 (UTC)[reply]
Why don't you suggest this to IUPAC or whomever you think is the most suitable authority regarding to this matter? You may become famous;) --Mayfare 03:16, 2 September 2007 (UTC)[reply]
Given that most authorities already recommend against use of the word inflammable for that very reason it's somewhat unlikely Nil Einne 15:20, 2 September 2007 (UTC)[reply]
I placed a winking face for a reason. --Mayfare 22:40, 2 September 2007 (UTC)[reply]

Of the two synonyms "flammable" and "inflammable", at one time only "inflammable" was widely used. In the 20th century people involved with fire insurance promoted the use of "flammable" (and similarly, "non-flammable" rather than "non-inflammable") precisely because "inflammable" could be misinterpreted. While there is apparently no hard evidence to confirm it, the timing and circumstances are right for Benjamin Whorf to have been the man behind this. --Anonymous, 11:58 UTC, September 2, 2007.

The short answer is that they mean the same thing. The world is trying hard to forget that the word 'inflammable' exists - preferring to use 'flammable' and it's antonym 'non-flammable'. However, I was always taught (UK English) that the two words didn't mean precisely the same thing - they were not opposites - but did have subtly different meanings. Inflammable comes from a Latin word: "inflammare" meaning "to kindle". Flammable comes from a different Latin word, "flammare," meaning "to set on fire". As I was taught, an inflammable substance was capable of spontaneously bursting into flames where as an flammable substance would burn if you set light to it - but would not spontaneously catch fire. I don't think many people make that distinction nowadays - and as a society, we should probably simply try to forget that 'inflammable' exists as a word and use 'flammable' for both cases. SteveBaker 14:53, 2 September 2007 (UTC)[reply]

(I just though of another pair of words like that: Imprint and print also mean the same thing - unlike impossible and possible!) SteveBaker 14:56, 2 September 2007 (UTC)[reply]
Moreover, that is why English have two different words. No two different words have the exact same definition. Otherwise, why would English have those two different words that mean the same thing? It would be redundant, adding pages to dictionaries, thesauruses, and so on and so forth. (A little bit off the topic - That is also why word choice is so important!) --Mayfare 16:52, 2 September 2007 (UTC)[reply]

Latin has two prefixes in- and īn-, normally written alike. The first, with a short vowel, comes from the ordinary preposition in; in some Romance languages it becomes en-. The second, with a long vowel, is cognate to English un-. The word inflammable has the first prefix; compare enflame. HTH. —Tamfang 20:44, 2 September 2007 (UTC)[reply]

Hello. Do epicycles on epicycles look similar to fractals? Thanks in advance. --Mayfare 03:07, 2 September 2007 (UTC)[reply]

Depends on you definition of 'looks like' - a bit maybe - they look like the patterns from a Spirograph more than anything I can thing of..87.102.42.128 08:54, 2 September 2007 (UTC)[reply]
An epicycle traced around an epicycle is the kind of drawing you get when you consider something like the position of the Apollo command module relative to the sun when Apollo orbited the moon while the moon orbits the earth and the earth orbits the sun. This isn't a fractal...but it shares some of it's attributes. An epicycle on an epicycle exhibits some 'self similarity'. The motion of Apollo around the moon is similar to the motion of the moon around the earth - so there is certainly self-similarity. But fractals have INFINITE self-similarity - so you'd need to trace an astronaut riding a jet-pack in a circle round Apollo and the astronaut's cat being swung around the astronaut's head on a piece of string and a flea on the back of the cat....etc. So IF you drew epicycles on epicycles on epicycles to an infinite degree - then what you would have would be a true, honest-to-goodness mathematically correct fractal. But if you stop short of that then you don't have a fractal. The reason a finite number of epicycles-on-epicycles LOOKS like a fractal is because when we draw fractals we are lazy and we stop drawing them after a finite number of iterations. So, if you look at our article on the Koch snowflake you'll see that we only drew the snowflake to a 'depth' of seven iterations. Drawing more iterations than that would have better approximated the true fractal - but the resolution of our computer screens isn't enough to resolve more than about seven iterations - so there isn't a lot of point going deeper. The same thing would be true for epicycles - there would come a point where a finite number of epicycles would be 'enough' to make it look like some kind of epicyclic fractal. SteveBaker 14:32, 2 September 2007 (UTC)[reply]

So moreover, may I say that Spirographs look like fractals only to a certain extent? --Mayfare 16:44, 2 September 2007 (UTC)[reply]

No! A Spirograph pattern is just one epicycle. You'd need to drag a little wheel along the line of a spirograph pattern to make a 'second order' epicycle - but you'd need a lot more than just second order - maybe sixth or seventh order might be 'close enough' to look like a fractal. SteveBaker 20:48, 2 September 2007 (UTC)[reply]

General Principles of Relativity Cant be Applied to light, Why ?

If a person is travelling with a velocity v opposite to the light, the relative velocity of light w.r.t the person must be (v+c) according to the Galliliean and Newtonian Principles of Motion Relativity. But Why doesnt it hold good? In special relativity it sas that the speed of light is constant w.r.t all frame of reference.--Oasa 04:18, 2 September 2007 (UTC)[reply]

Why? Because the universe doesn't work the way Newton expected at high speeds. Take any two speeds v1 and v2 (relative to you at rest), and general relativity says that the speed of one relative to the other must still be less than or equal to the speed of light. Specifically, the net speed will be (v1+v2)/(1+v1*v2/c^2)^(1/2) 76.225.157.167 05:00, 2 September 2007 (UTC)[reply]
It's actually special relativity that says that. Someguy1221 05:06, 2 September 2007 (UTC)[reply]

I am Sorry I Didnt get you to the full extent.Oasa 05:23, 2 September 2007 (UTC)[reply]


Could someone direct me to a specific article which explains how the speed of light is supposed to be constant relative to all frames of reference? From what I understand, the speed of light is different in a void and has been slowed considerably in controlled experiments.
This questions seems similar to "What happens if you're in a car going the speed of light and you turn on the headlights?" The Solar System is already traveling at extremely high velocities from certain frames of reference, so I don't quite understand why two objects already traveling at the speed of light in opposite directions shouldn't have their velocities added to one another just because special relativity says it shouldn't. The whole point is that it's all relative. We don't, and may never, have the ability to perform experiments to prove it one way or the other.
I've never been fond of the theory of relativity as a whole, but I'm not fond of willful ignorance either. Could someone please tell me where to find where it is proven that "the speed of light is constant w.r.t all frame of reference?" Thanks. --Demonesque 05:37, 2 September 2007 (UTC)[reply]

Thank you Demonesque. Thats what I actually what I wanted to know. The proof and why we must not add simply their velocities. Oasa 05:53, 2 September 2007 (UTC)[reply]


Not a problem, your question made me think, which I enjoy. And I have the supremely annoying habit of questioning established fact. --Demonesque 05:54, 2 September 2007 (UTC)[reply]
Relativity does not prove that the speed of light is constant. Rather, Einstein assumed that the speed of light measured to the same value no matter how you try to measure it (that's the most accurate way of putting it, constant in all reference frames is not fully explanatory of his assumption). Then, Einstein took this assumption to various conclusions, consequences of special relativity. These consequences (especially time dilation) can be experimentally shown to exist, and are accurately predicted by Einstein's theory. This is taken as proof by physicists that Einstein's underlying assumptions are accurate as well. As for relativistic velocity addition itself, it is a mathematically necessary consequence of Einstein's assumptions. A proof is provided in this paper: Mermin, N D, 1984, Relativity without light American Journal of Physics 52.2 119–24, a very good read if you have access to it. I would spell out the important steps of the derivation, but I don't have access to that journal right now. Actually, darnit, I also have a book containing the proof, but I seem to have left it far away from myself...Someguy1221 06:14, 2 September 2007 (UTC)[reply]
That is helpful, thanks. Ironically it is the mentioned time dilation which has always made me question his theories, as the idea of time as a dimension rather than a concept used for sequencing events has never sat well with me. His accurate predictions is what made him pretty much the only living scientific superstar. However, I sadly can't bring myself to accept a lot of his assumptions, which is why I was hoping for a proof, since I'm certainly accepting of a convincing argument; and when it comes to arguments, it doesn't get much better than empirical evidence. --Demonesque 06:32, 2 September 2007 (UTC)[reply]
Time dilation is "easily" observable (if you have a billion dollar particle accelerator). When atom smashers make new particles, many of them are unstable and decay with a characteristic half-life. However, the rate of decay is determined by the passage of time in the rest frame of the particle. From the point of view of an external observer, the time requires to decay gets longer and longer as the speed of the particles increases with exactly the proportion expected if the half-life is increased via the predicted time dilation. 76.225.157.167 06:55, 2 September 2007 (UTC)[reply]
Yeah, I'm aware of the use of Muons being used to measure time dilation, as well as high speed flights using atomic clocks, however I don't consider that evidence as conclusive proof of time dilation. Both logically and emotionally I cannot accept the possibility of time dilation as I believe it would necessarily enable time travel. However, I don't want to hijack this discussion towards a topic I've already spent a lot of time discussing pointlessly. --Demonesque 06:58, 2 September 2007 (UTC)[reply]
You've been asking for proof, but it sounds like what you really want is to understand the theory. These are totally separate things. It's pretty clear you don't understand it right now, and this is causing you a lot of confusion. I don't think Wikipedia articles are a good place to learn relativity at the moment, unfortunately. One online tutorial that I like somewhat is Ned Wright's. Let me know if it helps at all. -- BenRG 11:44, 2 September 2007 (UTC)[reply]
There are all sorts of experiments that show that the speed of light is a constant. The simplest to understand is the one where you measure the speed of light coming from a distant star in January - then again (using the same star) in July. If the speed of light worked in a 'Newtonian' fashion then we would find that the two results would differ by twice the speed of the earth in it's orbit because in one measurement we'd be moving towards the star and in the other, away from it. However, when you do that experiment, you get EXACTLY the same answer for the speed of light in both cases - thereby proving that the speed of light in vacuum is a constant no matter what frame of reference you are in. This is an experimental fact - quite separate from any theory - and it conclusively proves that Newtonian mechanics is wrong. So - now you have to come up with something else - and the only theory we have that fits all of the experimental evidence is Relativity. Now as to why the universe follows Einstein and not Newton...well, that's probably unanswerable. The speed of light is what it is - just as the charge on the electron, the universal gravitational constant and all of those things are constants. The fact that it imposes a cosmic speed limit and causes all of those inconvenient relativistic effects is also "how nature works" - there isn't always an answer to the 'why?' question for such fundamental things. Some things just are. SteveBaker 14:20, 2 September 2007 (UTC)[reply]
It should be noted that Einstein never came up with his assumptions on his own. If that were the case nobody would have sided with him from the beginning. Rather, he took assumptions that others had already proved or seemed reasonable — i.e., that Galilean relativity should apply to all physical activity, and that the speed of light was constant in a vacuum — and showed that if both were true, strange results would fall out — time dilation is a perfectly logical and natural result of those two facts being true, though it took Einstein's elaboration (which is really quite readable) to make that clear to people, and it took experimental consequences for people to realize that it wasn't just a logical trick that Einstein had pulled.
So how did 19th century physicists know the speed of light was constant? Well, let us reformulate that in the way that they thought of it: the speed of light does not depend on the velocity of its emitter. That's a little bit different, and a lot easier to grasp, if we are thinking of light as a wave and not as a particle (forget photons for a minute, they came later and are conceptually very tricky). The speed of any wave does not depend on the speed of its emitter — if it did, you would never have the doppler effect. So it was perfectly obvious, and nothing interesting at all, to a 19th-century physicist to think that the speed of light was independent of the speed of its emitter. It becomes trickier today because we think of light in terms of photons and often exclusively think of them as particulate, but we should remember that it does have legitimate wave properties and in fact most of the time acts something like a wave. --24.147.86.187 16:06, 2 September 2007 (UTC)[reply]
The trouble for a 19th century physicist was that in order for the speed of light to be independent of the speed of the emitter (as for example is the case with sound waves), there has to be a medium through which the sound is travelling (analogous to the air through which sound waves travel) - and the speed of the listener relative to that medium ought to be what matters. Hence the theory of the Luminiferous aether. The problem being that the experiment where you measure the speed of light from a star six months apart (or better still, the Michelson–Morley experiment) blows away that theory. Without the aether being present for the speed of the emitter to be measured relative to...the old theories kinda fell apart at the seams. Hence the need for relativity. The very name chosen for that theory ('relativity') expresses that the key point made by the theory back then was the lack of an absolute frame of reference (ie no more aether). Nowadays, most people have no problem believing that part - the tricky part for us is all of the time dilation effects. I suspect that, had the theory had only just been discovered, we'd call it something quite different. SteveBaker 16:52, 2 September 2007 (UTC)[reply]
Agreed 100%, except that it is sometimes worth going back to the aether-like explanations to make things like "the speed of light is constant in a vacuum" intuitive. Unfortunately these days we start by talking about photons and laugh off the aether without really acknowledging why it was seen as so important at the time, and why it was so conceptually difficult to get rid of it, and why things like "the speed of light is constant in a vacuum" feel so odd and unintuitive. --24.147.86.187 23:34, 2 September 2007 (UTC)[reply]
That bit about the speed of light being identical regardless of which direction the Earth is traveling is very intriguing, SteveBaker. That does seem to prove that the speed of light is constant in a vacuum. Does anyone have an idea where I can find more information on that experiment (or any others you might know if, as well)?
Additionally, what methods are used to measure the speed of light? I've never thought about that before this. --Demonesque 22:17, 2 September 2007 (UTC)[reply]
The experiment in question is the famous Michelson-Morley experiment, which measures the difference of the speed of light between two beams sent at right angles from one another, and finds no difference (no matter how you rotate it; if light was going faster along with the direction of the earth, you should find some difference in some direction). As for measuring the speed of light, may I direct you to Speed_of_light#Measurement_of_the_speed_of_light? --24.147.86.187 23:34, 2 September 2007 (UTC)[reply]


Thank You !!! Oasa 03:26, 3 September 2007 (UTC)[reply]

A good reference would be Einsteins General and Special Theory of Relitivity. I belive one of you inhibitations to understanding time dialation is the understanding of Space-Time. Four dementions that inherently we only percieve as three (being space) and time. After understanding the Theory you will note the higher the velocity of an object through space, the slower it travels trough the fourth demention being time, and vice verca. Thus the frames of refrence have independent times and thus view velocities accordingly.--Aaron hart 09:33, 3 September 2007 (UTC)[reply]

September 2

How quickly does a fixed point in space move?

How quickly does a fixed point in space move, relative to an outside observer? I have done some research on wiki, but linking the information together is where I need help. For example, according to the Sun article, the solar system is orbiting galactic center at 217 km/sec. The Milky Way itself is moving at 552 km/sec. Even the universe itself is expanding, although whether a fixed point is affected by this, as per Metric expansion of space, is something I'm not sure of, nor do I know what speed to apply to this. In any case, how do these speeds relate to one another? Is our sun, for example, revolving back toward where the Milky Way was, or is it moving along with the galaxy at an effective combined speed of both? And how does the expansion of space affect this?

To put it another way, if I had a particle with no velocity that was not affected by gravity, how long would it take it to traverse the solar system, and how fast - relative to an outside observer on Earth - would one say it was moving? And in what direction (i.e. solar system north to south, across the plane, etc.)? Thanks for your help in advance. Newsboy85 05:45, 2 September 2007 (UTC)[reply]


The Milky Way is moving relative to what at 552 km/sec? And what is a fixed point? Relativity is sadly very relevant when it comes to discussing things like this. The entire universe is in constant motion, from tiny particles to galaxy clusters. I would guess that the velocity of your fixed point would depend entirely upon what it was moving in relation to and what you defined as a point. --Demonesque 05:52, 2 September 2007 (UTC)[reply]
Due to the complete absence of any preferred reference frame, "fixed points" only exist relative to locatable objects. However, if you want to know how quickly two particles fixed relative to eachother will move, and affected by only the universe's expansion, this paper suggests they'll move apart at a rate of 500 km/s for every million parsecs between them. Someguy1221 06:01, 2 September 2007 (UTC)[reply]

To answer the question about what the Milky Way is moving relative to, as quoted from Milky Way: "Another reference frame is provided by the Cosmic microwave background (CMB). The Milky Way is moving at around 552 km/s[40] with respect to the photons of the CMB. This can be observed by satellites such as COBE and WMAP as a dipole contribution to the CMB, as photons in equilibrium at the CMB frame get blue-shifted in the direction of the motion and red-shifted in the opposite direction." I realize that my question is somewhat moot, but I'm just trying to get a better grasp on the concept. I will have to take a look at that paper, however. In any case, if we take the bothersome expansion of the universe out of the equation, how is the solar system moving in relation to the galaxy? Newsboy85 06:17, 2 September 2007 (UTC)[reply]

Is the article on the Local_Bubble, which the Sol System is passing through currently, of any use to you? I was reading it a few days ago and thought it might be relevant to your interests. --Demonesque 06:26, 2 September 2007 (UTC)[reply]
Note that although ~500 km/s seems like a high speed when compared to the scale of the solar system (it is almost 20 times the average orbital speed of the Earth relative to the Sun, for example), it is a tiny speed on cosmic scales. The relative motion of distant galaxies due to the metric expansion of space can be 100 times as fast - on a cosmic scale, the expansion of space itself dominates any local motion relative to nearby galaxies. Gandalf61 11:05, 2 September 2007 (UTC)[reply]

Insane crossword question driving me crazy

56 Across: Type of star system: Safely get X-Ray (7,6)

You can find it in the Times, Saturday September 1st 2007. The second word is most likely "galaxy" since it's 6 letters and all the letters are there. This leaves the remaining letters to be SFETYRE, so it's an anagram of those letters if my guess of "galaxy" is right. Anyone? —Preceding unsigned comment added by 86.144.233.185 (talk) 12:25, 2 September 2007 (UTC)[reply]

It's Seyfert galaxy, which incidentally do emit high levels of X-ray radiation. Laïka 13:50, 2 September 2007 (UTC)[reply]
Wot, no anagram indicator? Bad show! --Anon, 05:35 UTC, September 3, 2007.

Drink mixing

A common piece of advice to young people seems to be "never mix drinks!". Is there any scientific basis for this, and if so, why are cocktails apparently an exception? Laïka 13:48, 2 September 2007 (UTC)[reply]

The French always advise against mixing grain with grape - so they'd say it's OK to mix beer with (say) scotch - but not with wine. I'm sceptical too - but see if anyone can come up with anything. SteveBaker 14:02, 2 September 2007 (UTC)[reply]
Yes, it's an interesting question. I know that I'm always 'bad off of it' if I mix lager and cider (be it in a snakebite or by alternating pints. I don't know why - and TBH, I've never stopped to consider why. I'd quite like to know why now... :) --Kurt Shaped Box 14:09, 2 September 2007 (UTC)[reply]
Alcohol seems to be one of those topics about which there are many legends, for whatever reasons. In simple terms most drinks are roughly equivalent in terms of alcohol. When I was in college people were always full of advice like "beer before liquor; never sicker". I suspect most of this is just old wives tales. Sometimes a person may have a bad experience in one particular situation, so then they assume they know exactly what caused it, usually erroneously. That said, the chemistry of fermentation is vastly complex, and how a given drink may affect a particular person is very complicated also, so it's possible there's something to some of these stories. But in general, I consider them mostly BS. Friday (talk) 16:10, 2 September 2007 (UTC)[reply]
I can't personally attest to the veracity of the following, but I have heard from multiple sources (including career chemists) that congeners are the basis for this often-repeated observation. Congeners, the various impurities found in alcoholic beverages, may be one of many factors that lead to hangovers. It is thought by some that these chemicals may compete with each other and alcohol for the various detoxifying enzymes in the body, which in general causes these congeners to remain in the body for longer periods of time (and thus do more damage, if they're into that sort of thing). It stands to reason that as one mixes more congeners in the body, more competition is likely to take place for these detoxifying enzymes. Since certain classes of alcohol are likely to have similar congeners, it stands to reason that mixing alcohol classes (and thus mixing congeners) may result in a worse hangover. Tuckerekcut 16:38, 2 September 2007 (UTC)[reply]
That sounds fairly hypothetical - I'm calling WP:OR! Any sources for this? Nimur 17:34, 2 September 2007 (UTC)[reply]
Note that Friday's "most drinks are roughly equivalent in terms of alcohol" refers to those drinks poured into the official glasses. Of course this excludes pints of beer. And most Belgian beers and such are stronger than pilsener, yet usually poured in bigger glasses. And since wine is stronger still, it should be drunk from really small glasses, which I don't recall having ever seen anyone do. On top of that, different Belgian beers and wines have different alcohol contents, so there should be a separate glasses for each (which there aren't). Considering there are so many exceptions, one can safely say it's a load of bull. :) DirkvdM 18:52, 2 September 2007 (UTC)[reply]
Definitely this is true; however I was assuming someone who was drinking "advanced" beers is knowledgeable about them and thus knows what to expect. In terms of the beers that people who know nothing about beers tend to drink, it's safer to say they're roughly equivalent. If you're drinking a 9-10% beer, just count it as two "normal" beers. Friday (talk) 19:04, 2 September 2007 (UTC)[reply]
Here in the Netherlands, Belgian beers are drunk quite often by people who are not really 'into that', and quite often they are surprised at how quickly they get drunk (although the hangover is often lighter than one would expect). But the biggest problem here is wine. A typical wine glass is about as big as a beer glass, even though it is at least twice as strong. Champagne (or whatever passes for it) is notorious for getting people totally pissed. DirkvdM 09:24, 3 September 2007 (UTC)[reply]
Regarding the "beer before liquor, never sicker", the companioin couplet is "liquor before beer, you're in the clear", so while that might be old-wives'-ish, it's not on the same topic as the original question. The issue there is one of ordering, and I've heard it rationalized that one would get into a pattern of drinking during the first few drinks and not notice or be unable to adjust the pace appropriately if the drinks become stronger later in the night. the extreme "beer before liquor" is switching up to drinking straight whiskey on a pace with one's earlier beer-drinking—that's a lot more alcohol a lot faster than you were expecting. DMacks 19:38, 2 September 2007 (UTC)[reply]
It might also have something to do with dehydration. Alcohol dehydrates you (that's the major cause of hangovers, I gather), and perhaps being drunk impairs your ability to notice that, in which case you're better off if the last thing you drink has a higher ratio of water to alcohol. This is my theory which belongs to me.Tamfang 20:25, 2 September 2007 (UTC)[reply]
This question reminds me of Wikipedians editing while being drunk. It is too bad that I forgot the URL. Cheers! --Mayfare 22:55, 2 September 2007 (UTC)[reply]
My opinion is is not the mixture thats the problem it is the sheer quantity of alcohol involved. Six beers sounds a lot, while a couple of beers, two glasses of red wine, followed by a brandy and a port does not sound like so much. It may be time for an alcohol free week. Graeme Bartlett 00:00, 3 September 2007 (UTC)[reply]
Indeed. It sounds like there's a false perception of consuming less total alcohol because it's spread around differently. Nimur 02:49, 3 September 2007 (UTC)[reply]
I just want to through in my two cents, although at this hour its probably not worth much; but vodka always gave me a hangover, I latter learned that it containes a certain percentage of wood alcohol. Also the amount of alcohol consumed does not have a linear relation to B.A.C., This is due to dillution and, thus absorbtion. Ex if one takes four shots of 151 on an empty stomach in one hour, The B.A.C. Will Spike to a much higher level, and then level off, than if one chugged down the equivalent amount of Alcohol in Budwiser durring the same hour. To be correct I'ts not linnear, but you get the pint.

Timber shipping Markings

How do I find out all the new Timber shipping markings?

I have a book with them in dated 1975 but can't find any updated version Can anyone help me please

Thank tou Julie Simpson —Preceding unsigned comment added by 86.152.210.170 (talk) 15:00, 2 September 2007 (UTC)[reply]

Are you talking about Waterline markings to determine the load on a ship? Or, do you mean a standardized marking for the transport of lumber [21]? Nimur 17:35, 2 September 2007 (UTC)[reply]

Food Science Term

I'm trying to remember a term from food science with no luck. It means "the feeling/texture of food in the mouth" or something along those lines. It's a big component of how people respond to food, other than just pure taste. Does anyone have any idea what it might be or where I could ask an expert? Thank you.

That would be Mouth feel. DuncanHill 18:12, 2 September 2007 (UTC)[reply]

Interest in fuels from tree sap or resin

It occured to me that, scince tree sap has sugars and resin is a hydrocarbon, perhaps someone is developing fuels from them. I tried looking but as I know little of chemistry I don't know the proper words to search for. My curiosity has grown to a monsterous mental itch. I would be very thankful for anyone who can point me in the right direction.

JimKenneth 17:56, 2 September 2007 (UTC)[reply]

Ethanol is made by fermentation of sugars and can be burned in car engines - so yeah - sure, you could make it from tree sap. The problem is the amount of sap you'd need in order to make a dent in gasoline needs. I doubt you could collect enough to make it worthwhile to make an enormous chemical plant to do the work. But if you have reason to believe that a VAST quantity of this stuff is available on the cheap - then maybe you're right. SteveBaker 20:43, 2 September 2007 (UTC)[reply]
And note that some tree sap is valuable for other reasons, like sugar maple sap used to make maple syrup and rubber tree sap used to make natural rubber. StuRat 01:45, 3 September 2007 (UTC)[reply]
Wood alcohol is a common name for methanol, so I presume that trees are more suited to the production of simple, small hydrocarbon fuels. Alternatively, the wood itself is already used as a fuel, in the form of firewood, charcoal, etc. Nimur 02:51, 3 September 2007 (UTC)[reply]

I think you need to look at biofuels, in general any organic matter can be turned into fuel - the important part is the quantity - and it's unlikely that tree sap itself would provide enough volume - what's more likely is to use the whole tree itself as a resource.87.102.47.218 08:59, 3 September 2007 (UTC)[reply]

Staying up late

Why, after staying up to 5 or 6 AM, I can wake up very easily at regular hours in the morning, and only start feeling sleepy late in the afternoon? JIP | Talk 17:58, 2 September 2007 (UTC)[reply]

The first part of your question seems easy to me: your circadian clock hasn't had time to readjust to your new schedule, so it wakes you up at the usual hour. The second part of your question, though, is a mystery to me. I don't think I've ever heard an explanation of why or when we experience somnolence in response to sleep deprivation. --Allen 18:09, 2 September 2007 (UTC)[reply]

pressure calculation

If a vortex of air was moving at 160 meters a second and with an energy of around 600 joules, what kind of pressure would it put on an object and how far could it displace an flate surface weighing around 75kgs? Does anyone know the way to work this out? Thanks —Preceding unsigned comment added by 80.6.35.136 (talk) 20:02, 2 September 2007 (UTC)[reply]

As posed, the question is tricky to impossible to answer. What size and shape is the object? I suppose you could assume all the kinetic energy from the air was transformed into kinetic energy possessed by the object, with 100% efficiency, but that doesn't seem a sensible assumption in most circumstances. If that were the case, you could use Ek = m v2; kinetic energy is equal to mass times velocity squared. That would give you the initial velocity of the object anyway. I suppose you could use work done = force x distance, with the work done being equal to the energy provided by the air, but then you'd need to know the force. Hmm. More information needed I think. Skittle 22:56, 2 September 2007 (UTC)[reply]

What exactly is localized buckling? deeptrivia (talk) 00:47, 3 September 2007 (UTC)[reply]

It's when a surface that should be flat looks something like this:
           ______ 
→________/       \_________←
This can be caused by many things, such as:
1) Greater thermal expansion of the surface material than the underlying material (or greater thermal contraction of the underlying material), due to a different coefficient of thermal expansion for each material.
2) Compressive forces in the directions shown.
Such buckling may be a warning sign of imminent failure of the object in question. StuRat 01:26, 3 September 2007 (UTC)[reply]
Mathematically, how is it different from the usual buckling? deeptrivia (talk) 02:25, 3 September 2007 (UTC)[reply]
Presumably it's a qualitative description indicating that the fault is localized to a specific area or material defect. Nimur 02:54, 3 September 2007 (UTC)[reply]
(Incidentally, is there a reason why our article on qualitative research exclusively mentions the social sciences?) Nimur 03:02, 3 September 2007 (UTC)[reply]
It's just in a small portion of the structure, versus over it's entire length. A local defect, such as partial delamination of the layers, may be why you get localized buckling instead of buckling over the entire length. Localized buckling can actually be worse, as the forces act on a small portion of the structure, which may fracture, while, if they acted on the entire structure, it could withstand those forces. StuRat 02:57, 3 September 2007 (UTC)[reply]

I guess both localized and "global" buckling would correspond to a singularity in the stiffness operator of the system. Is there a way, then, to tell one from the other looking at the bifurcations of the system? deeptrivia (talk) 04:40, 3 September 2007 (UTC)[reply]

Numbness in foot

I dropped one of those metal folding chairs on my foot about 3 weeks ago, it hurt a lot and I was yelling, but as far as I could tell nothing broke and there was no swelling. Now its numb and I get this weird tingly feeling when I touch that area, does anyone know what happened? Did I damage some nerves or something? Is it going to be normal again? —Preceding unsigned comment added by 76.167.145.55 (talk) 02:00, 3 September 2007 (UTC)[reply]

You should really see a doctor. Wikipedia cannot provide medical advice. Someguy1221 02:06, 3 September 2007 (UTC)[reply]
I agree, see a doctor, that sounds serious (perhaps a blood clot, which, if it were to break loose, could be life-threatening). StuRat 02:35, 3 September 2007 (UTC)[reply]
I think/hope you mean "I agree, see a doctor, I'm not going to speculate on medical issues" Aaadddaaammm 05:14, 3 September 2007 (UTC)[reply]
Actually, I've decided to indulge our OP just a little bit, and point him to our section on chronic numbness. But let me assure you that just about everything listed under "Causes" is reason to see a doctor as soon as possible. Someguy1221 05:39, 3 September 2007 (UTC)[reply]
Stop giving medical advice! Chronic usually means the time course is >3 months, a "wierd tingly feeling" could feel quite different to pins and needles. In cases like these you say nothing but "please see a doctor", isn't that the policy? Aaadddaaammm 05:54, 3 September 2007 (UTC)[reply]
The policy is to not attempt to diagnose the OP, or recommend solutions to his medical issues, aside from informing him to see a doctor. Proving general information on medical conditions, or showing him where to find more information, is entirely within policy. And if anything I said is medical advice, so is saying chronic implies > 3 months ;-) Someguy1221 06:01, 3 September 2007 (UTC)[reply]
I had something similar when I bumped my foot into something, which resulted in the sort of pain one feels when hitting a nerve. Indeed, that's what it was, so my doctor told me. Except that normally this pain subsides within a minute. This one took about half an hour. Turns out that nerve is permanently damaged and remains very painful to this day (about ten years later). The doctor told me there was a possibility that it could have been more dangerous (can't remember how), but that was luckily not the case with me. DirkvdM 09:32, 3 September 2007 (UTC)[reply]

Amalgam Illness

What is the incidence of mercury poisoning in the United States? —Preceding unsigned comment added by 151.204.143.134 (talk) 02:11, 3 September 2007 (UTC)[reply]

"Mercury poisoning is listed as a "rare disease" by the Office of Rare Diseases (ORD) of the National Institutes of Health (NIH). [22] This means that Mercury poisoning, or a subtype of Mercury poisoning, affects less than 200,000 people in the US population" [23] Rockpocket 02:44, 3 September 2007 (UTC)[reply]
Given the title of this question you may also be referring to the Dental amalgam controversy. As best I know the general scientific consensus is that amalgam use has no significant side-effects in most cases. However this article gives a boatload of potentially useful references. --jjron 08:03, 3 September 2007 (UTC)[reply]

Entomology Question: Identifying Insects

Does anyone know what species these insects are? [24], [25], [26]. The pictures were taken in a backyard in a suburb of Southern California. —Preceding unsigned comment added by Schmoov (talkcontribs) 04:31, 3 September 2007 (UTC)[reply]

Pepper remains in the system for about 7 years...

Over the years I've heard a couple of times from different people that apparently pepper stays in the system for about 7 years- that it doesn't get digested or something.
There's nothing on snopes.com, theres nothing in Black pepper or its talk page, and nothing comes up on google - is there any truth to this claim? Can pepper be eaten liberally? Rfwoolf 08:52, 3 September 2007 (UTC)[reply]